Anda di halaman 1dari 128

BHUSHAN, MD

Liilivrmirv r,l (:alilnl nia, San Fr ;~ncirt.o, (:laah of 1!1!11 St-rim Editor.
U i , t p o r t i c l ~ d i m l o ~ . i i
W S W PALL, MBBS
Griv~1.nme111 Clcdirnl (:r)llrgr, I:hai~rligni-h,rndin, C:las?;of 1'39fi Srric~
Eclitnr. L l , nfTcxx5. I;aIvrrton. Krridcrlt in Illlerilal Medicine R: 1'1-evrn
tike Medicirir
TAO LE, MD
Linimr.;im t,r Chliforni;~,San Fraucircr,. Class elf 1 996
JOSE M. EIERRO, MD
La S a l k Llnivenih. Slesico Cjlv
HOANG NGIMEN, MD, Ml3A
Blackwell
Science
COYTSIBUTORS
Tisha Wang Ll n i\.ersi of Texas hlrdiral Rran cl-t, Class of 2002 tv Fadi Abtr
Shahin, MD d'nixersi ty {IS D a ~ n x s r ~Svrb, Class ol' 11r!)C3 ~s,
Vipal Soni, MD IT(:J,ASchool of Medicirie. Class of 19!N
Editorial Offices: C:onimt.rrt, Pl;~r:c.3.5ll Main Strcct. 3J;1irlt.n. hln.;u;tc
liris~rts(Y21.Ig. USA Owe., Mcacl, n ~ r i ~ rOX!? OEL, 11lgl;lntl tl 2?>JoIi11S
~ r r r L.nnrlon W(:l N 'IRS. Englantl ~, 25 ,linslir pl:~r.c. E d i l l h n ~
- ~ h f 3 tiAJ. Scotl:~nrl El 34 LTnivcrsiL~ t r e c , ~ ~ S (>~i-ltr>ii, lri<~l
orii* !5U53. Auhl~ali;~
Dcrrclr,prnent: ~ \ l n r . h'~'r~trhrtjck Prtlrlurrinri: In,rnn Hind arlcl Shaw
n C:il.~hexcr
Illantifarturin~:I.iw Flarlawn M:irk~liiig hhn;~gc;cr': li;lrl~ll-rnMtilcahy I:t
limcrr I r . \ i q hv l:.cslit- Hainlcls In!erir,r t l r s i p i I,? S11:lwn Gin
ljcrlgcr T\pt.sct t>v TccliDrrt~ks PI-iritccl:~i-tcl trr~undt ~ v C:apital Citu
Presk
Blackwell'9 Undeqmund Clinical Vignettes: Other Editorial Offices: Biochemistry,
3c l3lacku't.ll ITi~uc.~i~cl~al~+\'~.rI~t~ Cirnhl-I. IS~K (M;:VJ).I? 35-4) Ku~-
Iitt.itr.utl;ttiirr~ 57. Ill707 I3ct lin. C:crmanv Statrc nI'r\meriri~ ~~l~,--~~
.ll +lcnre ~ xw{; ~ ; ~ , ~~ l ~ j ~l ~ lPrinted i . ~, c ~ ~ . ~ ~ ~ ~ ~~ ~ l r
h Llrl~tr-tl i (Y2 03 04 0.5 5 4 .,? 2 1 7-1 0 E(trrirnm:+r o N i l l r ~ n ~ l
~ i s('I~LIu-ku. h tii. 'Ibkvo 10-1. J a p n l ~ -l Ulai kwell Srirnrhr T n p ~
a tr,irlt. [nark 01' llc i.; Ic>rua S t ~ t C'nix-cmiw I"rrc<, \ l3larku.t-11 S
( i r r u t. r Scirnrr I.~rl-,rt.k+te~.crF a i 1 1 1r Llnitrtl ~.ompnnv, 2 1 y 1
q. s , . , ~ ~ .\,-c.nllr. A ~ I ( ~ ~ ~ I ~. I n r k ~ ~ r I 1 , liin~rlom Trd
dr hiarks R c ~ s ~ n ; 501114-87110. L'S:2
Library of ConCatalogingin-PuhticationData n1111slla11. \.'ilias. B l a c k ~ v
~ l l1indrripnlnr4 cliriic:~lvig~wucs. 's Rinrhrmistw / authr~r. Vikiks Bllushan
. - 3rd etl. JJ. : CIII.- ( U ~ ~ r l r r g r o r rliniral vipettr?;) ~nd Rrv. r
rl. or: R i r ~ ~ h ~ n / iVikils ~ t ~ t r Hlii~sl!;ln [et a!.]. 31rl ctl. c l
!lI1!4. ISHN &652*f45:t5-{l .( phk.) 1. Ilioc4ie~nisln. T.;ives~urlies. Pllwicia
r~s 2. 1,irrnsrr - Urritrbd 51,rrc.; - I:samin;lrinns S~ttdv ~flirles. [ D N I N
: 1 . Bior11c111istt.r. 1;;lcr Krpon. 2 , BRinchr~nisrt~ Prr>hlrms anrl Exel-ci
hca. QLl 1H.2 I357513 2I)0?] I. TitIc.: Lt~r<lcrgro~incl clinir;tl v i ~ m r t t
r s .Rit,rl~rinis~ry. Tiilr: Diouhc~nictm 11. 111. Biort~elnict?. it;. Tiitlc.
\: Series. IZRI IY.5 .R4H 2002 .372.%'074klc21
...
1\11 rigl'lc rcwrvt.rl. NII 11;11.1 111 ~ h j hnrk Inxy bhr s rrprr~rlurerlin a
n v C o r n rlr h v :IIrv t.lr.ct~.or~ic or ~ n ~ c h a n i < :Incatts. includin
g inl'ol.mation stnl-agv ul nrltl rrtricvnl svstcms, ~ \ i t h n l ~ rr n n i ~
~ i n n r\.riliirp; p in h-nm t h e prrlh1isht.r. r x c r p l by ;I rr.vicwc.r w
ho may qi~iiw i~ricl- > X S S ~ it]~ . rrvic!~,. I R+ F ;
Notice
-111~~ t h u l rK t l ~ i s si .I r.olunlc. II;I\,cr:tkcn cart tlrai thc i!jlor~
na!ion cot~taincd ~ e r c i n acrtlra1c: anrl compa~it ic hlr with the sranrlart
ls g m r m l l r at-rrptcrl ar [ h e timr rdp~it~licalir,n. Ne~er~hcFess. is dif
fictil~ eIls1rI.c it LO 111,rt all thc inftrlrr~atic>~~ i\ crltir-clvaccilratc I
'or ,111 circurnstanccs. 'l-hc puhlislier a n d nrirhnrs do nor gi\rn guarantcr
1111: conrent< 01 111is honk ar~rldiqclnin~a n \ liahili tv, IOSF, nr rlarnaqr i
nc~rrrerla s a rniiseqlwnre, dircctlv o r inrlil-rrtlv, r > f I l ~ t .llse anrl
:~l,plir.atir~ii any of ~ h ctmtrrus of rhi?v r ~ l ~ r ~ n c . of r
CONTENTS
Acknowledgments Preface t o the 3rd Edition Haw t o Use This Book
Abbrwiations
x xiii xvi
Xvii
Endocrinology
17-Alpha-HydroxylaseDeficiency 5-Alpha-Reductase Deficieny Acrorneg aly
Addison's Disease Bartter's Syndrome Congenital Adrenal Hyperplasia Diabetes Ins
ipidus Diabetic Ketoacidosis Glucagonorna Birsutism-Idiopathic Hyperaldosteronis
m-mimay Hypercalcemia Hyperparathyroidism-Primary Hyperthyroidism (Graves' Disea
se) Hypocalcemia from Pancreatitis Hypokalemia Bypomagnesernia Hyponatremia Insu
horna Kallmann's Syndrome Kwashiorkor Lactic Acidosis Metabolic Alkalosis Nonket
otic Hyperasmolar Coma Osteomalada Pheochromocytoma Precocious Puberty Pseudohyp
oparathyraidism Rickets SLADH Thyroid Storm Vitamin A Deficiency Vitamin B, Defi
ciency (Beriberi} Vitamin B, Deficiency (Pellagra) VitalniPI C Deficiency (Scurv
y) Vitamin K Deficiency Carcinoid Syndrome Crigler-Najjar Syndrome
Gastroenterology
Dubin-Johnson Syndrome
Genetics
Gilbert's Disease Wilson's Disease Acute Intermittent Porphyria Albinism Alkapto
nuria
Cystic Fibrosis
Cystinuria Ehlers-Danlos Syndrome Fabry's Disease Familial Hypercholesterolemia
Familial Hypertriglyceridemia
Fanconi's Anemia Fragile X Syndrome Galactosemia Gaucheis Disease Hereditary Fru
ctose Intolerance Ho rnocystinuria
Hunter's Disease
Hurler's Disease Kartagener's Syndrome ninefetter's Syndrome Krabbe's Disease Le
sch-Nyhan Syndrome Maple Syrup Urine Disease Metachromatic Leukodystrophy Nieman
n-Pick Disease Orotic Aciduria Phenylketonuria (PKIT) Phosphoenolpyruvate Carbox
yhase Deficiency
Pornpe's Disease
Parphyria Cutanea Tarda
Pyruvate l n a s e Deficiency Tay-Sachs Disease yon Gierke's Disease
Xeroderma Pigmentosum Anemia-Folate Deficiency Anemia-Iron Deficiency Anernia-Vi
tamin B, Deficiency , Glucose-6-Phosphate Dehydrogenase Deficiency Hemophilia, T
ype A Heredita y Spherocytosis
Neph~oIogy/Urology
Neurology
Gynecology
Orthopedics Psychopharmacology PuImonasy
Methernoglobinemia 51aroxysmal Nocturnal Hernoglobinuria Renal Tubular Acidosis
Rhabdomyolysis Duchenne's Muscular Dystrophy Wernicke-Korsakoff Syndrome Primary
Amenorrhea-Testicular Feminization Secondary Amenorrhea Osteogenesis Imperfecta
Antabuse Effect Hyaline Membrane Disease Alpha-I-Antitrypsin Deficiency Hyperve
ntilation Carbon Monoxide Poisoning Gout
Tlir-oughout ttlc p ~ - ~ ~ d r ~ cofi u i i hunk, wu Ilave hid the s u p t this
porl 9f many fricnrls a n d collcagllcs. Spc-cia1 hanks lo our 91113purl ream i
nclutliiig A I ~ I Gupta, Andred Fellow%, Aii;t?tasia t2r1clcrso11,Srisl )ti Gup
m, Mona Pal!, Jonathan Kirsch anrl Chimg Atni t i . For prior co~-irrihutioi-is
thank Gianni Le Ngu!.cn, rue T a t ~ ~ na ~ h u rAles Grirnm, Sonia Sanros xncl
Eli7;lheth M ,
S;~~IC~UI'S.
IVc hh;~\-c cqjoycd working w i t h a wr~rldcfass international p ~ t h lishi13g
group at Blackwell Science, ii~cll~rling L.allm DeYoung, Ainy K ~ ~ f t b r o c
Lisa R;111;tgai1. Shawl1 C;~~S~IPI.~PI; Imma I-Tind k, and Gorrlon Tihhitr9. Fo
r h e l p with c r r ~ ~ r i nimages fnr t h e gentire serier we :dsn thank Lce
Martin, Kri~tc3pl1rr.Jr~11cs, Tina Panirri and Pctc1-Anclcrsrm at thc Llr~ivrrsi
??' Aliiharna, the of .h-~n'cl FUICCS Irlstitlite o F a t l ~ ~ ~ l and m ~ n y
nf oilr f ~ l l o w f c>~ya Rlackivell Scirnce ;i~ithorq.
For submitting cotnn-ients, corrections. editing, proofrr=ading, ;~rlcI assistan
cr across a11 of the ~ i c : . r ~ r tillel: in all rrlitinns, WP ttr collective
ly ttiiliik:
T<~i-a .AdaniovicE~, a r c ~ l ~ n C Alcx:j~lder,K1i.s Alclrn, H e i ~ r v 1.:.
i\s-yn. l A v t l ~ n Racnlr>~; a ~ a l n a r k n e n , Dean Baltholomrw, ;l~~ N
i~ Drhaslrisl~ Behem. Slumit Bhatia. Sanjav Bindrn, Dave Brin tori, Jriliannr B
r o ~ mhlr-sander Brormic, Ti~rnan~ , Callal~;in. 13avirl Canru, 131-i~anCmey,
Aaroll <:a~~ghry, Helw ri Chen,,rnt-rathau Cheng, ,4ruold Chcung, , h l r l l d
Chin. Simioil Chiosca, Yor~rlCho, S a ~ r l u c lClrung, Gr-rcchcn C o n i ~ ~ ~
t , Vladin~ir Curic. Christopher C o s g o ~ . eRonald Cowa~i. , liarekin R. Cl
inninghar~l. Sean A. IEalley. Ramn Danrlainrldi, S u n i ~ Dns, Kyan hr-n-ixnrlo
Dave.,]ohn Di~vi~l, Emmanucl ( 1 ~I Cnlz, Rohcrt DuMullo. NmfrwetDhillon , 'a Sh
al-~nila 1lir;sanaikr. I3wirl I ) o i l s o ~Aclnlr Etrhegaray, ,4lea ~, K ~ ~ s
r h iPriscilla A. Frare, David F r e n ~ 1Cristin fiau~ner. o, . Yohanncs Gchrc
cgziabhcr. Anil Gchi. ronv Crcorge, L.M. C ; o t i ~ ~ l r * o Pai.111Gc~yal, l
r ~ . R Iii-imm, Kajeev Gupta, ,ZEirnad I-Ialim. Sue IInll, L h i d Ilassclbachu
r. Ermra Hciniert. Micl~ellr I-liglcy Dan I-Ioit. Eric-Fickssn. Ti171- J ~ I c ~
s I > ~ . Jayara~nall. Sufldar Pei-Ni J t ~ ~ i rAar-than -Joshi. Rdjii i R. j
~ ll;~.F;iiva;l h~>acli, Sell] . t 1hi-p. Ral-or1 S.Kpssrlheitn, k ~ n Khan, I\~
~ch-ew a Pin-~vci ICo. Francis ICong, Pal11 Konitzky, \4'a1-ruilS. Ibackov,Rcr~j
arnitiH.S. L a ~ it, l ~ l r lLaC;lscc. Collnir t e e , S C O ~ Lee, G i t i l l
e n ~ ~ o L Lehmann. Krvin I , e i ~ rP;luI ,Lrver 1. N ~ r r e n 1 .evi nson,
Erir Ley, Ken Lin. ~~
Payel Lobanov, ,J. Mill-k Macldox. Xriun M;~rclian, S;imi r Meh la. Gil Melmcd.J
oe Mcssina. Robert Moqca. M i d i i l e l Murphy Vivek Nanrlkar-ni, Siv:~ Nal-av
nan, I:aivt=Il Ngityen, Lit111 Nguven. Deanna Nohle7a, Craig Nodurfr, George Kou
rni, D;iri~iT. O k ~ ~ r l.ld:irn Id. Pnl ance. P:1111 Pamphl-r~s. a. =Jinha Par
k. Sonny Pati*I.Ricarrlc, Pictl-c>l>ou. i ~ a Rahl. Xashita Randeria, R L. R;ich
an Rcrldv. Beatrill Rcig, ?ularilou R c y c ~Jcrcmy Richmon, . Tti Roc., Rick Ro
llci; R;tjiv Rn): Dirgo R u b . .,Z~~tlir,ny 12usscll. Sailj:~~ Snhgxl. Un-rlima
la Sarkar. J o h n S c l l i l l i n ~ . ~ a h r l Schmitt, I l Dxr-rrl S c h ~
~ h m a c I i r r , Sona? Shah. Fadi A 1 z Shahin. Mae SIleikh11 AIi, ICrlie S h
e n . ~ l ~ ~ s8rnith.John Stl~l:~k, tin Lilliat~ Su,Jiilic Sli ~lrlmanl, Rita
Sru-i , Sei I1 Sweetsel; r \ n t o ~ -lalayrro, Merita ~~o Tain. Mark Tanaka. Er
ic Taylor. J e s ~ 'Tl~onlpsnn,Tnrli Trchan. hvmnnd T~~rnrr, Okafb h'chenn:~,Eri
c Uvppanct). Ritlia V ; ~ r n ~ a , J o 'tlrqcs.Alan Mr:zng, E ~ ~ n i c t . ht~
Ilang, ,411rll; F+'riss, Amy WilJiarr~\,R t i i ~ yjrig, Haiiy Ziby, ,-\sl~i,;~
f r~ %ar~i;~ri nnvirl Xipf. ;wrI For yenel-o~~slv cnntril,trrin~irnages t o the
m i i r e [ ' t > d / ~ ~ ~ l l ? 7 l f ('linzrnl Vl,gm~ft~ 1 serics. we cnllecr
ively thank the staff at Stcp BlackwcI1 Scicnce in Oxfor-cl. B o ~ t o nanrl Bcr
lin ar w-rll as: .
Ran ni\rer- R, Reg,g N, C;illeqpir S. In/~rriorrr Ilioom~, dE(iiti07z. 2
O~ncv \lilr:~d: BlacL~veilScicncc Llcl. 2000. Fig~rrm 2.8. 3.4, .i.',X, 1X.10, W
5.32,115.6.
Be]-g D. Arli.ln?~r~r/ CIi71rcrrlSkiSh n ~ ~ r l P ~ ; y v i r n l D r ' ~ ~ p ~
o r i , r . Rl;lckwcll Sciellrc Ltd., II)Y!). Fijiurrs 7.10, 7.12. 7.1 3, 7.2.
7.3. * ~ . I , T . H . T . ! ~ . X . ~ . H . ~ . X . ~ , X . . ~ . ! ~ . ~ , ~ O
. ~ , ~ ~ 1Z.I;. I . . ~ , .~.T
-
Cuschic~i 1 Irnncssy TyJ,Grccn halgh Wl. Rowley DA. A. Gr;icc PA. C:Iin/rnr.1'1m
y1?,) X I I ~ ~ R I ~ I c ~ \ ~ ' P I I C Mead: SC~PIICP Lid. 1996. Figtires 13.
19. 1X.29. 1X.?~3.
(;iliey>ie SH. Fk~~i\fi>rd ; l f ~ ~ ~ / i ~ r r / . ~ f i f?~fit,tion i (I l o
q ~ ~ ~ ~ ~ ~ l T L ~ v ~ ~ ~tt/ ~ C;ln?rr~. CI.i~luv Mcacl: Bl:~ckwcllScicnce L
,tcl. !?000. Fip11.r~ 23. 20,
Etliotf T, Hastings M,Drs.;c.lbt.rgrx Ll. I,rr!urr.;\i)tt~\ IVI~tlira/ nrt J21ir
mhir~lr~~. 3"' I;t?itiot~. C3snev hlearl: !3lacku,elI Science Ltd. 19137.Fifi-ti
rer2.5,7.8,1), 11, 12, 14, 15, I f i , 17, 19,20,25,2Fi, 27. 29. SO. 34.35. 52.
*
Mehta An, Hotfirand .W. H u m n t c ~ ? rrt~ GItlnree.Osnev o rr Mcad: Blackwell
Science Ltd, 2000. Figures 22.1, 22.2, 22.3.
Plrasc lct us know if your name has been missed o r misspellcd and tue will hc h
appy to make the updatc i thc next edilion. n
PREFACE TO THE 3RD EDITION
Mk were very pleased with rlw o\~~rwhelmii~gIv positive s t ~ ~ d e n t feedback
for t h e 3nd edition of nlir I 'ndqq-mrnd I,'?inirnl IJip~ftes wries. Well ove
r 100,000 C O I ~ ~ Pof the UCI' b o o b arc in prim F
and havc been uscd hy qhldcnts all over
t 1 world. 1r
Orrr the last two wars 14-e h a ~ e acculntila~ecl and incorporaterl over a thou
sand 'updates" and improvements s~iggrs~ed vno, hv olir reatle1-s,incltiding:
many additions of specific boards and wards t r ~ t a h l c content
de1etion.s of red~zndant and overlapping cases
reordrsing ;md reorg~i1i7ation f all ewes in both series o
a n e w mxster iildcx by case narnc- i r each Atlas ~
correctinn of'a few factua'f crrurs
diagnnqis and trcatrnent uprlatrs
-
* addition of.5-"0 n r w cafe? i t 3
/tnn/nntj
every honk
and he additinn orclitlical exam photographs within IrCV-
And mnst important of all, the thircl cdition scts now include hvn brand new COL
OR A W supplcmen ts, one for each Clinical Vignefte serieq.
The Ir(,'I'Rn?ir S r i m w <,'olor/ltlr~.~ (St$ I ) includcs over 250 color plat
es, divided into gross pa tho lo^, iiiicrc~scopic patl~oloqy (histolo%), hetnarn
lnp, and tnirmhinlogv (s~ncars) .
* The C7Cd'I;-UltnLnlS(-iMlrp C,'n/or.~iilrtt(Siq')2) har owr 125 color
plates, incltiding patient images, dcrnrarcllngy, atjrl iilnclus-
ropy.
Exclr ;~tlas image i s d e s ~ r i p t i v captioned and li nkerl to its cor~l~
responding Step 1 case, S ~ e p "use. a n d / o r Step 2 MiniCasc.
How Atlas Links Work:
Step 2 Book Codes are: Step 1 Book Codes are: A = kb;trr)niy I35 = Rrlia\-i-ic~i
:~l Scirnt.? BC: = Rir,ch~miqtry $11 = R,lirrohinlnf13tkrl. I M'? = Mic,i-trhinl
n~y. 11 \ill. 1'1 = I'a~h.rtpl~vsirhc~p~, I fbl. 1 = I'i~tlinphy~iolo~, I1 2 ' \
!ol. i:'E = P ; u l ~ ~ ~ p h v s i r ~Vnl. v , l ~ g IrI PI-I = P ~ I ; I ~ [ I
I ; I c c > ~ L ~ ~
Case Number
ER = E r n r r ~ p n r v kl~dirinr lhll = It~rcl.nat Mcrlicinr. 1'01. I IM2 = Ir
lscr~~al Medicine. Ird.I1 NELT s e ~ r r ( ~ l o ~ ~ i>n = C)B/GYE P6D = PcrIial
rics SUR = S11rgt.r). 1'51' = I ' y c l ~ i a ~ i - v MC; = MiniCiasr
-
Indicates Type af Image: 1 I
\,7
/
= =
?/ A
IIa.~~i,tto!~>p
\
Indicates UCYl or UCV2 Series
kIifr-c>kiul[>~ PC; - (;r-oss P.i~hulogPRI = hJicrr*scvpic- ; ~ t h o l o ~ P
IF lllc Caw nun~bel(0.3'2,035, etc.) i q nor rollo~vedI a lertet; w thrn f1er.r
is u r ~ l y uric. imagc. Otllen+isrA. B. C . D indicate up to 4 imxgrs. Bold Fa
ced Links: In order to f i r e vou access in the largest nl~mhpi- irnagc-s possi
l~l~, havc clioscn to cross link the 06' we Stt-p 1 anrl 2 wries.
If the link i q holrI-raced 121is indicates thar tile link i5 direct (i.r..Stcp
1 Case with 1 1 Basic Science Slep I Atla< link). Ee
If' r l ~ e litik i s rial I>olrl-faccclthis inclicarcs that the link i s inclir
ect (Step I rase ~ z , i t l iClinical S c i m c r Slep 2 Atlas link c l r ~ i c
r VCTS~L).
We h w e ; ~ l s z ~ irnplernrr ~ t e d few stn~cturalchanges upon vour a scqucs
t:
* Essh c~tz-re11a i d f i r~ edi lion r)f L 11r.e
0111-
popidlar Fi7rl A id fur
tltr S:Si\fIly .Ti+ I (AppIeton 8c l,nnge;P/McE;raw-Hi11) anrl 1 ; i ~ Aid fir /
/tp I <VI~.II,E .TI+ 2 (AppIrtor~ LZ;lngr/McC;raw-TIill) k hr)ok ivill bc linked
to the corre~ponrlirrg UCL' caw.
lye eliminaterl UCV + E'irr;~,\id lirlkv as they freqt~enllv b r t o ~ out o f d
ate, as the Firsr Aid hooks are revised veal-ly. i~~
Tllc Cynlnr Atlas i 5 alw ql~ecinllv rleqignerl for. q ~ ~ i z z i n g raptions
:Ire rlesrl-iptivv and do not give m r a v t l l mw n a m e ~ d i rec tlv.
IVp I ~ q p c h c npclalurI UC3; scries will 1-emiii11a uniqrre and ruellt illte
gratci1 h l t ~ d ! .tocll that pr-nvides compacr clinic<~l cnl-relations tu h;~
sic sciuncc irafc~nnatioi~. e v arc. rlrsijincd Irl he e a y and Th fiin (con~p:
~~ativt.lv) roxl. aurl hrlplul tor I>OI l i r e n ~ng tv 1 1 i psatnq a n d thc
warr1.s.
1t.r inlitc vtmr corrcctior~s n d Y L I ~ ~ ~ S ~ for 111t%f~ ~ i ~ r terlia ~ C
) I I c li zi(v11 01. t t ~ e s e hooks. Fat- the I i t s t s ~ ~ l r t r ~ i w
n oeach Cactual cosil r~ I-eciion rrr new ~ifi-iiette thnl is celertpd Ibr itlcl
~rsionin the i'ourt EI cdi tion, vou \\ill rccei17e a pcr4sonalacki~otvlerlgrnrr
l in t the r e r i ~ e d lmok. If vou submit ovcr 20 IligE1-q1iali1~ col-rcclicl
ns, ;~rlclitio~ls n r w vigr~uttus v c a l w C C I I I ~ inviting you re or ~ wi
ll ~~PI. become a "Conbibutor"oon the book o f your choice. If vnu are i11 t ~ s
e s r e r l in I ~ e c n n ~ i n g a prhential ''Cont~ih~itnr" "htitlior" or o
n n fltture UCjr hook, or working \+it11 o u r tr;1111 ill c l c v c l o p i ~ ~
g a~lclitional hot)ks. ~ I C ~ S ;11so e-mail us your CL'/rrrmnr. P Ute prel'el
- that \-nu s ~ t h m i corrections u s suggestions tcn t ii plrctro~~ic mail to
UICVtcam@yahoo.com.Plcasr i n c l ~ l d e
rin not Imve access t o e-nlail, usr t h e following mail in^ address: Rlackwc!l
P~thlishing, Attri: LrC3' Editorr;, 3.50 kri~iiiStreet, h[aldt~ii, hlr\ 02 148,
LtSA.
HOW TO USE THIS BOOK
This series was originally developed lo address the increasing number of cIinica
l vignette question.; on medical exami~la~ions, including the USMLE Step 1 and S
tep 2. It is also designed to svpplement and complemenl the popular Fin! Aid,fo+
i h p ILYMIJX Step I (Appleton 8c Langc/McCra~vHill) and Fimt Aid for /IIP I r
S M i L Sl$) 2 (.2ppIeton Rc Lange/McGra~vI-Iill) .
Each UCV 1 hook u s e s a series of approximatvly 100 "supraprototypical" cases
as a way to condense testable facts and a-sociations. The clinicaI vi<gnettes th
is series are designed to in incorporate as many testable facb as possible into
a cohesivc and tnenlorahle clinical picture. The vignetles represent composites
drawn from general and specialty textbooks. reference books, thousands of U S M
U style questions and the personal experience of he authors and revicwcrs.
Although each case tends to present all t h e s i p s , symptoms. and diagnostic
finding.5 .For a particular illness, patien& generally will not present with su
ch a "completenpicture either clinically or on a medical examination.Cases are n
ot meant to simulate a potential real patient or an exam vigne~le, the All boldf
aced "b~urzwords'~ for learning purposes and are not are necrssarily expected to
be found in any onc patient with the
discasc.
Definitions of selected important lerms are placed within the vignettes in (su.~
~.r. s ) in parentheses. Other parenthetical cw remarks often refer to the patho
physiology or mechanism of disease. The formal should also help students learn t
o present cases succinctly during oral "hullet" present;ltions 011 clinical rora
riot~s. The cases are meant to serve as a cor-ldrn.seilreview, not as a prima? r
eference. The information provided in this book has bcen prepared with a great d
eal o f thought and careT1 I-esearch. This hook should not, however, hc considcr
ed as u yurlr solc source of infnmatinn. Corrections, suggestions and submission
s of new cases are encourager1 and will h r acknowledged and incorporated when a
ppropriate in f11h11-e editions.
5-ASA &G,s
ARWI
5-aminosalic~lic acid arterial hlood gasc.; adriarnvcin /hlroml;cin /vincr-i.;ti
nc/dat;t~.hwine
ACE
xcm-I
ADH AFP AT iZTDS :Il-L A1 ,T
AVI.
ANA ' W S
rlSD
AS0 .%ST
an tidiuretir hormone alpha fetal 131-nrein aortic in.;ufficiency accluired imrn
u~lodeficirnc syndrome acute Iyinpl~ocyl le~zkemia ic alanine transarni tlase ac
ute mvelnqenm~s leukemia antinuclear antibody arlltlt rctspira~on distrecs s\.nd
rnme illrial septa1 defect anti-strrptolysin 0
aspartate transaminasc
A\' RE BP BUN
arturiovcnuus
harium enema
blood pressure blood urea notrogen corona? atten; disease commclrt ac~iLtt y ~ ~
p h o h l a ~ t i c l leukctnia anti~eien coinplete hlood cozlnt <I1 congestive
heart f. 'I ill-P creatine k i n a ~ e rhroilic Iymphocvtic leukemia chronic my
clogcnous lcukcmia c?~nn~~galo\inls central nervous systeIll
c ; m
CALL4 CBC CHF CK
CXL
CML C3IV CNS COPD CPK CSF CT
CVak
c h m r ~ i c ~ ~ ~ I - I I C II I~I V ]PI ~ T I O Idisease O I )~I~
cr~atine phnsphokinase cerehrnspinal flnid cr nnptited loinop;lphv
rea-ehrnvasc~~lar accident chest x-rav
CXR
DIC DIP
rlissen~inatrdin Lra~~sculalr-nag~~lalion
D M
DM D'TKs DIT
deep tendon reflexes clccp venous thromho.iis
Ep~tt=in-Ertr~virus cIcctrt~c;ii-cliogra~>li'~' ~chr~carrliog~phy ejcction fract
ion e~opliagog;~st I-oduocienosropy elect roinvogl-aphy endoscopic t.etrograde c
holai~giol~a~zcrcatogl-aphy eryi hrocyre sedimciltatic~nrate forcer1 pira rail l
ory volunlu Iinc neecllc aspir;~tiorj FNil r rwponemal an tihocly ahsorption T;T
X-'-mS flt~orc=\cen ft,rcprl vital capacity R'C glomenllar filtration r , ~ l r
GFR GM p;rou~h Ilornlonc ~~,~troinrruliital GI yte GM-CSF ~ ~ a n l ~ l o crnaci
-nphage colony stirnulatirlg
E R17 ECG Echo E:F EGD EMG ERCP ESR FKV
Ijlclol'
GU H IZV 1lc.C I IEENT 1K 3
1-1
gr t~i~ourinal-y hepatitis A virus
u
FIPI HR IHRTC:
11s
ID/CC: IDDM Ig IGF IM
.T\;p KUR LUH LES L.FTs I ,P LV LVH
Lyres
MCHC: M(IT hlEN
human choric~nic gnnarlotrophin h r a d , cyes. ttais, now, anrl throal hn~nan i
~llinunodcficicil~y 11s vir human lurl kctcvle ail r ig~11 1lis1or-yof pr-esen~
illness heat-I I-ate I~~uman rabies immune xlr>hul 11 i hcrecli~;ln) l ~ h t . r
n c y t o ~ i s ~ identilicati on ai1d chirf complaint ins11lin4ependentd i ; t
b u ~ c ~ (11s 111e1li in~munt~glc~l~lilin irls~~litl-li gl-nwth Cat tor ke ill
tralnuscl~lar jt1gul;lr. vrnrllls pressure lricli~eys J~li-ctrr/hladder lactatc
ctchyclro~em;l~r 1nwt.l- esophageal sphincter liver C~~nction tcsts l u m l ~ a
puncture r Irft \.men~ric ular left vcnrr-icular Flypc-rtropli y cIcctroFyles m
ean cnrpuscula~ hemndohin concentration mean cnrpusczllar vulurme muTtiplc el i
c l o c r i t ~ ~ nenl~lasin
91R XI-IL. SII'II)M NFO YSt\â ¬II P!2 PIP FBS PE
PFTs
m! ocarrlial infarction nip<-hlnrrt h:~mii~r/vinci.i~tinr (Oncnvurin)/ prncarha7
.ine/pl-ednisn11~ magnetic resonance (intaging) non-IIoclgkin'< Ivmphoma n n t l
- i n s ~ ~ l i ~ ~ - c l e p c ~dii~hctrs i d c n t t~icllilt~s riil ~ U I os
( ~ ~ ~ t ltlv i n g ' ~ II~OIIIII) nclnsipl-nidal ;rn ti-inflamilla t n r drug
~
PbII
P 31hr
PT PTC : I : PTI-I
prnximal intc~ph:il;ingcal pctiphcral l~lr~otl .;n\ear pllysical r x a m p1iltnn
na1-yl'~1nctiot1 rcct.5 point or n~asimal n t c n ~tr. i i pnIrmnrpl~nnr~clcar k
o c ~ t e lcu
m
PL'D KHC RPR
Kti
rapid p I ; ~ ~ m n wagit1
respirarn1-y sate Rced-Sternherg (cell) rig11t vcri~rirr~lar righc r r n t r i r
~ ~ l a r I~rperti-ophv small ho~vcl F~Ilow-througl~ s v ~ ~ d r o r nori*inapp
ropriate svcrpt inn of ,kDH systemic 1 i11>i1'$ ~ r y ~ l i ~ m n ~ o ~ l ~ ~ sc
xuallv transmitted di.;u:r.;e thyroid I't~nctiontests 1 i';stlr plasminogrn acti
wlor tI~proicl-stirnuIating horinone irltal iroil-hi~lrlingraparitv I ransi~lgul
ar intrahcpatic porlosv.;lcinic s l ~ t l n I
RS
RY Tt\,Tl SRFT S1AI)I I SIX STD
Trn IP,~
'I'SH TIBC TI t3S TPO TSI-I TI-P LL'4 LTG1 LJS
VT)KTRT, Venercal Disease Research Laboratory 1:s vi till signs VT ven tt-icular
tachycardia \lrBC while blood cell IVPW W'olff-Parkinson-Whi tc {syndrome) XK x
-my
ID/CC
A lhsear-old girl is referred ro an endocrinnIogirt owing to lack of mem.es ( PR
ILZRY ,LMEKORREIFA) ant1 ahsrrice of puhic hail;
axill;~rv hair. arid breast clrvelt~pmesit{r ,W:K
OF SK<:ONIl4RS SE.:xV.U
c 1 I-\F~TFRISTI~:$).
HPI
Shc also complains of frcqzitnt headaches and ringing in her ears (duc to k~~pcr
ttnsion).
m
i ?
0
u
PE
VS: hypertension {RP I 6O/ln5). PE: f~~nduscopir exam nnrtnal; n o lymphadenopat
hy: no hepatosplenomeplly: a h ~ u n c e uf hl-rast t i w ~ e ; r n al>iloniinaI
or. pelvic marses palpahlc: n o a d r Ian, or. plthir hair; mttvar Iabia normal
.
I 7
z z
0
r0 0
<
Labs
Lyrrq: hy-pokalemia; hvpernntrem ia. ARCS:metabolic alkalosis (dlie to ~nitieral
ocnrt icoid act ion nf I I-rleoxl;costicostcrunr. ;tnd cn1-licosterone) . Sl~ppt
-esserl renin; increasr in urinary gonadotropins (due to attempt to compensate F
or lack of sew horrnours): diminished 17-ketostel-oiils (pl-odiict of sex hnrmon
cs): incrraseil prnges1ernne. p r e g n ~ n o l a n e .1 l - ~ E e o u ) ~ o r t
i c o s t e ~ ~ o n ~ ,
anrl rortiro~ternne.
Treatment Discus5ion
C;ltlcc~cost imitls. Sex tlnrnlot~es.
11decrease in 17a-hydroxylace produccs an increase irt 1Icleoxvcorticosttronc an
cl corticostrronr ( d ~ w ql~ifring e r a l ~ LC) m olisni of %exhormones tn atr
ln~ternne path\-ay): rcnin is SLIPpressed (due to aldostcronc ncgatirw frudbiick
l. Female? Fail 10 dwelop secondarv scxuaI chitracteristics: miller ttevelnp arn
higuouu external ~ e n i t a l i a (MN .F PSFI - n o r ~ r ~ u . ~ ~ r r ~ n n m
s x ~ ) .
17-ALPHA-HYDROXYLASE DEFICIENCY
ID/CC
h neonatal hay is Frrrough t t the pediatrician hy Tlir Cathcr, who o
reccntly rliscowrccl thal hi5 so11 does not urinate through his
penis.
HPI
Thch pa~ienr's a h e r also reports that hc cannot find his son's f testes (clue
t o c r ! ' p ~ o r ~ h j d j ~ I n ) .
P ~ n i small [or aKe (MICR(~PFI.\LLIA); nesles locntcd in inguinal f cannl hiIa
turally (CRYPTORC~ rrnrsn4): tll-iuary meatus lies i r l perh~cuin( H I P O S P
~ ~ I , scrntal sac hifid. ~~):
PE
Labs
Markedly reduced dihydrotestosterone with normal testosterone level. Ducrcased 5
-cr-r.etllic~ase activity.
Prlcic LrS: c r > n f i r ~ ~ l s st-ytol-chidism and al~sent t e n u . 11 Psych
osocial support. Gendor a w i ~ i ~ m e n r . Consider appropriate hormonal rrpl
acemeil~ thel-ally.
Imaging
Treatment
Discussion
autosomal-recessive rlisordcr OF viriIiza lion a f k c t i n g genetic males. 5-
a-rccluct;ur cnnvertf teqlnqtetnne to dihydrotestostcrorie. Lack of ~ype i5o71np
of 5-alpha-rucluc~tsc 2 prodtices a clotrease ill ciilipclrotestosteroi~e, whic
ll is responsible for virilization OF the cstrmrl ge~iitalia.
A11
7 5-ALPHA-REDUCTASE D E F I C I E N C Y
IO/CC
\ t
l?-year-nlrl ullitc maIr comrs to ilic cmcr:cnc!' rrmm tompIaining of srvere rer
ro-oshital headache (heh it~d eps) his along with blitrred vision.
H c aIso complains of ~ v c a k n c ~ s thc pnst frrv months a n d ari m7cr
HPI
increase in hat size ;IS well as ;ti1 irtal~ilitv w m r his werlrling tcl ring (
dur tt) growth in fingerwidtl~).. familv al5r-t tlntes a Hi< coarsening of his f
acial feahares and deepening of his voice.
m
z
n
2
E z
0
PE 1 s I ~ ~ ~ ~ e r ~ ~ t ( R~ s1.7OJlOO). PE: skit) ztlirk and oily: promi:: .
r P ir)n
nent forehead and jaw; enlarger1 tonguc and ~tidurliiig p q hep hvecn repth: l a
r y hands and feet: bitempord hernianopsia;
&
< 1 3
Labs
Hyperglycemia: hv~~r~hnrplaat~~iiia;intrreasedIGF-1;increased levels of GH that
fa3 to suppress a f t r ~ oral glucnw load. FA: inC I . P ; ~ \ P ~ 1131-vralri
urn. 11i.i XR: thickrni~ig skull; ero3io1t and enlargement of sella of htrcica:
tvicIrirEng .rli~~al pha!at~pesin hands and Ccct. MR, hcad: rtil;~rgerlpitr~itar
y alantl containing a 2c-cm mass.
hcidophilic aclrnnina r,l' pi I i~iiarv e;lanrl with ill-delined capsule rrer-ts
tnasr; efPcts on pitt~itarv and nr-arbv optic cl~iasrn; tumor rarrly malignant.
Urnsef~~ ~ a t k c mature cclls that arc I~iyhl\d, xri~r~~il(>~?~tic ancl cosino
philic: stair15 higllly for GH.
Imaging
CYOSSPathology
Micm Pathology
Treatment
T~.;irrssphenoitEnl ici-uslit-gicalarlenr)mer~omv the trealmcnt tn is ol'choice:
radiottlrrapp to r c d t ~ c c further growth of tlilnor. Mudicil tli~rapv tvi~
lroc~rrtjtirlcai\d/oi hmiiiocripfine iS w r %el.\fail5 or is cnntrainrlira~rd.
Disc~ssion Thc. rntlst common causr uC i~cmlncgalvi c pituitary adenoma. IT ruce
qu GH secretinn is preqent in childhood, gigantism ill)I>P;1I.F; in adults, acro
megaly appears. Hcarlache and,joinr pain are early crllnplaintq; l>1ixrr-cd visi
on i m r l visu2l-firlrl changcs occur lirtcr. Almost ever). or-gill1 in I 11eho
rly i ~ ~ r ~ . e aine ~ i z e 2<5% of s c : patients exhihit gillcnsc intoleran
ce. \'i~uaIficld c11;trlgrs ( e . ~ . ~ hitcrnporal Iicniii~nopsi;~) ot.clls s~c
ondal-p compression niay tc) o 'the rit'tvt~~ ille opr ir cl~iasm tl-le trmlrjl-
. f r>f hv
Atlas Links
' -T'I PM-BC-003
U_I_L1 MC-084
ACROMEGALY
ID/CC
A 37-year-old fcmalc is admitted LO the internal medicine ward for e~~nluation i
ncreasing weakness and intermittent epi~ortes of of dizziness, nausea, and vomit
ing related to stress and exercise.
HPI
She is a vegetarian, takes nu drugs or medications, and docs not rlrink alcohol
or smoke cigarettes. S h r reports an excessive craving for salty foods such ap
chip% and salted pennuls.
VS: tachycardia (HI? 1 10);hypotension (BP 90J65). PE: thin with dt-v ~llucous m
embranes: pigmentation of buccal mucosa and palms of hands; 110 neck tilasseq: c
hest auscultation r~osrnal; TIU ahd~)rriinal InaSses; n o heparosplenomegalv: no
Ivmphadenopathy.
PE
Labs
CBC: narmal. Lytes: hyponafremia:hyperkalemia. Glucose low; incrertsed BUN with
normal matinine: amylase and LETS normal; high ACTH; low cortisol.
Glucr~corticoicl and mineralocorticuid llorrnot~es. Nydrocortisonc on an e~nerge
tlt basiu.
Treatment
Discussion
P1.i wary hypoarlrenali~m(i2D~tc;on"s DISEA~E) rriay be caused hy autoimmune mec
hanisms. tuhcrculot~s infection, or sudden discon tinuation of chronic steroid a
cltninistration. Secoi~darv y h pnadrenalism i s clue to abnormalities of hypr>t
halarnic-pituitwv
ADDISON'S DISEASE
l'D/CC
X 25yearsld remale is admitted to the hospital for u\.nluation or persistent wea
kness for the last 6 months that has heen unre~ p o n s i v e multivitamin treat
ment. to
HPI
She denies allergies, sul-g-et-ies,pychnlngical prohleins, ti-ansfilsions, drug
lise. or any relevat~t past rnerlical history
m 2
PE
VS: heart rate normal; no fever; normal BP (exclude$ primary
11yp~'raldosteronism). well Ilvdraled: pupils eqltal and reacPE: live to light a
nd accommodation; 1 1 0 neck masses; nn lymphadenopxthy: chest normal: abdomen s
oft and nontendcr; no zn;lsser: nelr I-oIr,gic esam nnl-ma?: no peripheral edema
; scxr~al development appropriate for ngc.
u
a
r l
i2 z
0 r 0
Labs
CBC: normal. Lytes: hyponatremia; hypokalernia. :ZBGs: metabolic alkalosis. Jncr
eaqed plaqma renin (excludes primary hype^-aldosteronism); increased urinary rxc
rction of prostaglandins.
Micro Pathology
Treatment
Juxtaglomerular cell hyperplaqia on renal ?>iopw.
Indometliacin to dccrc;ise prul;z;~glanrlin synthesis. Potassium chloride st~ppl
ement? and potassiltm-sparing di~~retics.
Discussion
Bartter's syndrame is a hereditary disorrlcr characteri7,ccl by a defective Nt +
-Kt-261- cotranspnrter in the thick ilsrencling loop of Hetlle. Thiq results in
an impaired reahsorptivc consvrmtion mrchat~ism f saclium (due to enrl-organ rcs
iszance to anr giotensin), with urinary sodium wasting and a curlqequenl inrreaq
e in renit) pmd~~criotl (thmugh increased renal prostaglandins). Therefurc. ther
e is ilrr i n c r c l s ~n aldosterone i activi tv with hvpokalemic al kalosic.
Hyokalemia perpetrlatcs thc
BARTTER'S SYNDROME
ID/CC
X newborn is evalitated by a nennatologist because the intern whv petic?rmed the
delivery cannot tell whether the child is male or female (nhi.rnrc:~~ous E K ~
~ . ~ L I A ) . (;
HPI
'The child is also lethargic a n d lacks sufficient uh-engtl?to suck on mothvr's
milk adequately (due to ~aIz wasting).
PE
Ambiguous exler.na1 z~nitalia: increase in size of clitoris; fitsion of labia t
the point of resemI>liilgscl-otal sar. o
Lytes: hsponarremia; hyperkalemia. Increase in 17u-OH progesterone ancl its mcta
bnlite, pregnanetriol; increase i urinary n 17-ketosteroids (defect is distal LO
I 7. 20rlesmolasc.); eletazed scrum ACTH. Pre11;ual diapnoqis is p o s ~ i h l
c 14 to 16 weeks at ( r l t ~ er> i ncreaqe in 17-a-01 progestero~le)Kacyoqpe: 4
6,XX I I .
fentale.
Labs
I
Treatment Discussion
Cortisol, dehydrucorticosterotle acetate i salt wasting is present.
Lack of 21-hydtloxy?lasecauses a decrcasc in cortisol with a conrequent increase
in ACTSI, which in tlu-n pl-orluces h!.perpIrrsia of the aclrcnal.ercsultir~g a
n iract-ease in androgen producin tion that gives rise lo signs of female pseudo
hermaphrodithn (ar in this case) or enlarged genitalia i n the male. May occur w
it11 o r without salt wasting.
ID/CC
A P'i-year-olcl lvhire male complains of excessive thirst (POLYI~IPS I A ) and i
ncreased urination ( I ~ o ~ x L ~ R ~. ~ )t w r y tiiluttd urine. I i h
Tlw patient drinks wvei-a1 litcrs of w-,~tcr every clay. H c w;14 well until thi
s timr. Thc paticrlt alro idmi15 to t i e q i ~ ~~u-itlation nl (including noctu
ria) ni' large v o i ~ ~ n l that :!re clear and watery. es
HPI
rn
Z
PE
VS: slight ~achycardia. PI:: miId r l r y ~ ~ e s s ofmucour ~ ~ ~ u i n b r a r
~ e s ; w 0 . i 7 i,is11;dfield iecting normal; nn papilledema: pupils c q ~ ~
and rral
active.
E
z 0
7
Labs
Low wine specific gravity < 1.006) : low urine osmolarity (< 200 inOsm/LI; eleva
ted s e m osrnoldity (> 290 mOsm/L); hypemahmia: water deprivation test dernc~us
tralusin;lhili ty to r o i ~ r e il-ale ~u-ine n with fl~xicl restrictiml ( u r
i ~ ~ aor;ii~c>l,~Ii ry tv rontinues to he Iovl.
Imaging
Treatment
C:T: inn? show rnaqses or 1esinns in hypoth;llarnz~s.
C:~nlral(primarr) diabetes insipidus: irltranasal clesmopre~sin, diuretics a n d
.%DI3-i-eIe;isir1gdr11:~ $11611as rh1orprn13arnidc,
car-ha maze pin^ anrl clofi brate. Ncphrogrnic (srrr~ridat-v) diabetes insipidus
: aclcl indornetharin. amilr~rirEe, J a r and hvdrocl~lorr~tl~iazidr.
Discussion
Dialnetcs insipid~i.;is trlusurl IJV srs ADH d ~ l i c i ~ ( I ~ K I ~ . ~ u < I
) ~ c y or 13)' n-nal un l.eqponcivei~ew r1r)r-l ( ~ t ; l ' t )(:E:KIC: OK <EC
'OND,+RY) to t~~ Primary diabetes insi pidl~c ran he caliserl by surxical, ~raum
atic. o -anosir damage to thc h ~ p o t h a l a n ~ or pituitarv sialk rltu-it~g
i us pregnancv (SHE:EH,~R:'sS Y K U R O ~ I F , )Deficiicliicy (11 rZI)I-1 resu
lts in .
*
DIABETES INSIPIDUS
ID/CC
X 2s-vcar-old seamqt rew i 5 admitted to tllc inlernal lnerlicine ward hecause o
f ~naEai~e, confi~sion, abdominal pain. vomiting, and cliarrhca.
HPI
She
i 5 x kr~nwn iim~lin-dependent diabetic (IDDhl 1v1)c 1, jln'enilc onsi*t).One da
y before her admission. ?he werl t 0111 tn cele1)ratr hpr. I>irt?~dar- drank alc
ohol t i r ~ i i lshe becarrle intoxicated ancl (r tre nlrn forgor to adrnirlist
er i ~ ~ ~ ). ~ l i n l
P'E
\IS:tachvcarclia (HR 92); hvpotension (BP 90/50) ( d i ~ e hyp* 10 volc~nia)rapi
d, deep breathing (K~lss~,\Ll, : RFSI'TR~TIOV).PE: dehydration; peripheral cyano
sis: cold, dry skin; peculiar fruity breath smell (due to ketonc I~odies, iicelo
acetate. and P-OHhuryate).
CBC: leiikocvto~is(14,000) (without infection). I .ytes: h!.pona11-e~nia (1.30 ~
n E q L ) . ABGF:markedly reduced bicarbonate J { 1 0 mEq/L): acidosis (pH = 7.
1). Increasecl keloncs in blood; increased crca tinine; hyperglycemia; increased
anion gap { f ~ e t w e r r ~ arid 18) (an inn gap is calctilatetl as fnllnws:
I0 [N;I t- K]-[CI + HC:07]); increased a~nylnse(cvi~hottt pancreati tis) . Vrk g
lvcosru.ia; ketor~uria.
Nauogasu-ic t~the, correction of [l~tid defirit (caution owing to rick of prodtt
cing eel-chral erlen-ra). Potassium. Gr;~cltiaF Iwveri ng oC glr~cosc with insul
in.
Ketoar irlnsis might hc the fir.sl ma~iil'rstationof diahctes. I1 is morc connno
ri in instzli rldependent diabetics I ~ : I Yh yperomrr I lar cum:\. I( t~runlly
Cnrlowq a period of' physical or rnpnral stress ( c . ~ .MI. acute grieT) 01- i
nfection. ,
Labs
Treatment
Discussion
DIABETIC KETOACIDOSIS
ID/CC
/\ .iRyei~r+>ld hi tt- Female comcs tr.} src her illlrsniqt hecauw of I\ 1 1rlc=
uelopn~et~r 1r oi'polylria, palydipsia (due to hyperglvceinin). and a skin erlip
tion tlzar comcs a n d ~ u c ) in \cliffcrrnl parts of c' liur I~rlrlv ~ l . t ,
l < c l l . ! ' 1C. .Htc;K/ltOKY ILWH). ( ~
Sllthalso c.c)nipl;lin.;rtf incrci~sillgl l ~crrui i tie111 diarrhea, nausea, vo
miting, weight lass,and nrca\ior ial wraknc-ss anrl dizziness.
HPI
PE
1 s [lor-rnal. PK: palien1 WPII livrlt.att=rl:marlirrl pallor; erythema': tous r
ash on anterior chest, leg, and arms; no ncck massrl;;
lungs clcar to uscu cult at ion; h e a r t so~ii~ctr rhvlhrnic; ahclnmen sorl: 1
10 III;~SFCS:pel-iro n ~ a G ~ I I F :IICI Iynpharl~~r~opat 110 l hv.
Labs
CXC: ;tnet~lia H h 7.4 mg/dl,). Markedly increased s e r w n ( d u c a p n Iwels
; hpperglycemia.
Imaging
M R/C:T: ?.:icr-tt enh;~ ncirlg mass in hnrlv and tail or pancreas; ~e17el.di ~
rme1 aqtases. l t
St~rgiciil rc~moval. Sircp~ozt)citi il'm~tast;~tir: it~sulirl: praphylacric hepa
rin and /inc (for skit1 t;l%Ii); oc11.eatirle.
Glucajionomn i s a pailcrcatic islct cell ~ ~ c o p l a s(of rr cells) that n~ s
rcrrre\ ahnoi~inallv 11igli atrlotilnt.: rtf g l ~ ~ r a g c ~ n resuI tin^ with
symptomatic hyperglycemia: it mav :~lsosccrctc gllstl-in. ACTH. anrl %el-otoniu
. I;lncagonomas arise from ry, is1c.t crlls in he pitntrrilh. a n d t l ~ r niqj
oritv ( > 70%) arc n ~ : t l i ~ i a r iC;lucagon.ntn;rs l. nlav also br nswcini
erl with nllzltiple enrlocsine rleoplasia (MEN)
type
Treatment
Discussion
r.
-
Atlas Links
a n.PM-BC-009A,
PM-BC-OOJB
GLUCAGONOMA
ID/CC
h 24-ycar-olcl wornan i s referred r o t h e endocrirlologi-is~ Ilerause
uf ccnrcern over excessive facial hair along wirh lixir on 31e1-cent tral ~ h p
s and ~Iliglis.
HPI
The patient's menses are regular. with an average flow 3 to 4 days. She is not r
aking any drugs.
la91 ing
PE
Hissu~ismnotprl; no clitoromegdy presrnr (nn evidence of virilizaliol~); abdomin
al nr pclric mass palpat>l~ abdomen or 110 per pel \vagina.
Normal total testosterone level^; norrnd DH13AS: norinal uriilc rilr 1?-kctohier
uicls.
Labs
Imaging
Treatment
Discussion
US, abdomen and pcl~5s: h adrenals and ovaric.i normal. hot
No therapy required.
tIir.sutisin that is dispr-opr,~-(innate the paticrl t's e ~ tiir hackto h groun
d and is accompanied by normal periods is trrmed idicpathic. If tesrosternne anr
l DHL%SIcvuls are tlormal, rhe paticnl can br I-oasstir.et1 1lia1 ftir conclitio
n is bcnign. If the nnset of hi~'Slltj~~1 is pubertal with irregular periods. th
e possibility of pulvcrstic orariarl syndrotr~e exists. Reccnt-cjnsr~hirquhrn in
an ; t d i ~ l f+tnale, especiallv whm as\rlcixted with amenorrhea, rel quires
compler~ n v e s t i ~ ~ t i otu esclwle an adrenal nr ovarian i n
t~lrnnt-.
pbs!
*
+
HIRSUTISM-IDIOPATHIC
IDJCC
,"l~~~-vear.nld wum;lrl is referr~rl an iutcmist bv her farniIy to PI i i c ~~ i
o n e I l e c a u ~f persisten1 hypertension that has heen i r o tmrcsponqive t
o conventional treatment: she also complains d ~>~wfoirnrl muscle weakness (clue
to li!pcl kalrrrl ia) .
HPI
PE
Sbt. is a two-pwk-a-clav srnokct- who drinks occasinniillv.
n
VS: ~lornialIi~art rate; hypertension (BP 200/ 100). PE: n o paIIrr: retinal hem
orrhages, exudates, and AV nicking; Iungx cltS;rr; no hcart 1nurrnu1-s;nl~dorncn
soft; no palpahlr masscs: no I!m~phar~cnopatl~p
CRC:: i~lcrcaqerlhen~ntorrir.Lytes: hypokalemia; hypernakernia (qeconrlary to I~
vpcl-aFdnstcronism) . .4BC;s: high swum hicarbonate. Glucost, normal (vs. r-ctop
irACTH prr~tluctinir . F,I:C;: left ) vr~~tritttEar I~ypc-rlrnphv n ~ c lstriiii
i. LA: 1 1 0 proteinuria. i Aldosterone leveLs high; renin levels low (primary E
l!-pe~-aldusternilism).
2 rJ
X
E 0 6
rn
Labs
<
Imaging
GT JMR: 1.7-crn rnh:~rlcingIcft :dren;il rniiss. h'P-T,!) irldacl~olrsf ~ r - r
, l ~cintig~;~/,liv t i w . Arlrenal vcnnus sampling shows pasi >10: 1 n t i o o
f'aldostcrt~nt. I 1 the left v c ~ s u x o 1-ixht.
I~lon~rri~Eos;tcell ~ i g t ~ hei adrenal adenoma.
Micro Pathology Treatment
t .eft arll-ellalectmnv.
Primary 11~pcraldor;teronis~n typically prr'icnts wit t i I ~ v p t t ~ - ~ ~ n
%ion.hy~okalcmia. ~ p e r n a ~ r c m iai~ri h i ~ , incwnsed I~ical-honate r l
~ to e ~ it~crraurcl secrctinn o f alclnstc.rone ~ J ;In it(lrtnal adeJ nonla (C
OWN'S SYNI)KU!~E) rlr hypcrpli~sb. H!yertt.nsiot~ is chnrac turisticallv unrespo
nsive to ACE inhibitors. Surglcall~/ cr)rrecrakle causes ot-hvpel-tension i n r
l ~ ~Conn'q s!mdrnme. rl~ phcochrnmocvtomn, rcnat ar-tery strnr>.;iq. iund c-nar
ciation rjL' !lie wria.
Discussion
HYPERALDOSTERONISM-PRIMARY
ID/CC
A .5.5-veal-*lrl menopausal female comes to sev her inlernist hecaust of pro~~rs
sivc constipation and excessive urination over t tle past 2 months; she also con
lplai~is palpitations both at or rcst and (luring exercise.
She I1a5 read "all abo~it" osteoporosis during rnenopauqe and i 5 afraid of deve
loping i t , so she has hcen t a k i n ~ abundant calcium
HPL
supplements and vitamin D injections.
PE
W: hrai-t rate 80 with skipped he;iis heard ( K E m t : r t ! A ~ E S T R ~ ~ S
~ - S T O I . E S ) . lungs clear; nu ncck masses; thvroid not PE: palpable; no
lymphadenopathy; muscle weakness with hyporeflexia; ahdomrn soft wit11 decreased
howrl ~rll~nds; masses; no n o abnormal pigmentation: soft tksue calcification
in skin of arms a n d legs.
Labs
Markedly increased serum calcium ( 12 mg/dL) (always correct calcium l e ~ ~ fm-
serum albumin). Phosphorus normal (makes el primal hypcrparathyroidism less lik
ely). ARGs: mera1,olic alkalosis. Increased EUN. ECG: short Q-T. PTH-related pro
tcin Nn rlprerr ed.
Aggressive ref~vrlration I? tiol-ma1 d i n e ; diuresis with flurosewit rnicle l
o inrreasc sodium and concomi tan1 calci~im u c l - d o n . e Calcitonin, plicam
ycin, or bisphorpl~onates(etidronate, pamidronate) may he ntceqsar.):
Treatment
Discussion
Hypercalcernia may occur in hyperparathyroidism. milk-alkali s~mdi.omr,mrilriple
myeloma, Adrlison'5 disease, s:lscoidosis, prolnng~d im~nohilizatian. metastati
c nropEastic disease (due to incrca~ed nstcoclastic resorption). and primary neo
plastic disease (clue lo prodtlcti on of a PTH-I i ke substance). Fifty percent
orser-utn calcium is hound to albrirnin: most of the re%tir activclv ru;rbsorht.
d in t l ~proximat luhule toger h e r rvith sodium. r TIiiq i-eahqorpi ion is dr
creased with expansion of extracellular fluid volr~mc.
HYPERCALCEMIA
ID/CC
A 55year-olrl white female goes 10 her fhmily doctor for a row tine physical and
is found to he hypercalcemic.
SPie is asymptomatic except
C t mild o-
HPI PE
polp~a.
VS: mild hypertension. PE: no neck masses: thyroid nnt palpaMe: 110 1 mphadenopa
thy lungs clcar: humr vnr nrls normal; ) ahdome11soft; no masses.
Increased serum calcium; phosphorus low; elevated FTH; increased alkaline phosph
atase. I:X: increased urinary calcium; clcvatrcl urinary CAMP and h ~ ~ d r o x
c r o l i n e 1eveIs. ABGs: hyperchlorernic metabolic acidosis (nor.mal anion ga
p). ECG: short Q-T.
rn 5 :
0 0
r*
Labs
z z
0 0
_<
r
G I
Imaging
XR: nuhperin~teal hone resorption; cystic Iong-bone lesions (URC~VN TUMORS). Nuc
: increased hone uptake on honr wan.
Soft, r0111ic1,well-encapsulat~rl. vellnwish-brown single parathyroid arlenoma w
eighing 2 g.
Gross Pathology
Micro Pathology
Treatment
Discussion
Chief cells withiai adrnoma. Surgical removal.
Primary hypersecrction of parathymid hormone rnay be callsect h\: an adenoma (va
sl n~iijoritv cases), chier-cell Iipel-plasia, or of' carcinotna of the par-ar h
vl-oirl glands: it is com~nonlv asymptc~matic and is frequently recognized durin
g rourinc physical exams. U l ~ c n is yrnproma.tic. peptic (rlc~r it pain, pnlv
uria. polydipsia, constipation. and pwlcreatiti%mav he the p r ~ ~ t n t i n g s
vmptoms. May bc assuciatcd with multiple endocrine neoplaqia ( M E N ) mmdromcs
I ancl IT.
Atlas Pinks
D'!TI PM-BC-013, PG-BC-013
HYPERPARATHYROIDISM-PRIMARY
TD/CC
A 9E-yrar-old fcmale vi~itsIier family doctor hccnusc of ar~xiet): intolerance t
o heat, nctvousness wiih trembling hands. and weight loss dcspite a normal appet
ite.
Shr is aIso conccrnrd ahout inc1.uasi13g protrusion o f her eyes
HPI
(~xr~rrmr~r.nms).
PE
'CIS: tachycardia; llypeatcnsion (BP 150/80). PE: wide pulsr pre+ uire: sweaty p
alm5; warm skin; exophthalmos (dnc to enlargenlcn t of extraocl~larnmscles); gen
eralized enlargement of thyroid gland wit11 hrnit ( ~ ~ ~ F T I ~ S E : nndul:~s
E;OI'I'EI~) lesion4 over. atlterior. i l ~ p ~nutlower leg% c (PR~:TIB?:~L I X
E I I E ~ ~ A ) . M
Labs
Markedly decreased TSH (due to negative feedhark of'allionornously secrrird thyr
oid hormones) : incrcascd T ,T4, , and free T, index; positive TSH receptnr anti
bodies and anrit~uclear antibodies: hypercalcernia.CRC: anemia. Kuc: incrrtilscd
rarlioactive iodine uptake tlleasurenlcnt; cnla t-ged glnncl.
Irlcr~asrd vascztlarit~ thyroid gIarlc1 wi tlr s)mmeirical enlat-gvoF me131 .
Imaging
Gross Pathology
Micro P a t h ~ l ~ g yThvl-oicl gland Il~~vpertrophy hy~erplasia; and rcducrd t
h!-rt~id hormone stcjraq a t ~ icollnid; iilfil tra tive ophiliaim<)p,a~ l hy.
Treatment
Discussion
Antithwnicl drugs;radioactive incline.
,4150
called rli fC~lsrr nxic goiter, (;raves' di~ease the Innst is cc>mmr>n caclse of
Ii~purt t~yroirlism. is idiopathic in nature bill It has an autoimmune basis an
d i associated with I-ILA-T38 and s HLA-DRS. LATS, an IgG. is rrsponsible fhr so
me rr~anifestations. Signs and ~y11ipio~11,~ to excess circulating lhyroirl hora
re drie
ItlOrlP.
Atlas Links
KTT3I PM-BC-014
MC-O9oA. MC-090B
HYPERTHYROIDISM (GRAVESf DISEASE)
ID/CC
A ~t I-ytbal-nlrl o h e ~ e rcini~le cot1)e.i to thc ER with severe epigas tric
pain radiating to the back arcnmpnr~irrlhv n;Iil.;ea and vomiting; S ~ C had hec
n advised to undergo laparoscopic rernoval of' ' ~yrlptnmatic small gdlbIadder s
tones.
Shc was admit terl r n I he sl~rqici~l floor ancl tl-eatcd lor pancrcatiti<. On
the third dayashe det'elopecl numbness of the Finqem and around h c mouth and to
ngue a well as painful leg m p s s
( I I\FO<:A1.CFMTC TF,T,T.~~T).
HPI
Z
rn
CJ
5 2
Z
g
P -
PE
VS:hjpotcnsion; tachycardia; f c v c ~PE: dellyd~.a~r.rl in acute , and
rli~trr~ss; hilatcril basal h!.pow~l~ilation; abtlo~netllender in epimesopstri~t
r~l; I~?pocalcrmic signs pr-eqent; ;~l?d~~cliclt~ and flexion oL'f~3ot whcn pcro
rleal ilrrve is lapped ( P O F ~ T I ~ T . IT:IZOVI,>II. s ~ c ; ~ livprrt-sciti
~l~iiiw ~ i l e ~ a i r ~ g ); wl l galrat~ic rlirrent (ERR'S SIGN) : facial spa
sin on tapping ovrr cheek ( C H V ~ EK'S GIGP;) )91 ;
car.13al ~ p a r n ~ with ~ t ~ o r c l ~ ~ s i aby hInorl prcssurc cuff ~ c art
erial n (TKCIL'S~;F~~L:',F SIGN).
Labs
CBC: mnrkcd leukocvtosis ( 1 5.000) with nrl~trophilia. Amylase and lipase marke
dly eIevated ( d u e lo acutc panurtati I is). I.;CC;: Q-T prolongation. R/lar.k
edlyreduced serum calcitun: nurmal
serum albumin.
Imaging
KLTR: increace in gastrrrcnlic space: sentinel loop. C X R : s~nall Ipi't pIe11r
al eftl~sion. Hcr~~ori-hap[. paiic~.eatitis wit11 edema and areas or gray-tvllit
c necrosis: in1I-aperitonealIiee hcmorrhxqir fluirl: chalky-\vhitr filt nccl-o~i
s (saponificationof calcium with lipids).
Gross Pathology
Treatment
T r e a ~ n c r ~ n t i t i s 11' calcium giuconate. p .
H Y P O C A L C E N I A F R O M PANCREATITIS
ID/CC
ASS-venr-nldfenlale cotnplainsofweakness,painfulmwcle
cramps. and constipation.
WPI
She sufTcrs from chronic coi~geaivrhear1 Cail~~re (CI-IF) that has bceii ire;lte
d with digoxin atirl f~msemide. was also on oral She potawium 'tablets tr11t has
disconti~ll~rrth ~ m t bccausc of gastric
llI3WL.
PE
VS: irreyllarlv irremilnr pulse (atrial lihrillatio~~) ; hypertt-rlsion (BP 145/
1)0): n o Scvcr. PE: well I-lydr;ltccf;ronjlrnctivir ric)rninl: jugular \*enour
p l ~ l f esligh~ly increawrl, S3 heard; milrl Ilcpatc* mrgalv ant1 pitting ed~r
na lower legs (all due to CTIF) : dcep of i~rtdnn reflexes hypoactive.
CRC: normal. r.y~es: hypokalemia. ECK: tlatt ening 01' ST segnient and T wares:
prominent U waves.
Potassium-rich Ibnds (chick peas. bananas, p:lpaya) : oral potassil~rn supplcmcn
ts; gastric mucos;lf ptot rc~ive xqetlls; magneqium supple~nen~s (rleficier~cy;
mqnesinm ik.equentlr cocsi~ts) of . Potas5ilin1-sparii~g diuretics.
Potent diuretics such its filrose~nide frequtltlfly muse excessiw renal loss n p
oras~iunl C with s>nlptomatic I I ~ o L ~ ~ ~which, ~ ~ I T G i f sererc. may he
lifc-tl-tzcatcning. In patients on digoxin, hypokalemia greatly increases toxic
ity.
tabs
Treatment
Discussion
Icb?l
HYPOKALEMIA
1D/CC
A .lRvu'ir-old female who has been on total parentera1 nutrition for- 2 weeks co
n~plains weakness, cramps, palpitations, of tremors, and depression.
Qnr week ago, die ~ l t i d e r ~ e ~ i i .fir1 h m;ljor abdominal opcmlie^ tinn
Tor intestinal Listula and sepsir.
HPI
PE
VS: tachycardia;hvpatension. PE: paticrit 1ooL confused and " t - t i 11 rlown "
: agitation rvitli muscular-spasticity and hyperreflexia: heart soi~nds clisclos
e skipper1 hcals; mild hypoaer.arir>n lung at I~nqes: ahtlomerr wiih I ~ I I P P
o l o s t ~ t ~hag^ at site of fistula; no c ~y peritonpal irriration: no s~~rg
ical wound i11Crction. CBC: neu trophilic lenkocvtosis. Lytcs: hypomapnesernia (
< 0.8 rnlnol/L): br)rdrrline II ypr>kalctnia:hypocdcemia (suvcrc trvpc)n~a~neser
nia rcrlt~ces PTH ~ e r r e ~ o n ) : ?rl-I~our low urinary tr~:~grreciuin excre
rion. ECG: prolonged P-R and Q-T intervals; wide QRS: tall T waves; premature ve
ntsicuIar ectopic conbactions.
h l n ~ ~ ~ e s stipplm~entation. i~un H~pnkalernia and h~pr-rcalccmia r~solve w
ith m a ~ i r s i u m replaccmc~i t.
I-lnnieo~rasi~ n r s g n e ~ i ~ i~ n l or s achieved till-ough a halancc beiwer
n inrestinal (sn~all howell absorptinn and u r i r i a r ~ excretion. Dcficiunnr
is associated wi tll rI~e of iI large amrstlnr of IV u.ic fl1iirF5, alcrnholiq~
n, intestinal malahsoi-ption or diarrhea, inadequatr rcplaccmr~~t parc11trl:iI ~
ititiirion. in k~mhiol-kor or marasmits, pmlongerl GI suctioi~, t eqti nd l i ~
n ~ lrcnrd mhulat in a. a r i d o ~ i a i. ~ d ol'dt-1t.v ETSIICII a5 ~ l i l l
r e t i c cisplatin. n ~ e t l ~ o i x x ~ tl'ie ~. atc. amphntericin 15. cyclor
porinc ancl amino~lycosides,
Labs
Treatment
Qisclrssion
HYPOMAGNESEMIA
IDJCC
X 37-ycar-olrt white Cernaic romplains of narrsea, vomiting, and headache on her
firs1 poslopc.rative dav: thr charge 11 tlrse fnunrl hcr having a grnnd tnal se
izure.
HPI
She had clcctive surgery for a hen ign left rwarian cvst. Her merlical history d
iscloses no prrvious illness.
PE
VS: nn fcrer; normal hcart rate. PE: wcIl h~dratcd: slight confilsion and lethar
-gy as ~ v e l I general weakness; slight incrc;rse in as ,PT; HI) Meetling or d
eliisceiic~(opening of sru-giral t r m r n d ) or infection li-om vu-gical ~vntl
nri;n o p~ritonealsign$;significant I~ilateraI louw-extl-einity erlema. CBC: nor
mal. Lytrs: l~ypunatrcmia (N:I 115). Remainder of routine lab exams norm;tl; i~n
r-ma1 cortisol (done to eurlucle pnssiI,te adl-rnaI insurficiency): Ferlun osmol
ality < 280. For h!pcr- or isovt~lcmic hyponatrcmia: watcr rcqt tiction (with c;
\lttion to avoid r~srnotic ti-a1 pr~rl cen tinr ntyeli~~nlvsis yndrome. which ca
t1 ncclu- while rpqtnl-ing sodium lcvels too quickly). Fur hy-pmntemic hvponatre
mia: isotonic saline, s?uwly.
Labs
Treatment
Discussion
Hyponatlwnia is the rnosr cornmot? elrctl-olvr~ distl~rbauce seen in ho~piralise
d palients and i~ often iatrogenic in nature. 111 a ~m~operative setting. the me
tabolic response l o m,~tIrna to inis crense sccrction oFADT-I. among othcr h o
r r n o n u ~which. colt. pled wii 11 ( I V ~ I . X ~ ~ ~ C rVI F I I acltt~i~~i
st ratio11 ni' llypn~ot~iciiids. may ff
HYPONATREMIA
ID/CC
A 1Ryear-olrl ohew white fernale w h o works as a j a i l i l o r is hroligh r t
o the EK in a coma alier heing founrl on the iloor of her room.
Hrr hushancl reveals t h a t she has been having episodes of eariymorning dizzin
ess and c u n f ~ ~ s i o n assc~ciated i t h hunger and w walking; he adds I ha
t I hese symptoms disappear after eating. He also stales that the patient Iiar f
rcq~~entlv nenoI1s and bren irritable.
HPI
m
z
w
X
E Z
PE
13:tachvcardia {I-IR 105): BP normal. PE: patient comatose; mild skin pallor; co
ld, sweaty hand-s;110 focal iieurolngic deficits; lieart sout~tls rhvthiiiic; nn
mtu-m w-s; I u n g ~ clear; abd o m m soft; no masses: peristalsis present.
Normal hcrnnglohin ( 14.4 mg/dL); B U N ancl creatinine normal. Lytes: normaf. H
ypoglycemia (blood glwose 58 mg/dL,); elevircd i ~ ~ s u l ielevated plaqrna imm
unoreactive Gpeptide n; (vs. exogetlolis it~sulin actn1inil;tratton.whei-e (>pep
tide is low) Positive 72 hour Fasting tcst (incrcascrl insulir~IevcClq despite h
ypoalvcemia) .
0 r 0
< G )
Labs
Imaging
CT: l . k m mass i tail of pancreas. Nuc: mass takes up ocn treotide.
Single adenomatous tnass.
GTOSSPathology
Micro Pathology
Treatment
Findings accol-ding to t y p ~ islet cell ir~vnlved. of I~nmediate V gI~icose n
f ~ ~ s i o surgical resection. T i n;
T h c most common pancreatic islet cell tumor is Pcell insul i ~ ~ o i n ~ ~ s u
a lhenign). Othcr types include glucagonomas, (a ly som;ttosiatinoma4, g a ~ri1
1011ia~ t (ZOI.I.~KGER-EI.I ~YNTIROMF) , .rsnri and exces4vc VI1'-secreting clrt
nor (VERNFR-M~RRISON SI'N~ROMI:.). ~ F T P I cell tumor< may 1~ secn ill multipl
e endocrine nenplasia {MEN) m d r o l n e s .
Discussion
"
INSULINOMA
ID/CC
h lR-year-old fcmalc is hi-ought to a local clinic becanse she
has never had a mensrr~ial pel-iod (PRIMARY AMENORKHFA) and show$ a lack of brea
st development.
HPI
She ha5 a cleft Iip and palate. On d i r e c t ~ d questioning, she reports a di
minishccl sense of smell (H\'POSM.II&).
PE
VS: normal. PE: left cleft lip and incomplete unilateral cleft palate; marked hy
pasmia on olfactory testing; heart and lung ~nunrls within normal limits; nn pal
pable mass in abdomen and p~lviq: pubic or a d a r y hair; no breast tissue. no
Labs
CRC/Lytes: nonnal. LFTs normal; decreased GnRH; low FSH
and LH.
Imaging
XR, skull: normal sclla trircica. MR, braitl: ahwns olfactory bulb (s) .
Gonadntrnpins.
Treatment Discussion
Kallinann'ssyndrome is an X-linked disor-der chatxterized by dcficiencv of GnRH
with a resuI~ing decrease in FSH and LN levels, producing a~ isolated l~ypogonad
otropic hypogonadiqm. Ir is tvpicaIly associaled uith agenesis or hypnplasia of
t l ~ c olfitctory blilh.;. producing anosmia or. hypostnia (lack of stimulus fo
r GnRH production due to absent olfactory bulb catecholamine
- .
KALLMANN'S SYNDROME
ID/CC
A 2-year-lcl ~ i r l the daughter ol' an African immigrant, is admit, ted to the
pediatric ward owing t o an increaw In abdominal girth arid failure to thrive.
HP1
She recently arrived in the United Slates horn her h o m e collntry She was brea
st-fed until I year of age, at which time her mother can u ~ iof milk. She is ap
atl~elic t and irritable and has hccn having frequent cpisodev of diarrhea.
Height and weight i fifth percentile: skim and hair depigmenfan tion; thinning o
f hair: dry skin; h~perheratosison axiltae and q u i r t ; hcpatomegaly and asci
tes; genrrrtlized pitting edema: toss
m Z
u
PE
2 E
0 r 0
$
Labs
CBC: atlrmia; Ivn~phope~lia. Hppodb~uninernia (normal it1 n~ar~wrnus).p s : hvpo
kalemia: hvpomaqewmia. L
Imaging
patjcsearitis).
US/CT: h t t y livrr: KUR: patjcreatir calcification (due to tropical X R delaye
d hone age.
Gross Pathology
Micm Pathology
Fatty infiltration of liver.
Intestinal rnurosal atrophy wit11 loss of brush knrder enzymes; atrophy of pancr
eatic islet cells; ~vidrspread fattv in filtration of liver.
Treatment
Reqiore acid-base anrl electrolvte balance; meat infections; gradually initiate
high-protein diet with vitamins and nlinerals.
Kwasliiorkor is a form of maInutrition caused by protein deprivation with normal
total caloric intake.
Discussion
KWASHIORKOR
ID/CC
A 68-year-rhd ohcsc ni;ale is rushed to the ER ahel- he wvas found unconscious o
n thc flooi- of his office. I Ie had Ixcn on mrdication for corctnary artrry diu
ras~.
On adinissioi~, i~ round to be in an acute statr of tiqsue hypohe perfusion ( s
r i n m ) with a 11arelv pal paldc pulse. hypothermia, artd hl-adyrardia.Inlrnud
iate treatment for- cardiac s l ~ n c k is
H PI: PE
hegun.
Labs
EGG: ncute an trroseptal myocai-dial infarctioi~, Increased serum lactate; Ilvpe
rphosphatcmia. .ZRGs: severe metabolic acidosis (pH 7 . 2 7 ) ;hicarbollare 14 m
Eq/L (luw). Increased anion gap ( I <J) with nr kutoacids; HUN and crratil~ine r
r normal.
Trcat pt-ecipilati~~g cause of acicloqis; arlministcr biciu-lronate if pH is < 7
.2. 'Treat shock.
Treatment
Discussion
iZ Ftate of incrcasrtl levels of lactic acid in blood (t\~:rrr: c m o s a ) ~ ma
y be due io a number C T call'ies, including shock and sepsis ~ (Imr 11 or \\.I~
ich incr-ease lact ir acid prorlitctiotl rll~e hypoxia). to tr~ethanoE poisoning
, metfnrmin toxici~ and livur f;tiFltrc (due y, to failure of lactic- acid to hu
r c r n o v ~ r ~ rrom blood by its transfurmariar-i ro glucose). T ~ i ~ ~ ~ i
po p l is an cstirnatini~s the total u r C
~~nn~c;lsurccl pkiuma at-tinns, such as pi.c~~riil%, organic acids, phosphate, a
ncl sulhtc. Iricreascrl anion-gap n i ~ t a l ~ o lacidosi~ ic i a clue in ~alic
platc poisoning, alcohoI (u-g., methanol. ctharlol, propyl~en glvcol) intnxicari
nn, lactic acidosis, renal failure, and rliabctic kctoiiciilosis.
LACTIC ACIDOSIS
ID/CC
i\ 1-vrar-nld I-rn~ale faken Ir) I he erilrrgeiicy t-r )r,rtl hecause 01 ic pers
istent vomitins (20 timrs in 21 hour?) that has buuii unrc-
sponsivr
HPL
trl
in tram~lscl~lar. tirmetirs. an
Ml~iEc.on a L~rnily vacarion to FTori(la, slie t m s ~ i v e n vanilla ire rrrai
ii t har wa\ 13~i11.q sol~1 t i 1 I ~ st rrer (~lairy o P 311dIneat producrs ma!
. hnrkor siap1ivlc)coccal e n terntoxins tliat prod~rcc rood ~>oisni~ing).
\IS: tachvclcarcfia: mild fcvcr: livpotc~rsion. PE. low urir~iu-y vollime; eyes
sunken; poor skin turgor jvilh tlr.yne+srti'skin anrl niucous ir~enlhranes; leth
argy and proximal m~tscle weakness (clue l hypokalcniia) . o
rn
z
0
u
z
c7
PE
2:
0
~1
6
<
Labs
CEC: increased hematnct-it (clue to l~einoconccntraticm);l l i crrascd BLrN. Lyt
cs: hypokdemia: hvprrrchlorcini:~.Urk prc-lleirl. uria; high specific gravity. X
RGF:metabolic akalosis. F.CX;: ST-srgmrnt atlrl T-wave d e ~ > t - e s ~ k'owave
s (11r.pokalemia). i ~~:
Treatment
Discussion
Fluitl and clcctrolytr replacernunt.
Dehydration mav he isotonic, hypotnnic. or hypertonic. U l c n c a ~ i s r d prn
lractcd r.orrliting, it leads to mutakolic alk;~losis IJV due tn a rlc-crease iu
li~rl~.ogc.ri c c ~ n c r i i ~ r a ~ i o i ~;I cornion with pensalol-y rise in
P<.o:! (due lo dirri inishecl alcrolar ventilntinn). 'I't~pron tlarr ion o l '
\ o l u t ~ ~ e stimu1:llrs rhr proximal renal tzi hu-
*
METABOLIC ALKALOSIS
ID/CC
.4.5~~.7.'ear-nlcl who is a liiiown non-insulin-dependen t man diabetic (NIDDM t
ype LT, maturity onset) and who has been reccicing an oral 11 ypoglycemic agent
is hrnug1;ht ro the emergency
motn
it1
a stuporous state.
HPI
For approximately 2 wecks, hc had been treated for a URI with oral antibiotics a
nd hroncl-tudilators.
PE
\'S: tachycardia: hvpo~ension, severe dehydration with dry PE: oral iilacosd and
lotv uhinary vol~~rne; i ~t n pat sernicnnscious and confuqed: pupils react bil
arerally ancl normally to light: evidence of prolilcrativc diabetic retinopathy
r,rl fiinrlliscopic exam; no focal neurologic deficil fi>t~nd.
tabs
CRC: mild Ienkr~cvtnsis( 1 2.600). Markedly increased blood glwcase (900 mg/dL)
; increased serum and urinary osmolality (> 350 mOsm/kg). Lyres: hpernatrern ia:
mild hypokalemia. Normal anion gap,AEGs: normal serum hicarhc~nate( n o acid* ~
i q ) .Elek-ated BUN and serum creatinine (suggestive of prerena! ,uotcmin). UA
: glycasuria with no ketonuria.
Hvpo~onic (utle-hair normal) sali~le. Insulin inf~ision(e.g., Iowerdose than in
ketoacidosis) . Potassium and phosphate s u p p l e Inenr as needecl.
Hyp~rosmolal;hyperglyccmic nunkctotic cornn occurs mainly in olrier NIDDh4 patie
nts and is t~sually associated with a n episode of phvsical or mental strrss (ch
eck fill-silent MI): i t i s not associated wit11 ketosis o r ketoacidosis. Volu
me dcpletior~ qevet-e is ( a v e r ; ~ ~ c deficit 25%) or total horlv water). a
nd the mc~rtaliv fluid
Treatment
Discussion
F*r
NONKETOTIC HYPEROSMOLAR COMA
-
ID/CC
A 44-yearilld male is admitted 10 the mrt1ioprclic department hecause he s~rstai
ned femoral neck fractum when he fell from a a small stool; tlre tvpe anrt magni
tude of the fracture arc nnl. cornpatible with the patient's agc and impac~.
HPI
The patient recently ernigratcd from Somi~lia and states that he has her11sl~ffe
ring from increasing leg weakness and persistent lower back pain.
m
z
n
s
-<
PE \5: normal. PE: corripIc~e rigl~t fkmnl-al n ~ c k fracr~~rtr; palpaon
tion, tenderness of lumbar vertebrae and pellic r i ~ n . Labs
i 2 r
? 0
G1
MiEd anemia (Hh 10 g/dL). Lyres: normal. Increased akdine phosphataqe; d e ~ ~ e
a s e d levels of 25-OH-D,; hypocdcemia; hypophosphatemia; itlcr-eased PTH.
Imaging
XR, hip: surgical neck Frmor;~lfiacutre. XR, lumhar spine: collapse of lumbar ve
rtebrae; generalized osteopcnia; pseudofractures {app~arance nondisptaced fixctl
lres representing local of 1,one resorption).
Excess ostenid hut poor mi11eralir.a tion.
Micro Pathology
Treatment
Vitamin D, calcium (and sometime5 phosphate) supplements; surgical treatment of
fracrure, phy~inthcrapy.
A pnnl- diet in vitamin D and calcium, lack o f sunlight exposure, intestinid ma
labserption, renal insufficiency. nl- t ~ r g e organ ret cistatwe may lead to o
st~omalacia the aduIt (or rickets in chi?in rlren), with defective cdcification
of osteoid.
Discussion
7
OSTEOMALACIA
A 4O-r.c~-old male visits 11 is internist for at1 evnlualinn of sudden (P.L~c~sw
M.\~.) attacks OF headache, perspiration, and anxiety; a~tacks are prccipitatud
ky rxerciw, enlotional stress, postural chanprs, and, at times, ~u-itlaliot~. Ve
ry high blood pressure has heen rccordud a t lhe titne of previo ~ paroxysms. Th
e par ient has a good ;ipperir~I L I ~lnoks s I cachcclic: blond prewurc rcco~de
d between prorvsrns is normal. T h c p;llier~lhas tln Ili~tory suggeslive or ren
al discase.
PE
VS: hpertension (BP I#0/120). PE: Ilyprrtrr~sive recinap~thp changcs on ~ ~ I I
I C ~ Z I F C ~ P ~ C exam.
Labs
Elevated blood su~gar d u e to Encl-eased ca t c c h u l a m i n ~ s.) L p s : (
nol-in:~l.Increased "tlinur urinary frcc catecholamines and vanflyImandelic aci
d (VMA) levels.
Imaging
CT/MR: 5 r m lert adrenal mass: very high signal on T'L-weighred M R . Nuc: MLBG
localirrs rn t~unnl- mctastahes. and E n c a p ~lat dusky-colored,rouricl ltirn
or ~llass erl, with compi-esqrtl arl~-es~al glnnd rc~nrlan at periphery and foci
of necrotr sis ancl hemorrhage.
n'es1.7 of pleomotpllic large crlls wit11 hasophilic cyroplasm airrl chrome-stai
ning granules in vascular strclIna; at-gentaffin stains positirr: mert~l>ranr-hn
~lnrl sccrr-tory 9anuleq on electron ini-
Gross Pathology
Micro Pathology
crosci>ptr.
Treatment Trcal h ~ p r r t r r l s i v r crises with pliar~ll~.cologic alpha an
d bera I~lockacl~ (pretreal with alpha blocker^ prior to bcta Moclier5); r esecl
ion o tumor. C
Discussion
P h ~ o c l ~ r o ~ n o c ~i ~ r , m most ctlmmon trrmor of the adrenaI s the a
medulla in adult%: sy tnproins are prc~tlucrdby increased proils duction of cate
chofamines. Of t l i e s ~ tlrtnors, 1 0 Y ~ exrraarc :~cli.enal,10':; bilateral
, 10% malignant. and 10%~ familial; 10% occur in children, and 10% c l i t May h
e associated wit11 aci. mul~iple enrlorrinc ncoplasia (MlCN) IW 01- IIB syndrome
s.
Atlas Links
"
PHEOCHROMOCYTOMA
IOJCC
9-year-old1P1~1ale hl-ought to Irer ~rerliatl-ician ~ c c a ~ ofs e is I ~ breas
t enlargement.
fI~l-motl~el-alsoreportscyclicalva~~ndbleedingancltl~rapglearancr or pubic and a
xillary hair since he axe of 11; an oldet cousii~ rlevcIopccl simil:lr signs anc
I symptoms.
m
HPI
RE
Full) duvrlopcd hr.r;tstu: ;~xillaz-y and pubic hair- pl-ment: tmrrnal
r CI
0
I -
Labs
0
Increased plasma FSH. LEI, atirl estmrliol; plrhel-tnl pattel-n of
9
Imaging
XR: advanced bone age. US:clvai.~ enlal-g~rl prtherr;ll tn
n.51
si7c with
forniaiiorl. (TI'fMK:
ria
pirt~itarv Iesinn.
Treatment
(:ri
RH ;lgr>niuts:p$ychiatr.ic vi1>1x)ri;' "1 r i IIU'C)US P : L ~ - C ~ p c m O S C
nr
hlr causr.
Discu~sion The most connnrm czrusc o f PI-ccocirlusplthcrt!) i s irfiopathic or
r o r ~ s ~ i r r ~ ~ i o i ~ ;m ln ~ ~ t c:irlrrc inrltlrle hypotIla1;lmicleqr
~ : ~on piridt;u-y tltmors (pincxlomaq. ha mar to ma^. ~Iiuinas)or Trsiorrs
'CCI PRECOCIOUS PUBERTY
ID/CC
A 5-year-nld bop is krot~ght the pediatrician because of interto nji ttent numbn
ess and leg cramps.
HPI
His fathcl- is also concerned a b t ~ u lhe fact that his child is t shorter tha
n hi< classmates.
bridge; right convergence
PE FdI, mund face;short neck; flat nasal
squint and left cataract; delayed dentition; positive Chvnstck's and Trous5eau's
s i p s .
Labs
CBC: normal. Ly~es: hypocalcemia (< 8.8 mg/dL); hyperphosphatemia (> 3 mg/dL). I
ncreased plasma PTHJ no increase in renal cAMP and pt~osphate clearance with PTH
infusion.
lmaging
XR: fourth and fifth metacarpals are short; prematum physeal closure; thickening
of cortices with rlemineralizaiion.
Calcitriol and calcium supplemencation.
Also called Seahrigh t-Bantam syndrome. pseudohypoparathyroidism is an X-linked
dominant disorder in which there is r e s k tance to PTH action on the renal tub
uIe and bone with resulting hypocalcernia. Two types exist according to the resp
onse of CAMP lo PTH. In type T (as in t h i x case), patients Fail to exhibit a
phosphaturic resporlsc or increased CAMPafrer administration nf PTH. Type I1 i s
associated with Mbright's hereditary os~rody*
Treatment
Discussion
PSEUDOHYPOPARATHYROIDISM
ID/CC
A 15-month-oldEskimo hrsy is hrotqh t IU the pediatric clinic br his parcnts I~c
cal~sc delayed dentition, poor g ~ ~ w and develof th opment. frcqilcn r cl-+na.
and wraknes~.
T h r i n h n r ' s diet is deficient in eggs and dairy products. and h e spends
I I I C I F ~of' h i s time indclorq (i.~., I~aq exposlue t sunlie no o
'MPI
light).
0
PE
Irrit;~hili!~; pour rnu.;ct~lardevelopmet11 and i n ~ ~ ~ r l e a h tone: dornin
al distention: hypvpotonia of all mlrscles: antcrios fontanrTle open; softening
of occipital and parietaI bones with elaqtic recoil (T.R;ZTIC)T:-WES) i r m tal
1msr;ing: r n l ; t ~ - ~ e ~ n e n ~ ;f of costochondral junctions (~4.c:t-rrnr
. ROSARY); howing of leg; lineal chest depression along diaphragm (HARRISON'S( >
I'I'E) (K :O
Serum caIcium normal or slightly low: decreased serum phosphorus; increased alka
lil~e pl~crsphntasr: low 1,2,i (OH)flitamin
1) le~,el.
r l
E
z a
6 G)
<
Labs
Imaging
SR:widening of growth plates: ostcopcnia of ci-aniill and long
honrs: irregularity
and cupping o 'distal ends elf lc~r~g f honeq; pw~rcit~fracnlr-es rnelaphvsis (
1 , o o s ~ ~ a' s F ~ ) . in W
Gross Pathology
Excess iilnount of uncalcified hone at,jl~nction cartilage; hone of str~tcI1~C1
p ~ ~ l l ~ dnf shape bv gravity; increased ostcuid anrl o~tt scams: ostcopenia:
frontal hnssing of sk~111; pigeon hreagt clefor-
miq.
Micro Pathology
Treatment
Defective mi tr rrali7ation of octeoid in epiphv~is and diaphvsis.
Increase calri~imand vitamin I in die!. 3
Discussion
Rickets i q a diseart. of infi~irty and childfinor2 involving defective minerali
zation of osteoid in h o n ~k c l ~ t n n s and the neusurnziscuIar- syqteln hec
at~w Iow vitamin D or cdcitim in the diel; i r can of ;17sohe due I r low \ t ~
i itigl~iexposure (\.it?tnin D ronversic~nin h skin) ;~ncl chrunic rcnal railui-
r (BLrK and phosphorris Icv~ls arc high).
MJMC-308
Atlas Link
ID/CC
,I\ r31-~en1--(71~1 sm nkrr present< wit11 l~c;~clachc, male wcahcss.
fatigue, and decreased u r i n a output (( ~ L I C ; ~ ~ R L ~ ) . ~
HPI H r was i-ecrn~ l cliijgn OSPII v
wi tll oat CCU carcinoma of the lung.
PE
I:ardiac st)unds ~~nrrnal: rnl~rrnlii-s: :trrt~!th~~~ii~s; no no 1 1 0 pitting e
r l v ~ n nu lirp;~tc~mcg;~lv;jugz~larplrtllor-i~ n o rviclence ~: no ( of card
i:~crlisc;isc); n o ;~steri~is,.j;~llrlrlict', qpider- nevi, or.parotid c n larg
cmcnt ( n o cvicluncc ol1icp;itic disease).
Labs
Decreased serum sodium ( I I W ~i I - I ~ ~ . M:L ~ ) S decreased semrn o s mola
lity (< 2SO m O s ~ n / k g ) :i ~ c ~ r nor;lor% ~ ~ l RUN anrl sel-t~m crea l
i l ~ i l ~1e : p t - n ~ c i n l ~ r i(no renal tlisrase): arll-enaI anrl tllyr
oirl 10 a func,liorl tests normal. U/\: twine osrnoIaLity markedIy increased (ve
~.uls p~vchogenicpoIwlipsia wttcl-e osmnlalitv is dccrrasccI} : 11vpern:itriuria
(l~rinary > 20 mEq/L). Diminished blood Na uric acid level ( ~ ~ r n r r ~ r c
i ~ a .r r n )
Water restriction
13111sa
Treatment
Iiigli-sa li r l i ~ Demeclocycline. ~.
Discussion
Syndrome OF inappropriate (incrr;tst-rl) s~cr-elirtri antidiuretic al' hormone (
SIAI)M) occ~u-q with pit IIPI- increased hypothalamic srcr elion (e.5.. Ch'S dis
~ase. pnrtnp~rari\,e states) or ecropic secretion (e.g., rlittlnrC S ~ I C ~ J n
at cell carcinoma vf thr Ilrng). a< Thrrr mnv altl~n increnretl srnciliyily to L
IIPeffert 0riU3I-I l,c
SIADH
ID/CC
X '19-ycar-ulcl fcmnle is ht-r>tlghtbv anih~~lance tJie emergency to I.CIOIFI ti
c~m Ilcs \vi-r,t-kpli~cc owing to confi~sion, agitation. diarr I i ~ a2nd vomiti
ng. .
HPI
Hrr cistcr has myasthenia gravilis. She gives a histo1.17uf recent weight Ioss (
7 kg) and a recent severe URI.
rn
Z
PE
1s fever (39.3"C) tachycardia w-itlth irrcgolar p ~ ~ l shypotension ': : c: (11
1' 1 00 1 5 0 ). 1'11: irri t;ibili ty: delirium: exoph tI-iaT~-l.tr~q: cli ITil
sc inst~l'~ ~~otltendri-; inaweq: n o pcritnne;d irritation; dcrp nnrl no t e n
d m reflescq hrisk; n o neck .ctiffnew or focal ~CIII-ologic
a
r l
0
U
r
Q
GI
<
Labs
CRI:/Lvteq: noi.iiial. LP: (;SF v ; l l ~ ~ e \ norinal. W:G: i11rial fillril la
r inn. Elevated T ,l r r ~, anrl T,: low TSH. , T,
Treatment
T ~ . ~ a ~ i i ~ e n in hibit ion n r thvntirf hortnotie wni lieqic Iinv02t~es
( r t i t h pr~op~l~hir,i~racil or rne1liirna7ole); i~il~ihition srorcd nf thvro
id hrrr~tlonc(with iorlicic and corticostc~-oicls): sr~ppre'ision of' 1 ht. peri
pIi~r;~l vf'fec~sf t hyl-oi(l I~ol-iilr>li~I 1 propranolol) : n (wi 1 rligiuili7
atinn r)l'parienr<t\-iltti CHF xnrl atrial fibrillation; acetarn itloplien I nr
f@ver;and treatment or prccipit-nting f;lctors
Discussion
Thvroirl qtnrm, :I medirnI elnergencv, i~ llsaalfv prvcipitaled I y s~~l-gir;~I
or n~edical stress (c-fi., infcc-tirm) p1;tcccl on ~rntreatcd o -111idcrtl-uatrd
r Fiypr~-thy pat icn 19. Pi,evri~t rjf postoperaoirf ic tivt. tl-iqes is cf'i'r
cterl tf~l-ollgll LISP or iodine ancl :~ntithv-nid dl-11Ck.
THYROID STORM
ID/CC
27-vc~itr-old Ctil~an political diusidetlt visit5 a n~edica! clinic complnining
or diminished visual acluty, primarily at night.
HPI
Ilc rcccntly arrived in thc Fnited States bv hoat af ~ e sprnrling r su\~cr;~l v
uars in prison.
PE
VS: nor~nirl. rorijunctiva shows diininish~rltear I~~hrication PE: with drvness
(XEROSIS: when locali~ed, forms patches known as it Ritor'~ SPCIIS) 3% !tmell k~
ra~inixatioti as and small rnl-ileal ulcers
(SEROPI ITH~UAILZ).
Micro Pathology
Keratirtixing mrraplasia in conjmlc~iva; f~llicl~lar 11ypel-keratosis with glsnr
lt~lar pll~gging.
V i ~ ~ m Ansupplrmcntation. i
Treatment
Discussion
Vi tarnin A
(RETIVCN.) is a fat-qoluble vitamin derived from p - c a r t ~ t ~ n that is rls
erl for the n m t h e ~ of rhoclopsin in the es i~ retina. for ~ ~ ~ o healing,
and [or epithelial celI growth and nnd differentiation. Night b1indncs.i (~IGV.U
OPFA)is an early s y m p tom c ~ vi tamin A deficienru: conj~~nctiviil f xerosis
a11dRitot'r SPOTS are early signs. Corneal ulcers may; progress to erosions anc
l eventual destrr~ction cornea ( r c r ~ i T 0 M U C t Z ) . of
V I T A M I N A DEFICIENCY
I
ID/CC
A 3E-year-Irl I2lack rnaIe who i~ kt1014-n 10 he an d c o h ~ l i c come5 to the
emergency room with shnrrness nf breath, conf~~qion, foot drop. ancl swelling c
~ his Icgs. f
HPI
H e arlmits to getting drunk a t leass tltrrr tirnes a week. Hi5 diet
consists mainly of canriccl soup ancl cheap "junk foorl" thar Ile eats rlr~ring
the p~rir,cls which h e is t l r > r drunk. in
m
z u
PE
VS: tachvcarrlia. PE: dvsprlea; j r ~ p l a r vcnour rlistention: extremities wa
rm t o t o ~ ~ t cardiomcgaly; l ~ e p a ~ c ~ m r g a l 1 ;pitting h; 2y cdrlna
uf horh lower uxtrcmitics: confr~siun with nystagmus; dccru;~serldeep tendon re
flesrs.
Increased RaC iransketolase activity cocficicrlt; l o w rerunn and urine thinmin
e Icvels.
0 r 7
2
0 0
~1
r-
<
Labs
Imaging
CXR: cardiotn~galy i ~ h w hasal Iilng rongestion.
Ct'er n i c k e ' ~ e~~cephalopathp rhows congcstiori, 11cmol-1-hagrs, anrl necr
osis in thalamus. hypo tl-tiila~n~is (marnmillarybodies). and par.aveli t r i c
~ ~ l a r regit~ns.
Gross Pathology
Micro Pathology
DemveIinizatioi~of pcriplicral nrrves t+ith axonal drgeneration a t ~ r fi.agmen
~ai l ion. Thiamine. Relore arln-iirlist~ring glucose to an 3lct1hnIic. thiamine
shol~lrl given to prwrnt enctphalopatl-rr (d~rt. rlehe to plerinr~or thiamin? i
s1 glycolysir pxthwavs). Alcoholics should also recrivc I or orill f'olate anrl
mulfi\itaminr. V
Lack of thiarninc procl~rct-s LVernicke-K(~~-sak~>ff synrlt-nme a9 well
as high-output heart failure (wet beriberi) and polyneuropathy (dry beriberi). T
hiitnsine py-ophouphxil- (TPI-') is a cohctor for thc k e h s q c l c cnqmes a-k
rt c>glnt;~~-;~ie dr11r.d~-ogenase atlcl ~ r ~ . te \delydroger~ase well as tt.a
nsketolx~e ~ ~ i i as (pcntosc phos-
Treatment
Discussion
plla~e pathwav).
7
VITAMIN B, DEFICIENCY ( B E R I B E R I )
lD/CC
A 4.3-veal--oldalcoholic 1 li~panic m:llc ~ h recently tir~derwri~t o ; I srrarl
gnlatrcl hernix repair 11ccornc.s irri~aT>len d weak, surSer.s a ~ i g n i l i c
a twcight loss. ancl devrlops ik rash o n his Cite, h i s neck, ~t a ~ t d rlor
~si~rri the c>l'I~i.: h,~iids:11c also qr~fl'el-s f-rnrn diarrhea and altered me
ntal statcts.
,%fttcr hi< operation (which i n ~ u l r c d 5-crn small a
HPI
I?( ~welreqectinn), the p a t i c n ~ hecartic. t o r ~ ~ i and anorexic wirh la
ck of ci proper retur.11 of 13c >we1r1111ctionfur al~uiil ~veekq. diet IiacI 3 H
is ber.11 hilccd o n corn J)I.C)(~~E(.IF.
E r v t h c r ~ ~ a t onotrpruritic. I-iypcrpi~mcnr d , scaling r a x h nf ~~s.
t fat*c., 11cck (CASU'SNEC:I;UT'E), and dorurm or hand.;: angular stom:ltitis (C
I-!EIZ~MS) and glossitis: rlirriiriishctl tc~uch and pain srrir:~l o ~ iri ill1
Li)~ui i eutrernitirs: apail~v. conli15ior1, ancE disurie~~zation.
PE
Labs
Micro Pathology
L r k low levds r d 1 1 tt;rrp N-mcthvliiirnt i n i ~ m i d c 1i
Atrophy ancl l~lccrationuf gastric and iiltrstinal rnilcosa; poster i o r cc)lt~
rima shnw 1lr11rr)n;ildegen~r;ttion z~ntl l ~ ~ n v e I i n a t i o r ~ . r
Ch-al I I irotitlamirl~.
Vit;tnki~iRi (n.i,\c,ru)d eficirncv (PFI IAGRA) i%~ o i i i i i ~ o nseen in ly
Treatment
Discussion
alrnlinlics and is Icss l'reqlrrntly see11 in patients w i t h GI disord e r ~ i
t 1 elrlerlr. patients. In pnticnrs wit11 carcinoirl syndrnme. 01. tryptophan,
rlic precursor of niacin, i s usrrl lip to f c l r ~ sern~i inniu. It i q ~ l s
l ~ n laccotnpal~ierl o t h e r B vitamin deficicncics. ly by
*
VITAMIN
-
B, DEFICIENCY (PELLAGRA)
- - -
ID/CC
X9-n~c~i1tl1-olrl~cliit~~em;rleishtmiglitp e r l i a ~ r i c c l i n i c to~l~e
buc:rtlse of listlessness and anorexia.
S l ~ rs the Clil~~gl~C J i111 unernplovrd poor LIT-ha11 couple and i let. ~ has
iic\ el- bcf~lr urn a pcdi;~trici;ui0 1 - take11 an\ medication. e 1 I c r pare
nt5 re pol-r a diet of' iulsuppIcrnent~d cow's d k .
rn
HBT
Z
PE
ITeakn~ss: pallor: hyperkcratosb aurl hemorrhage perifollidtis o f <kin nt' IC)I
VPI. ext~-eillities, Cnl-earin';.and al>rlomen:purp~tr-ic skin rachcs: splinter
hernorrharges in nail bccls, of hands; tcndemcss aricl sl\+cIling distal llenlri
r nrld costocl~orldral of junctions: bleeding gums: petechiae srcn orcr nasal an
t1 oral
a n
n
s
tr
E 5 u
<
Labs
CRC: microcytic, h!pochrornic anemia; tcukmpeliia. Plasma and plarclct Zcrcls of
;lscorhic a c i d Io~v: prolonged Heeding time.
St<:~uhpel-inqreal hrmnrt-11age5:I3o;th IPF;F
Imaging
ancl knees s l ~ o w
"gso~ind glass" appcararrcc of boilcs :tntl epipllyse.i.
Gross PathoIagy C;~-o~zri g r ~ bone shows climiniuhcti o w o j ti rt>r.matio~~,
liemarthrocis, and s~rhy~el-iocteal pel-iartic~~lar and hrnlorrhage: defective
coliagen (vitamin C I~yclroxvl:~tcs prolirlr and lysinc.); endocl~ontl1.;11 Imne
foi-rr~;~tion ceaqes (os~er)l>lasrc i l tr, Term osteoid) ; fa existi 11gtr.ahe
cular hi-ittle and n~nceptihle fl-acturc. to O1-i71ii\corl~icacid (Ibidi doses m
ay
acid
51orles).
Treatment
P I - O ~ I I C P ~ a l a t and o r
tlric
isc cuss ion
Sri ti~miii ( . \ s t : o ~ ~ l c,-\cIE,) C : clcficirncy, or. <nil.cy.i s ol>se
rved in
snlokerq, ot~cnlogirpatientq, alroliolirs, infants, and tllc cldcrlv.
*
V I T A M I N C DEFICIENCY (SCURVY)
ID/CC
X 37-vear-rdd t~ameleqs alcoholic man (with a diet deficient in lrafv vrget;~hle
s)come% into the emergency ~-nnm with weakness, bleeding gums, swelling in his r
iahr k n e e due to hloorl collcc1i0 (TIEMAR~IROSIS). bloody vomit (HEI~ATXEMESI
S). 11 and
HFI
The patient's diet consists of one mual a day of leftnire~s horn Fast-food rcsta
iira~~t.;. was given ampicilh h r dixrhca NP 2 weeks a50 (Iluading to suppressio
n or'titarnin K synthesis by colmi ic hactcrja).
PE
Thin at~rl malnourished witit31 poor hygiene: con,jur~ctival and nail hed pallor
; subcutaneous ecchymosis in arms and legs; right kn e~ hemarthrosis.
:Inemia ( H b 9.7) ; prolonged I T and IrIT;normal platelei count, fil~rinogcn I
r-vel, a11d tl~roml,inI ime.
Labs
Treatment
Discussion
Vitamin K suppl~n~etitatinn.
Coagulation Cactnrs 11. MI, IX, and X are drpendenl on vitamin K for thcir activ
ity (tllrougI1 y-carboxylation). Rmarl-spectrum an tihiotic use, tnaFai~sorprion
, tl lack of dietary ii tamin R rcan sul t in cleficiency maniksterl as hleeclin
g. Since these factors are
ID/CC
X 47-yexr-olrl male high-school teacher visits his in ter-niqr hecatise of chron
ic watery diarrhea and hot flashes m e drinking alcohol; a ft:w months ago he al
so noticed a p e c ~ ~ l i a r redness of his face.
HPI
Evcrv tirnc Ile works or cxtrcises in the sun, Ile dcvelnpg on exposed area5 ( ~
~ T O T O ~ E R M ! I T ~ ~ ) .
ik
r;lsIl
PE
1%: normal. PE: in no acute distr-ew:redness of face; n o neck Inasseq or i nci-
eased .JVP:systolic ejection murmur grade VTV at pulmonary area, increasing will
1 i~~spil-arion (pr~lmnnarv stenosis); w h e u z i n ~ heard: ahdortlen soft and
nentenrler; mild hepatomegdy.
CRC/I,ytes: norrnal. Glucose, BUN, crcatinine. a11d LFTs normal: no ova c)r para
sites in stor~l. YA: incrreaqed 5hydroxpindoleacetic acid (5-HIAA) in itrine (a
prr~chictof serotonin degradation). KCB: Iadder-step air-fluid level$. L-GI: sma
ll howel loops kinked, causing obstruction. CT: ~tal-like ickening of mesentcrv
clue to 111 desnioplast ic retrac ti nrl; vapte liver metastatic Icsic~ns.
Labs
Imaging
Gross Pathology
Firm, velIo~v, ssl~bmsl~rnsal nodule in a segmcnt of ileuin.
Argentophilic ccFIs ( K L ' L C H ~ S W I:F,LLS) i r l thc in~esiii~al - y > t o
f c~ s Lieherkiihn invading inro mesentery; nlal-ked fibrotic reactior~. Octrtot
idc, cyprr~heptacf inr
(sox~.~Tov,.\TIN AUA~.O[:).
Micro Pathology
Treatment:
Discussion
Chi-cinoicl 1 mot-s arise h o ~ n gastroin testinaI tract CH. 13r111iclii. the T
hest tnrnun secrete serotonin ( I F I - ~ R ~ ~ ~ ~ R Y ~ ~ . \ \ I I N E ) . pr
oducing rhe &pica1 clinical ryndromc. T h u ~ c may br strrlosis of r h ~ pulmon
ic and tricuspid valve anil right-sided heart Failwe.
lD/CC
A 3-week-ulcl n1;llr is sccn hy n i~roilatologisthecausc of severe jaundice that
;tppearerl hirtli and has txen worsening ever since.
H e ic; (lie firr~-hr,l-li chilcl of a I~eaI~hyJewish col~ple. His ~norhet. had
ara i m e v e r ~ ~ pregnancy and delivery ~rl
HPP
PE
Avcn~gc wciglit ancI height fur age: in no acute distl-eqs: marked jaundice (jau
ndice appears at Ievclr o f bilirubin around 2.5 to .'3.0 rrig./rlT,); ~ l i g h
~ hepalc1mrga1y.
Labs
Markedly increased senim uncon.jugagzltedbilirubin ( 15 mg/dL): very low Fecal u
robilinogen.
P h e n ~ h a r b i ~ l tvpe 11. Fnr type 1 , the prognosis i s guarded, ibr wit
11 the likelihood o death in l h e first year of life. S
Treatment
Discussion
Criglrr-Naijnr syridromc is an iilhcri tcd rlisorclrr of hilil-uhin ~ ~ ~ e i ii
s111 m l is charac1eri7ecl ~ S Va deficiency or the enzyinr ~ l thi~t glucumnyl
transferaqe and Iience I3v 311 inahilitr; tn con-jr~gatc I~iliruhin. with accumu
lation of it~clirect bilirubin and risk of ken] iclrr~~q 13raiil d i ~ n ~ a g r
hi1 iruhi11 coilceri trat ion5 wit11 (:?L (J 2 0 r~~g/dL). > There a r e twn ~y
peq: tvpe I, which is more scircrcarlrl is a~~tusoinal ruccssivc. and hpc TI. wh
ich is autosonial rlominant.
?
CRIGLER-NAJJAR SYNDROME
ID/CC
X 2 l rear-old f ~ n j a l e lege s t ~ ~ r l tevisits bpi. ga~tmer~rerologist c
rjl n for a n e ~ ~ a l ~ r a t or n i o fatiPhility anrl intpl-mirte~it right u
pper quadrant and epigastric pain.
HPI
She asked her Carnil? doctor 2 refer her. to a g;+ct~oet~ter.ologist 0 because s
he was cor~cerr~ecl abo~ac pait) despite 11~1-c l o c t ~ r ' s her r ~ a ~ r i
~ r - a r ~t~ e "iiolhing i~taportaiit." that i ~.;t\
VS: normal. PE: mild jazandice in coii,juiictiv;l anrl undesneatk tongt~e; ~ l h
ydrat erl and i 11 no acut r tlisi t-ess; nrt 11el~~tospIenow l niegalv o n abdo
minal exam; n o s i p s of hcpatic fiiilur-c.
PE
Labs
Increased direct bilirubin (vs. Gilhei-1's syr~dl-oinr, which in hr7~r1-biliru b
inrrnia is indirect) and indirect bilirubin; liver enqmes mildly elevated. UA: h
i l iru hin and ~u-obilinogcn (vs. Gilbert's S ~ ~ ~ T C F I I I C of coproporph
yrin T and ratio ) ; coproporphyrin H in urine 5l (ilormal = 1) . I :
IJS: 110 ~a1lstr)neu: liver nr~rmal. Nuc: I-IIDA.
tin
Imaging
hiliwv excretion on
2 <
Cross PathoZogy
Liver a~nrin;llsize a n d dark p e e n in color. {vers~ls Rotnr synrlrorne. whic
h has 1 1 0 gross liver ahnormaIities): ahscncr of galll~lacldc*r inflammation r
)r stone<.
C:~nrrilnhl~lnl; lysosnnlal granules rzrith brownish pi~rnvizt(catecl~olaminrs).
Micro Pathology
Treatment
Discussion
Supportive.
D~~'f>irl~Tohnson cvriclrorne i s a Ilrn ign autosomd-recessiveclisorTIPI. (vs.
I~illx=rt's syr~rlrome) derectivc canalicular bilirubin or excretion cliar;ic t
crizcd I,? cpisoclus of ir~trrmittert~ l ~ l l d i ~ e . $i
DUBIN-JOHNSON SYNDROME
IDJCC X 19-yui~~.old with a URI visits hi.; family rtoctnr male
becausr. he
is cr~ncerr~etl i~lmnt yeIlowness i his eyes ( ~ ~ l x n ~ cM c ) , h e n ; hich
has nc~ticerl whenever he is fatigued or. i q surfering froin a minnr inIection
.
HPf
Hc has no I~istnry clark-colorcd ~~rinc.. nf clav-rlrlnrtacI stools, al>rlominaI
pain, blood transfusions, rJr- dl-t~g uw. He is i m r i ~ i ~ ~ ~ i z~ c li n h
epatitis R and dues not rlrink ;ilcol~ol. a ra s~
Normal except for mild sderal icterus: no hcpatosplenomega$ n o signs or chronic
liver. failure.
PE
Labs
Mod t-ril~ely increased serum bilirubin, prcdominarrtEy unconjugated; nnrmal ser
nnl tran~anli11ase5 anrl a1kalinc pl~os~-lhatase: nr)rlnnl serum alhurniri: seru
m bilirubin rises after 24hour Fmt.
Treatment
N o rnutaholic 1i.earment m i l a h l c or- necessary.
The most corninon oxa~nple irliopatl~ic of hyperbiliruhincmia is Gilbert's disca
xe, wl ~ i c h s autosnrnal rloniin ari t with rariablr i punrtrance. It i q due
ro defective hiliruhin uptake by liver cclls anrl low glncluonyl transferaqe ac
tivity. Bilirl~lrir~ Irvels seldom rxceed 5 n qJrlL, nlairlly unccrti~jnga~ed, r
nily wry invcrscly allrl
Discussion
"b* GILBERT'S DISEASE
I D/cc
A 1 9-~~=:1~tolrl is brought to her familv doctor hy her parfrrrlalccnIts. who h
aw- III>I~CC'CI thal she has <tarter1behaving oddly; 2 davs a g n thcp ncjtiucd
that 11t.r eyes were yellow.
Sltr. ;1140 ~ ' o t ~ ~ ~ > l ; lc i11'~tremor o f her hands at rest ancf somr t
'; rigidity w h e n twin9 tl I grasp ~ l > j i ~ ~ t s pulgli.1 affcc~tirnn) (b
asal . TIIP parents or the patient xrc first cousins. F;llicrlt S ~ O M ' Sfli~p
pi~ig Lrcn~clr(.x~I'I.:K~xI*;E nF hand<;slil-latnp ex:~nlina tion rcvcals copper
deposie in Desccmet's membrane uf ~ l i c -cr lrnca (KAY~~.R-F~lrR ~lh;i;s); ah
dnminal pal patir ln rlsrr ~hrwc rla0r1erate splcnomeply.
HPI
PE
Labs
CBC: Ilcmolv~ic n u m i * ~ a (ilur L o uxitlati\,e IZRC: darnage IT c a p per)
. AST and ALT elevated :is wet! ;IS ;jtkali ns phosphahrc c :IIICI hilil-t~hin,
,111 clitrct ;~nrl t ~ d i ~ .decrease in serum I]( i ~c~: ceruloplaqmin (rolrl>
el--~ti~por-t protrin) ; increased urinary 1-3 ing copper ( I l ~ l l ~ ~ ~ ' t
l l ' ~ :l incre;~srd ~ l ~ t . \ ) ~~rin;tr.y ;*rid tu-ic
Imaging
CIS: enlargement of lir-rr nnrl 5plcr.n.
Gross Pathology
Micro Pathology
Cnpper a c c ~ ~ t r ~ ~ l l ailzi n n hrain, and cnrnca. t liver.
Livrr hiopn. shows acute. inflam ntirtioii. i n n - r i ~ w d cnppel- Icvel~, pe
ril~nl-131 Ii!>rosi<( n n c r - n n o d ~ ~ l a r cirrhosis); intracytopla+ mic
hyaline bodies ( ~ I A I . LFI<Y I~OIII~%I .I scc11: degeneration of basal p d i
a wit11 cavitation. cspr-ciallv of piltamc-13:hypel-plasia wi111gli:ll pl-ulifc
-~-;itiol~ ~ l l c~ n t i c ~ ~ I i i~ I C ~ P ~ . of l lI r
;111tl
Treatment
Discussion
M'ilst m ' s clise~sr.s ;In autosomal-recessive in [let-iteddisorclrr of i c.opp
el- ~ ~ i e t a h n l i n~appecl chnmlnsornc P 3. It is char-;~c~el-s~n to i ~ r
d increased absorption of copper from the intestine and Iw diminished excretion
in the bile tviltfl t - c s ~ ~ l lrrlpper rleposia~~t zion, primarilv in t h r
l)fi~in ancl liver.
A
WILSON'S DISEASE
ID/CC
h 2:3-~~ear..olrl whitc female i s brought ro the ER because of strange, dreamli
k e hallucinations and blurred vision [hat she csperienced 1 rlay after spendin
g all morning in the sun painting h r r I ~ o ~ t s e (exposure to sun Inav prec
ipitate a t t a c h ) .
HPI
'The patient had i~ndergorle hvo previous laparotomies for apparent acr~tc abdul
nen. hrlt no pathoIogy was found. S 1 I ~ a s 1c had srveral cpisr)dc.sof recurr
ent abdomhd pain.
PE
1's:110 fcvcr or tachyrardia. PE: pupils ;Ire of unequal sizc (ANISOC:ORIA):geue
nlizccl weakness and hypwactive d e ~ p t e n d o n retlexey disorientariun; foo
t drop: urine very dark and Couls~nelling. o photosei~si skin lesions. N tive
UX: increased urine parphohilinogen and y-aminolevdhic acid. Lyles: hyponatremia
.
Liver infiltrated wi i h porphobilinoger~; central and peripheral ncrvous spsiem
n-ivelin sheath degeneration.
Dcgeileration of mvulii~i sheath.
Labs
Gross Pathology
Micro Pathotogy
Treatment
Iligh-carhohyrlt-ale cliet; glucose; lwmatin.
Acurc i n r c r m i t t e ~ ~ i posphy-ia is a n autosomal-dominantdeficrien~y a
t i enryrne nf porphy~in in rnetaholisrn (porphobilinagen dearninase) that lead
s tu sys(eimic symptoms. acl-11~ abdominal pain. ncuropsychiarl-ir signs and syn
lptotns, and CNS and peripheral neuropathy. Acutc inlerrr~i t porphy-ia i' difrc
rentit teta ated frntn other. pol-phvtias hv i ~rlack of photosensitive skin les
ions. Sun exposure and drugs (c.g..sulfa, barhituraltcs) rnry precipitate aft ac
ks.
Discussion
*
ACUTE INTERMITTENT P O R P H Y R I A
A 3-year-olrl farmer from FIorida (with abundant sun rxpostrre) rclmeq ~osee his
clerni;~tologist fnr ail ut3altt;~tir~n t-r01.3 cell1 increaqe i size and chang
e in color of a skin lesion that ha4 n ~ C ~ piwwr I ( H I the dorstum of his h
a tlrl (a sun-exposecl arca) T I f i l l 4 vearq. ;
Tlle patient is nn albino, I I L I ~ hr has not b r e i l ahle rn comply with hi
s dermatologist's orders to wear long sleereq whilr working i t 1 the Iie3d.
M i t e hair, including eyelaqhes and eyebrows; eye exam shows
nystapus arid poor clevelopment oFmacula wit71 hIuc iris; poorviqual acuin. /9(3
/3.:713): ckin is piilk-rvhi te width lack of pigmenlauon t l i r r ~ u ~ I l n
boclv: nuinc.rr,lis actinic (SOLIR) kernloses ur on f a c r x11.rl scalp as well
as on clo1~11tt1 t~anrtr;dcerated of lesion with indurated edges on tlorsurn of
hand with l~yperpigmentatirrn.
Labs
Gross Pathology
Tyrosine assay shows absence of i l ~ e en/vme. Parche~ scaly, i r r e g ~ l a r
h!.pertrapt~icd skin in suri-rxposed or ,
arras (actinic keratosis).
Micro Pathology
Riopsy clf lesion o n dol-511mor hand shows cpidcrnmr,id (squamous cell ) cancer
wit11 rpithclial prar1.r.
Surgery and/or clirmothcrapy T r rkin cancers, avoidance uf o sltn espoulsre, ~n
;nlagemct>i visl~ali~~~pail-lmlent. ol
Treatment
Discussion
1211>inisms a Eier~ditarv i ~ ~ r dthat may hc gcncralizud or i d er IocaIizcd a
nd is tr ansmit~cd an a11tosnmal-clo~~iinanr as or a~r!o\ont;il-rrcr~~if~p 11 i
s aFw;~yc: ingui4iecE I 'various trait. dist n degrees or h y p o p i p e n ~ t
i o n " the skin, Flair. iris, and retina. o i Tl~c dufcct lies in rhth pigmenta
tion, rlnt i l l the n ~ a ~ n h e l of mclanon-trs p r c s c ~ l l the h o d r
T l ~ e a u s ~ an absence of ill c is tymsinaw, the enzyme that cat;ll!~es thc
c o n v c r s i o ~ ~ t!.rcjr;ine of ln di hydroxyphcnyIa1a11i11eand melanin. Th
ere is a marked inrr-rase i 11 t h r i ~ k k i n cancer. ~ OC
'A
ALBINISM
lD/CC
A 3'7-war-old man presents with dark, 'blackenedspo& in h i s sdera and ear car
ti la^ a wcll as back pain and restriction uf s motinn wit11 pain and swelling o
f both knee joints.
Dircc tcd questioning reveals 1 hat his urine turns bhck if left standing. Incre
ased pigmentation in ears, conjunctiva, nasal bridge, neck, and anterior thorax
(OCHRON(I.SIS): arthritis of spine, both knec joints. and fingers.
HPI
PE
Labs
IT/\: elevated wine homogentisic acid (causcs urine to darken
lipon standing o r with addition of a kaZine substances). 1
Imaging
XR: calcificarion in cartilage or knur menisci and .rt~ist; premature arthritic
changes.
Symptomatic treatmet11 o f arthritis.
Treatment
Discussion
Alsir called ochronosis, alkap~otiuriai s an a~~tosomal-recessil~e d i s o r d ~
of ~ ~ o s i ~netallol characterized by the absence of r ne ism homogentisate ox
idase due to a clefectivc. grrle on chromosome 3 villi accumulation ~Tfiomogenti
~ic in cartilage, giving a acid dark blue discoloration lo tlte lisstles and lea
ding to degerlemtivc-joinl diseaqe.
7
ALKAPTONURIA
ID/CC
A n 1 1-year-old white female is Ix-oi~gIltto t t l r ER b.r. he1 paretlls h e r
a ~ i w I'F.ver: difficulty breathing, and a productive cough of with greenish
sputmn.
HPI
I-lrr pxcnts arc. nf ncjrthcrn Enropeall rlescenl. She I ~ n s hisa tr)rv nf rec
urrent Urns and foul;smeKmg diarrhea sincr infancv.
PE
V6: tacliycardia: r ; ~ c l ~ v p n ~ a 4.5). IT.: mild c-yann'iic;malnour(IiR i
s h m e n ~n a d polyps: 11l;perrr~nn;lnre 111tlgp e r r ~ ~ w i o n : In wit11
barrel-shaped chest: scattet-erl mlec: hel>a~ameg:ily.
Labs
High sodium and chloride corrccr~ti-a~ions sweat test: in
J ' ~ ~ ~ t ~ i o r n n r ~-n s~ i ~ ~ r ITo~~no/tlrilu m u rm, 7 jn/i'~r rnznr
and S/r~pI~dntnr~.lr,r nrrnw r in uputlim cul1u1-e.PITS: increasetl W J T I,C: 1
.ntio. 11lc-1 rasud fecal fat. ABG: Iwposeinia: hlperrapnia.
Imaging
C X R : lkw rli l i ~ ~ h ~ n n c lii ( n ~ o x ~c ed
~xs~u) ~ c iillcd with mucus; c ~ n p l ~ v ~XK, parannsnl s i ~ ~ l i sopacific
atioii of sin (Isrs. c~~~a: r~:
t
~1. rn
Z
Gross Pathology
.4trophic pancrca.; ~%itli alrnmu~ cornplrtr ~liur.t~ptiol~ r l f acirli and
rn
Micro Pathology
Treatment
Infliui~rn;~torc l ~ a n g e . v ,\iitihiotics, Init~-'~t diet, atad qltpportive
mrasures: rrcninl>inanl hu~nan DKnsr (clc:lr~escstraccllu1:lr DNA froin ncutrop
hils in ~ p t ~ i i ~ tit111aIpcla~nilor-ide, ii): ni~qal rni.tirc)sreroicls. sv
stemic col-ticnctt=mids. dccnnjicctrln ts: ctlncidcr lung b-ansplan1.
I:v\t ic iihrc~qiqir an autasomal-recessive r l i ~ ~ a trIe a ~ d n e to a l is
mu tatill11 in llic Tong a r m of chromosome 7 (lli~nd qS1) in the
Discussion
fillrnqiu translr~embrxneconrluctance replatnl- (CFTRI IT CFTR function is dcfic
icnt, chloride and waier transport i~ ~In\vrcI ancl ~ e c r r t i n n are in3pis
fatccl. s
na5ric
g-rne.
A
CYSTIC F I B R O S I S
ID/CC
,4 1 5 - ~ a r - o l d femalv is hrougllt to thc cmcrgcmcv room from school Col!
owing the s ~ ~ d d development o f severe, intermiten tent right-flank pain tog
ether w tlz nausea, voiniti ng, and blood i i her urinc (a picture typical of re
noureteral qrot~e). n
HPI
Her medical and Fdnlily l~istorv u~lrcmarkahlc. is
PE
W: tachvcar-dia; ~lorrnal BF: sIight fcwr. PE: short stature (drie to lvsine def
icienn,); in acute distress; constantly switches positions in bed (dr~r: renal c
olic) ; ahdorninitl trr~dc-r~less; to no l~eritnnealirritation; co5tnvel-tehral
angle renctel-new.
Increased urinary excretion of q t e i n e , ornithine, arginine, and Iysine cl1
1 ut-ine anlirio acicE cllrolx~a~ognphy to it~testitlal (diw and rcnal defect in
reabsorption).UA: hematuria; hexagonal .cryst:rls ( ~ f I.:IVE) upon cooliilg o
f acirliiled urine sediment. 1i
Labs
Imaging
KUl3/1VP/C:T t~i-ography:radiopaque ctonr i area o f righr n kiclney.
Low-methioninediet;i ~ ~ r r e a fluid intake; nl kalirlize urine: sr
Treatment
penicillarnine.
Discussion
Gstir~uria an autosomal-recessivedisorder of rlihxic amino is acid metabolism {d
uc to impaired rcnal tubular rcabsorptien); it leadr: 1 o increasecl c y ~ e i n
e urinary excretion a r ~ t kidney stone l
formation.
v
,'
CYSTINIJRIA
IDJCC
. 9-~.~ar-+lrl i s bl-ol~ghrto the rrner.Rency rnom with pain, 4 Imv
i n a h i l i ~ move his left ~ h n ~ ~ l i laenrd: flattening or the nor-n-ral
to r ~ ~ u n d r d c ~ ~ l contour (,SH<)CIE.LII..I< I S I . O ( : I I I ~ ) N )
that sh d~r I) rjccurred when Ile tried t o h i t ; hall with Ili.; hxt at ;r I
ocaT I baseball field.
HPI
He I !as dislocated his Ieft shotalder nine times before and his right shoulder
three times before. He also has a hi\tor.y o f eav bruising.
PE
Hypet.efastic skin: "cigarette paper" srilrs in arm? of' tl-it~rrlla: hyperexten
sibility of joints; lcft slionlclrr disic>caterl;111i1lriple h r ~ ~ i w s skin.
nyer
Clu~tingprr~filcnorinal.
Labs
Imaging
Micro Pathology
XR: lrft shouldcr clislocntcd.
C,
Cnlli~~en fibrils ol'cier-~r~is skin larger than normal nnrl irt-PWof Inr in out
line nn electron inicroccnpv.
9
= I
Treatment
Stipporrivr. Ehlrl-s-Danlos ~ n d r o m e also known as c l ~ l i s is hypercla~
~ica. Faulty collagen synthesis p l - o d ~ ~ r 1 s mpes or E h l e r ~ D a n l
o s e0 s3-nclromc, SOTTIC of which arc- autr)sumal suce5sive (type m), otllers a
l~~ocoinal dorninant (type IV)).2nd others al;.;ocia~ed with N-l in kerl receqsi
vr t ~ . a t ~ s ~ t ~ i p(s i y e IX}. Pror~eto a n ~ u r v ~ m 1 on ;md diwect
ion in the gl-eatressels.
Discussion
"
EHLERS-DANLOS SYNDROME
IQ/CC
A 17-vrarsIc1 male prcseilrs ~vitli r.pi.i.orlt-sof' painful, btlrning paresthes
ias along his palms and soles along with markedly
diminished visinn
it1
his rig11 t eye.
HPI
I-Iis maternal uncle died of chronic rend failure ;II the age of 40.
Cl~r.;t of purplish-red, hyperkcratalic lesions o n &it1 arn~uncl ers limbilicus
. h ~ ~ t ~ o c k f . ccsotum ( : \ V ( ; I ~ K E U ~ I O M A S )ri@t and ; corn
eal leukomatous opacity neum2oqic exan1 normal except l i w painLi11pxreqthesia-
; along arnlq anrl soles: pitting edcma in lnlver extre~niues.
PE
Labs
Elevated serum creatinine and RUN (p:~ti~tlts u5rlnlly die of rr~1;iIfailure). Z
TA: proteintat-ia: hroarl casts. PHS: l e r t k o c v t e 5 reveal rleIiciei~cy
or+di~ctosiclasc. of
Lipid deposiriori in epithrlial and enclothelial c ~ lof ~ l glornuruli ai1~1111
j11lesF ~ > A Vc~1.1.s) renal hiopy: Iysosornal accrum~~lat ( on don of gIycosph
ingoLipid (ceramide hihesoside) in thc fornr of "inyclin bodies" on c l c c t r
o ~ ~ rnicsuscvpl; of ski11.11 t.ni.r, kidt~rvs, n~ld CNS.
?'rear pain crisps sympto~nrlcically; rcnal failure nl;w rrq~tirc
Micro Pathology
Treatment
renal trans1~lat1tation.
Discussien
Fahry'~ rtisease, a sp37 ingolipirlosis, is a rare X-linked recessive disorder o
f g!v~osphingc~li~~ic~ ~neli~holi~rll I I S Phv a deficienq C;~ ~~ of a-@actosid
a.;e A a n d Ilv t h e consequent nccumulation of
FA B RY'S D I S E A S E
ID/CC
% 28-ymr-old wl~i tlraIe cor~~plai r )I' s c w e retrosternal pain . te 11s radi
ating to his lefr arm nnd+jaw.
HPI
Hr- 1 1 i i ~ llad a physical exnm in a long time. IIc adds tlmt his not father
died at a young age of a m y o r d i d infarction.
Aztgt~iuhed. dvqptiric. clia~>llnretic male wirlr hand r.li~~checl tn C I I P F
~ (iI I ~ ~ Y P C Ii x n or mvocarrlid pain 1: soft, clevated plaques s on eyeli
ds (s,4vrrmrASVW ): arcus set1 il ir: pail11111 xmthornas of Achilles tendons a
n d patellae.
F.Iey$~rerl Cl<-\lR: elecaled moponin T and I. ECG: MI. Extremclv high levels o
f LDL.
PE
Labs
Gross Pathology
Premature atherosclerosis in Iarge arteries.
GI
Micro Pathology
Treatment
Fnam cclls wi1E1 lipid chiu-ktctcriutit o f ;tltleroscleroric plaqtlc=s.
Ui1.1,r x e r c i ~ e and c l ~ o l c . ~ t e r o E - l c ~ ~ rlrugc -lia l~ fli
~ ( ~ t lH M G .~~r~ ~ t gl~~
rn
z rn
A
cn r,
Discussion
Farnilial hypcrchnlustc.rolcmi;iir ;~lsc~ c;illc.d t?y,e 11 hvpe1.l i p n ~ ~ ~
( i r l r ia: i I i.: a1i autosomal-dominant defect in LDL 'r rn receptor with a
ACIIC f r c q u c r ~ ~ v 1 :300. Hc)TT~o/!e 111;iv h a v ~ of' p t ~ ;III LDL
C O I L ~eiglii t i 1 7 1 ~ l ~ i j !of I I I I Z ' ~ I I ~ I I . I~ i
"
F A M I L I A L HYPERCHOLESTEROLEMIA
IDJCC
A lt<vear+lcl white I'emale coinplains or slidden midepigastric pain and nausea
after rating french fries.
HPI
Her history reveals that she and a sibling have hacl similar episodes o f abclom
i~~al in the paui. Carrfitl qurstinning pain rlisclnses that she experiences flu
shing every r i tne she drink4
dcohoI.
PE
NonpainFul, yeflowish papules on race, scalp, elbows, knees, and btlttocks (ERUP
TRE L~YTHOM.\TOSIS) lipernia I-etinalistjn fiindu; scnpic exam: hepatosple~~om~g
aly; o n ~ i n a m~iscle abd l guardinga n d palpable tendernpss.
Labs
Elevated serum amyIase and lipasc; very high h-iglycesides:moderate ~levaiint~ f
' ~ e r u n ~ .n cholester~rh and pliorpholipids.
Lipidladen foam cells.
Micro Pathology
Treatment
Low-fat diet: woidailcr of alcohol: rxer-cise:fihlic acid and niacin in ~electer
l caws.
Discussion
Elmilia1 hypertiiglyceridemia is an autosomal4omir1an t disor-
fi
F A M I L I A L HYPERTRIGLYCERIDEMIA
ID/CC
I)oy is rtft-rred to the pediatrir clinir for evaluation of anemia and multiple
developmental anomalies.
, 4 1 1 P-vear-old
HPI
Hi< parenis r-rpnl-tthai lir bleerts: easily.
Pale and mentally retarded: qnlal! hear1 ( M I C R ~ C : F P I I U , Ilor%. ~):
ht=ig-hi anrl weight h r a g r : li~erl>igrt~entatirrn torso arlrl of thighs wit
h cafC-au-bit spots: decrease i size of penis; dccl-case n in s i x o f eyes ( M
I C : R U P K T H , X ~ ~ -absence of both thumbs. \); CRC:: decreased M%Cs ( L
F L ~ K O P F X I ~ ) . pli~telt?l~ (~rl~ou~O(:\-lr)1~1'N I). arlrl KIIC;s (AKE
LIT\ ) (pwtsrom:.wr ' ) Incl-eased 3rrcls or I 1. HhF. llone marrow c Ill-oinomm
es rhorv divcrsc alct-I-ations (hreaks. constrictions. and translotatir~ns).
PE
Labs
Imaging
Treatment
Discussion
SR:bilateral absence of radii, TiT/C:T: hypoplastic kidneys.
Marrow mansplantation,andrugens, cr~rticustrrt)ids.
Far t cotli'r arseniia ir: a cc>tlge:pr~i~al. .I!11ocnmal-recrssi~~(1disorder ch
aractcrizccl 1 7 COIIS titlt tional aplastic anemia due to defective ~ DNA repai
r, pre.il~rnablvI a rcsull of' viral i~lfectiun :S causing hyperscnsitiv-ity 10
DNA crors Iinkir~g ,:<en ts. 11 if a\sociat~d with mt~ltiple ~ ~ l ; c u l o ~ k
e l eand v i q c ~ ~ -;uiomalies, proximal renaf m tnl al tuhulr~r acidosis. an
d a higher incidcnco of Ir~tkcmia.
FANCONI'S A N E M I A
ID/CC
A 10-ycar-old male is rcferr.erl lo a gelzrtic evaluation clinic by hi.; pcdi;$t
i-ician h ~ r a ~ l 01'e mental retardation. s
Hir rnorher- did n o t take any drugs d l ~ r i i ~ q pregi-tancy,did 11er
no1 s ~ ~ E e - r any major illl~esses. from was seen hv an obslrtriciati
HPI
per-ir>rlically, n d was munitorc-cl ir~lr;ipartuln. a
PE
Paticlit well cleveloped physicallp with ,grade 1 mental rctardation; nt) cridcn
ce uf cardiovascular, gcnitr>urir~arv, hepatic elCI~FC~I~U. Paticnt has been suh
jrct ed to hasir ancl cnrlucrinologic lah profiles, a11 of which have yielded no
rmal r u s u l ~F;arvor).pe: . "fragile gap'" at end of the long a m on X chromo
some. r Supportive.
Labs
Treatment
Discussion
The second most common cause of mental retardation after R c ~ w ~ i synrlrnmc i
n males {wo~nexl carriers), Cragilc X s y 's a1.r rlrol~~e slrould he suspected
in any tnale patient whose menml rctarclation cantlot be explained hv otlrcr dis
ease pmcesses. I t is oftrn ilss~cia will1 inac~.~-ctrcllidisrn, te~1 large ears
and jaw, a Iiigt~piicIieri voice. ancl corlllectivr r i s s ~ abnormalities and
~e demonsti ares genetic anticipation (wc>rscni~~gthe C ~ ~ S O T ~ F T of in $
(~rrpssive generations) or+-ir~g the expansion of trinucTeotide to
repeats.
ID/CC
X >-HIOH~II-OIII ~ I i i f e inale is taken to his farnilv doctor ~ C C ; ~ I I
F C of' Ir t ltni-gy. feeding difficulties, and yellowi41 skin (I.LUN~JII:I.:).
The cl~ildhas Iwell vomifing o n ant1 om sincc t~irtli.
11-1-i t:thiIi~; jaundice; cataracts; hepatomeply: g ~ o w t h and drvclol3111en
in fifth pe~.cenlilc; t rduma.
HPI
PE
Labs
LTA: gdactosl~ria: m i ~ ~ o ; ~ c i d t ~ rh a ;~ ~ ~ l i ~ Hypoglycemia; a ill
i ~ iz~ria. inc-r.c.;~\eti ,\LT and RST:elevated direct ancl inrFi~-cct bilir.~
tl>in: prolonger1 1 . p r ~ ~ h r hnvc markedly reduced galactose-1T ' ~ r ~ t ~
~ phosphate uridyl transferase activity and rlcxtt t.(l #alaciose-1pl-tr,spl~at
c.
Gross Pathology
Eal-lv hepatomegaly and Ihtty changc with gi;~rlt crlls lending to cirrIiosis: g
liosis of crrel>r;+l cr)rAteu. h;iqal ganglia, and dentate ~iuclctis crr-rhrllum
: catamcts. of
Livcr. eves. anrl ]>rain most s r ~ ~ c r i~flrc~ecl deposits of ul~ h!, plactos
el-phosphate and galactitol: kirI11ey.hcar r. a11cl spluen alw in.r~nlvetl.
Micro Pathology
Treatment
Limit in rake o f milk and n her galxctose- 2nd I ; ~ c t o ~ e - c ~ t i ng r 1
;li~ii
Ihocls.
Discussion
C:alaotuse~ni;ii.i all autosornal-recessive lack of c n n ~ n c galactoseI -phos
pl~ate ul-irlvl rt-ansle~ase; pl-cwncc of cataracts differthe
ID/CC
An I 1-yr.ar*ld Jewish male pi-tsenh with weakness, epistaxis, and a left-sirled
ahdaminal inaqs.
I-le has a t~istory b n ~ i s i n ~ and sustaining fractures oT easily following
minimal trauma.
Me11till r ~ t ; ~ r iioii; m nltiple purpuric patches: skin pigmuntala~ mild h~
patomegalp; massive splenomeply markud pallor:
HPI
PE
t ion:
110
Iyrnphaclenopathv or ic~el-us.
Labs
CRC: 11urmocvtic, nor-mochrorn ic anrmia: thmrnhocytopcni:~: low norm;tl MTBC co
un I.. LFTs normal; bonr marmw biopsy charac~eris~ic; trol;~l~d MrKCs c l ~ r ~
~ o i t rectucecl P-glucosi~ltl-ate d a s ~ c t i v i ~ elevated serum acid phos
phatase. a : XR, spine: hico~lrave(H-shaped) vrrtebral borlies. XR. knee: Er3cnm
t.yt.r flask deti>rmilvof distal temur: nsteopenia. CT/US: enlarged q p l ~ e n
with mtiltiple nndules.
Bonc marrow t,iopsy s l i o ~ myeloplirhisi5: r~plared s Ily Gatlcher's cells 21
1 to 1 0 0 pin in qize: characteristic w e d paper" cytoplasm d11e to int racyto
plasmic glucocerchroside rlepnqi~iot~; 4 stain poqirive. T S i . 4
Imaging
Micro Pathology
Treatment
Symptomatic. elr Lyme replacement with purified placental or recornbi~~ant P-gl
~~coqidase, acid splenectnrny.
Gauchcr's disrase i s ar I autosomal-recessivedeficiency of gluc~cerehrosida.;e
with accumulation of glucosyl-acylsphinaosine in hone marrow, spleen. and Iircr.
Discussion
"
GAUCHER'S D I S E A S E
ID/CC
..2 .5-1nr)nthoIdmale is b r o ~ ~ g l i the c l o c t ~ I~ecause TreI tu r or q
i 1 1 r nausen. vomiting, nidrt S W P S ~ I~ . u ~ c ) T ~and lethargy. e1 , I ,
HPI
Wlcn the t w . esclusivulv breaqt ferl (i.e., ~ L I I - ithe inia; ng tial f011r
rnorlths aRer birth), h e w;l<~vell; onset o f symptoms 11ie coincided with I h
e occasi(~rr;il addition of fruit-juices m the I>~tl>v's ciict.
PE
La~y-looking. slightly jaundiced I~ahy: tniltl g r o w i l ~ retardation; hepato
rnegaly.
Labs
Marked hypoglycemia; rructosernia. UA: li?ucIos~~ria: tcst ui-it~e For- roducing
cuvr positive; dipstick ti)r ~Jucosc ncgitivc; fr,ucto\e tolrr-ance test not ad
\ isahlr ( m ; ~ v causc- srceI t.
h y ~ o ~ l ~ c c r i.i i a )
Micro Pathology
Treatment
Liver biopsy I-evealslow aldolase B activiv {confirmatrrrv test)
Rct111-nto I~rcastfeed i and c;wert$.
t q as
fole f UOC!;
i t ~ 0 i ~liui I jliicrs, 1
fruits,
Discussion
:In7 foorl containing Cr i~rlose sucrclrr (fr~rct + glucow) or osu miw r atrse s
!mptoms in patients with l'r~icrr~w ii~lolerar~ce, un autosomal-recessive defici
ency nf aldolase B ( c n n m e used to .ipl il Iructosc--1-pho5phat in to g l v
c ~ r ; ~ l c l ~ h v d e clihydroxyancl acetorie pliosphatc), resr~lting n nrcu
mttbtion of f1 i~cto.;t--1i plloqpt-ra~e wilhin livcr uclls. This iuliihits gIyr
olv.;is, gl t ~concugcncsis ct g l y c o g ~ r ~ ~ ~Ifylc~~.$-sta~~cling. an l s
i~. it rn? leati cirrl~osis and kidney IhiIul-P.IXf'tel-rntirll di;zgrlosis is g
alac tr-lqctinia.
41
HEREDITARY FRUCTOSE INTOLERANCE
ID/CC
A 9-!.car-uld male i s referred t o t h c pediatric ciir~ir I>ec;ll~sc of ~xng~-
essive mental retardation, diminished visual acuity, and hone deformity in the t
horax.
Thc hny ~ p n s hm-11 in Malayski and never had nnv prenahl scl-cening.
HPI
PE Tall and thin with
elongatud Iirrths (Mar-fatmid appearance) ; finr liair; abnomalIy long Fingers 1
. 4 ~ 1 ~ 1r ~ n ~ r; pectus I'I'I-I') : excavatum; lenticular dislocation (F.C
TOPIA I.T.NTIS) ; malar flus11: high-arched palate; genu vnlgr~m; carrliovi~scul
a~' exam normal.
Labs
Increased serum methionhe; increased urinary homocysthe.
XR: generalized osteoporosis.
Imaging
Micro Pathology
Rrain gliosis: Fatq liver; arterial i n t i ~ n a l thickening wirl~out lipid dc
posiriuli: drg~nel-xtion ztjnuIar Sig;tn~ents Icns. of or
Treatment
High-dosc
~ I T ~ ~ U X ~ I ~ C rol- cystathiorline (cofi~cror
~ytithe~se;
eCl.i~ctive only in scJrrretol-ms of disease): ~ n r t l ~ i o n i n e r e s t r
i c t e d dirt: cf-stui~lc. and folate supp1ume11t.i.
Discussion
E-Iomocystinuriii is ;tn ar ~rosnmal-rccc.ssivr disturbance of rnethionine metab
olism c a ~ ~ s Iw a deficienq o f cys~aChionine ed synthetase in liver cells wi
ll1 accu~nlulation homncvstine. Majur arterial of and venous thromboses are a c
o ~ ~ s t athreat herause oCvessrl nt
*
v
-
HOOMOCYSTINURIA
ID/CC
I\n 1 l-vc.:ur-uIrl male is qcnt to the a ~ ~ ~ f i o r ~clinic v his pcdia~ e t
r I)? t ~ j c i ; ~ r ~ 3n c ~ ~ l ~ ~ a r t fideafness. For > o n
HPI
His tearhers
r.e>ult.
note that
sclic~r>l i~rid~ r l r lrhxt his aradpmic i
hc has nut hucn p x y i ~ ~atrenrion at g perfc,rmancr II;I\ SIIFI'P~PCI
as
a
PE
Coarse Facies and large tongue; 4hc1tt Ttaturc: corneas clear (YS. Hirr ler's di
sci~se) dimplecl &in in hark of arnrh atld 111i~hs; : n o gibl)u< ( a c ~ ~ i r
- a n gkvl) hr>~i%l le ln-esen L (vs. I-IUI-I er's rlisr,ase) ; r l r , 1 1 1 ~
i t inl'ril nt,rh~lal- ~ s i o n nt1 Teft scaprili~r I s arca; \ t i f h n i I I
or ~ joi nrq; deaFness.
I
Labs
UA: increased urinary heparan sudfatc and dermatan sulfate.
Metar:rrpal
I 11 irkrniiig
Imaging
Micro Pathology
with tapering ;it erlrls.
llrtachrc~rnaticgr-anule5 ( R r i ~ u nonr~.:s)in bone marrow le~tkn.I
cvtc?; ~ i n n i o i i c Il~iirlr u l t ~ ~ r le~ ~ r i n g pregnancy
rn;tv rlerecl
rn
ahi~ort~talin:
Treatment
Discussion
z rn
2 irl
S~ippr~r-tivc.
Hunter's clisease. or we I1 m~tcopol\s;~crfixr~irIosi~, is an X-linked recessive
diqease ai~rlis Iew s c w r c than Hurlrr'q cvndrotne (nrpe 1 ) . Him ter's rli
nua'ir. call he rliflei.c.11~ iaterl from I-l~u-I~T-'s l r o m r in that it t'c:
tlure\ nn crrrnral opacitivs and syr r i i h r r 110 mrnlal rrlatrIntion ol-Irss
severe rctard;jtiur~lllan lhat li)ru>cl in I ll~rler's: howc-vrr, cle;~fricw s
prew-llf. .C:al~scrl a i hv deficiency of idwnorn~lfate sulfatase.
*
HUNTER'S DISEASE
ID/CC
i"\ 2-year-old white maIr i~ brought t o the nphthalmnlogiqt for a11 e ~ l l n a
t i o n eye clouding. or
HPI
The ch ilrl has a phvsical and mental disability ~'el'y 5irnifar to ihat of his
olclet. brother.
Short stature; very coarse, elongated facial features (MRs;okzrshr): bilateral c
orneal opacities; retinal rlcgcncration and p:~pilletleln;l; sarldlc nose rleirl
t-~rlitywsltolic nrurmur in sccol~cl right interro<ral space; enlarged heart, li
ver, and spleen; kyphoscoliosis will1 I tun har gibbus (acute aizgle kyphosis) :
q!iFf, in~nlobile, aiid con tract ed Iargr join ts.
Del-nrntan allfate arid llcparan sulfate in 111-ine; a-biduronidase deficiency i
n WBCs.
PE
Labs
Imaging
XR: rlolichocephaIv: increased dialneier. of sella lurcira; clcfcol~ nlntion of
ver-~ehr.al Imdics with rcoliosis and kyphosis.
Ftrcrcascd n~~~copnlvsaccliaride (MPS) dr-pusition in hemlrt, eye, con nrc~ivcti
wuc, CNS,cart i l a g ~ hear.(,and bunc. ,
Gross Pathology
hlPS inliltratic)~~ inin Micro Pl~athology Erllargecl heart; thickened crrdocarc
li~uin; 1i1n;i c ~ coronary artpries: menirrgu;~l f and ncuronal deposirs pmrlls
ciriq hydrocephalus; mexaclrrntnari v granules in ivn~phocvtes
ancE
I i~tioa?rs. 1
Treatment
Suppni-lire ophthaln~alogic, skeletal, and cardiovascular trcatrnenr . Co!-rsirl
~r tnc marrow transplant. l>r
.%Is0kilown ns gatgo~~lirrn, Hurrcr's c!ndrolne i q (he most cornI ~ I U I rnucr
~poly~arcliaridr~sir I (mE) . Jt is autosoma1 recessive I iiiid is c:~uscdby a d
eficiency o f a-iduro~dase. Death usuallv ocrllrs IIV (5 ro I0 p a r s or age, u
sually secot~dar.yr> cartliovasc~I las co~nplicai iolls.
Discussion
Atlas Link
T Z 1 - r n Z-BC-058
HURLER'S DISEASE
ID/CC
A 2.5-vcr-olrl male \<sitsa fertility clinic 3s part of an evaluation of inferti
lity ~ I A he is undcrgoil~g L with his wife.
His mcclical history discloses fruqricnt sinus infections (%INI'<IT'IS) inlcl ch
rtn~ic cougl~ i t l l spulum fc~i-rnaion w t (~ac~r\.c!-~lr.:c:r,~s~s) .
VS: ~~ot-m;d. apical impulse rc=It o n fifi 11 right intercostal PE: space; all
a u s r ~ ~ l r a t nroci reversed (IIEXTIZO~ rv .\~ior ; liwr on Icft i) s~de a
nd spleen on right ( S ~ ~ U S !NI~IXSU\).
HPI
PE
Labs
CRC/LTL~F: normal. Strllen an;~lvqis l ~ c ~ w r s immotile spermatozoa.
Imaging
Discussion
CXK: clexrr-ocardia.KP'B: qitus ir~vers~ls.
hi-t;igrnrr'.; svntlrome (it1so calletI imnioiile cilia svrldl-nme) is an autoso
mal-recessiverliqarder c hal-acterizcd bv lack of dynein ( ATt'xsr:) artns horn
the niicrotuhule~ axonrlncs in 1hr cilia of o f the s i n ~ ~ s and f ~ r o n c
h irunrlr-1-iny tlirm in~motile. cs . Sperm nl-P also i ! ~ m l ~ . t i(lC ~ I I
P~ c ~ f l alarelack of dvnein 1. The lack of e ~ l i n u c ~ ~ ~ c l c a r act
ion causcs frt-quun1 inFr.ctions. ing
m
rn --I u
Z
cn ri -
KARTAtENER"S SYNDROME
ID/CC
A 1C)-~;ear.-old m;tle visi 1s his familv phvsirian because 11c is embarrassed a
t having F q e breasts.
HPl
PE
H e also cr)rnplxin~ rrequen t Eleadachcs and impotence. or
Tall, eunl~choid 3 0 ~ 1habitus; tnild ine11ta1rctard;~tior~; 1 ~ testes small a
ricl firm; br-msl enlar-getnenr (c;rnr:r:onew~'czl: female
dis~l-ihutinr-i f hail: o
Labs
UA: increased urinary FSI-T; decreased 17-keioslrt-nid.
Imaging
Gross Pathology
XR: dcl;wcd phyqeal clnrttre; shnl-t fourth metacarpal.
Testic~llar atrophy Testicular fibrosis and hyahkation; lack or spermatogenesis;
LC-ydig'sin trr~titialcells scarce i~tir!have Crbanry n ~ o p l a s m i c rhat1
g.e: female sex chromatin bodies (RARR ~ ~ I F in )cells. R Y
Micro Pathology
Treatment
Discussion
Trstosterotie.
Also known 2% t~sticular dysgenesis. Kliucfclter's s!)ndrorne is thc most coi-li
mon cause of tnalr hypogonadism. Alteration ic rli~e c ~ p t - ~ s c n nf three
sex chrnrnosumcs (kiryotype t the c~
47,xxY).
KLINEFELTER'S SYNDROME
ID/CC
A 5-monthald rhild is hrc111~111~ l i r 'jltlrlii~l t r) t-irian hcrau~c .or gro
uqh rctardatinn anrl difficulty feeding.
Hi5
HPI
parentl; t1r)re illat thr chilrl h:~sIwen irritable artd "cliff" ( s ~ ~ , ~ s I
. - I ~ : I ~
PE
VS: normal. PE: patic11 1 underdeveloped Ibr-aye: reflexes hyperactive: pamvcr~c
bralrtluscles and hamstrings trnse (tur;t~~'n.): rna~cl-nal milk sucking reflex
weak ;in<] p u n c t ~ ~ a ~by lp~riorls er 01' rrg~r~-~iti~ti(~~~.
Labs
Rarir lab work rlithin tlorrn;~llimits. L,P: incrr;tred ] ) I . O I P i~1I I t C
SF.
Gross Pathology
Aun~lal tlrl while-ma ttri. cci-cl~ral. a cc~-c.helli~~; h,iral ganglia ,tr ~d d
emyclination.
Ritcop~iilicpel-ivascul;tr-tnultinrtdeaietl qltrhoid r ~ l l s (M~IC.KO0
Micro Pathology
Treatment
Discussion
Poor progno~is. wit11 dcxth
1t.;rli1ll!
occ~rrr r;2l>irlly. i11g
Xlw T:IIIP(I doboid le~lkodystrophy, b~l~l>e'xdisr;jsr i s an m~tosornal-recessi
ve, iimilii~l gul~ctic disol-der cl)a1-arr~1-i7ed a hv deficiency OF gal:ir~ n s
~ ~ l r e r a n ~p-galactosidase. irlp
KRABBE'S D I S E A S E
I D/CC
,4 2-vear-old male is Pl~-o~igli tlie p~rlialricianby Elis mother I to because o
f' repeated, self-mutilatingbiting of his finsen and lips: the par ien t ' [noth
er has a150 nnticecl abunclant, orange5 colored "sand" (uric acid crystals) i th
e chiEd's diapers. n The ~notherreport% that rome m o n t h s ago the child's ur
ine was rrd, hut sht. look 1 1 0 actiori at [ h e time.
HPI
PE
Poor head control, difficl~ltywalking, and cliEculry rn;~intaining an el-ert. se
ated position: choreoathetoid movements. spaslicitv, nl~cl hyperreflexia un neur
ologic exam.
Hyperrrricemia ( > I 0 mg/dT_,).UA: crys~alluria: microscopic he1nal11r.i:~(due
to renal calculi).
Labs
Imaging
Treatment
XR: irrugular amputation of several finger^.
Allnpurinol . Removal of primary teeth.
Lesch-Kyhan syndrome is all X-linked recessive n~etaholjc discasc rcsr~lting fro
m a deficiency nf an enzyme of purinc rnezabolisrn, HGPRT. If' lefi ~lntrearerl,
patients devclop fr~ll-blown gouty arthritis ar~cl urate nephropahy as well as
subcutanenus tnphaceous deposits. Compulsive, ur~cati trnlFd>le rl~structivc beh
avior is typical oF the disorder. Pr-ez~atal diagnosis is possible.
Discussion
*
LESCH-NYHAN SYNDROME
ID/CC
A Fi-tIa?.old ni;ilr prrsrn 1s rvitli seixl~res,l i f i i c ~I!. lrt-rrling. ;~i
irI t ~ ito111iring; hi.: n1olllr1- ~.rj>orluipeculiar, nlapk-Su~;;lr-lik~ ; odo
r on his diapew.
Hi5 niotllei h;td ;1n ~tt~reinai.lcahle liill-trrm \xginal drlivcry.
HPI
PE
\S [in rever. PE: Cull-tcriii n c o n a l r with irr-cg~llai-t.;pilatinns, ': r
ml~scnlar rigidit;v (.w\\T~(.ITT). ar~rl iuncled cr-ncnri~lm; ~ m d u s (113 f n
c>rrn:~l; e c ~ ~ l i a~>rlor urinc a n d swrc;tl; w h ~ n p lin rfiilrl'u I~ear
l .~lipj3ort h a n d ) is .;l~clclc.nlv ( will~draivni l l s l ~ p i n e t r q i
tion, pap tic~lt L~ils rmct wilh ~ l o ~ . l i l r ? lS I P ~ I S ~ C ) I ~ - ~
~ C I U C ~ ~ followrri tu P ~~II flrsiorl :~nrI :~rlrlz~c.tir,n a r m s ( . '
c ~ s ~I h(' rl: MCIKO E F I . ~ ~ ) . oS ( R
Hypoglycemia, rZ13C;s: rnrtxholic acirloqis. Marked elevation in blood and urine
levels o f the branched chain amino acids lrucinc. isolrlicirlc. i3Iloirrllt-ti
citlr. ,uid v3liiie :I< tvc.11 as c-irci.~arcrl IrvrIs ol'alaninr. t l l r ~ o n
i n c XHCI s c i - i ~ ~ c . .
Edetna nf h l a i t ~ wit11 gliosis and w l ~ i r c matter demyelination.
Restricting intake of branched-chain amino acids from diet; dialvsiq: rhiaminc.
suppIcrncntalion.
Labs
Gross Pathotogy
Treatment
Discussion
Maple c y - i ~ p I I . ~di <east, is an autosomal-recessive hrancli t-(II ~P ch
ain a - k e t o a r i r l ~ ~ r th;11 r r w I t \ II-L~ITI (ivy o ~ i d a t i v
e ix ilefec rlCral-hosvlatioi~or rhc I ~ r a n c h c d - c l ~ a a-ktkro;~cids.
iu 77114 clt-:r:~rhosvIation is ~isl~;lllv c n i i ~ p l i ~ I ~ e t ;I compleu
e n n rtie srslcm ac I)? I (a-ketodehydro+~nase) usirlg thinminc a coc-~~/\rr~t.
. \ i deficiency of' i l i i b cnryt?nc. sytern c;uisrs tir.ine lo I-r:~\-e111r
chari~c~t=ri\ric rnaple x\.rl[p orlcll- ; ~ n r I rails(=<Ch'S . ~ v m p t o m i
n ~ h c s firs1 fk~z. v ~ e 01' s lik. ~ k
MAPLE SYRUP U R I N E DISEASE
ID/CC
.-Z 3-vcar-rjld rvtlitc male is Iwouglit to llie pediatrician because of increaq
ing diffictrlty walking clue to spasticity.
HPI
I'he rIiilr1 harl Iwen rlpi~elopingnor-n~ally up mrdic:tl 1tist.nr-v is unrcmark
ahlc.
to
now. and his
PE
Difficulty climbing s t a h ; ataxia; widebased gait; rxterlsor planlar respotls
e an tl hyl>erreflesia.
Labs
LP: incrc;~srtlp~.otein (S (vs. rerphral palsy). Dccreasrd in :F peripheral ncl-
vr. conductian vulr,city. MR. brain: rlc.myclinativn.
C;enel-alizerl demyelination (CILIC to clcficicricv of arylsulfa~as~ A intrrfrri
ng with nr~rmi~l mctaholisrn c~fmyeIin lipids) with gliocis.
Imaging
Gross Pathology
Micro Pathology
Tuluidine I)IIIC staining shnwr hrowni6h (~IEI.\(;~IRC~~L.\TIC;) granllEe~in nli
godrnclrocyte~ and ~ l c ~ i i - cofrgloh~ts ~ ~s pallidus, tl~alarnu~. rler~sat
e alirl n~iclc~rs.
Poor pr rlgrlosis; 1x1 tienis llecnrne i~~valirl'; within a fcw ?cars and die 13
eIht-e p~lhcrty.
Treatment
Discussion
Mctachrornatic let~kodsstrophvis an autosomaf-recessive disortler nf cphingnlipi
d naetlhcll ~ F R I that ili clue to : deficiency in the I enzyme arylsulfatace
A will1 avcum~~lazion oTs111Catides the in renu al and peripheral nervnns system
as \vclF as in the kidnrvs.
METACHROMATIC LEUKODYSTROPHY
ID/CC
1 1-naonth-old Jewish m i ~ l c Ashkenazi desccnl presrllls of with globallv d c
l a v ~ d e v r l o p n ~ e ~ ~ t diminished visual acuity. d anrl
At1
HPI
His parents t'erl thal ~t-ie haby i q nor acql~iring new skills and t h a i exil
;tirlg otles are regressing. 'TIiey also feel that tllrir child cannor see nl- h
ear properly.
PE
Lymphadenopathy; hepatospIenomeply cherry-red spot on macula nrl rl~r-rdu~cnpy;
malnr~z~rishcrl inf-iln~ wit11 proruhera~l t at~dnrr~en; global developmental rl
eIap; Ilr~oacuGs.
CBC: mild
no^
Labs Micro Pathology
mochromic, nnrinorvtic anemia.
Bone marrow biopw reveals ~phingomuclinase deficie~-tc?l in
rl~ltnl-erl i n fibroblasts; chnractcristic "foarr~ k cellc" containing
sphingomvclin and cl~olrsterol.
Treatment
N o freatment available. Carries a poor prujinosis, ~ i l deatli h nccul-ring wi
thin a iPw wars of birth.
Discussion
Nirmann-Pick disease i q an autosomal-recessivedeficiency of sphingomyelinase wi
th acaunulation of sphingomyelin in the Iysosornes nf t3istiorvtes in thc brain.
hurl t- n~arr-ow qplee 11,ancl liver.
N I E M A N N - P I C K DISEASE
IDJCC
,4 2-?car-old fcmnle is referl-cd LO a pcdinrsic clinic Ihr evaluation ol'lcthar
sy, weakness, and persistent anemia Llrat has been ~rni.espousi\'eo ti-eatmcnt ~
4 t iron. folk acid. and vitamins L h C iuacl B,,.
SFic is thc third-born child o r a Ilcalthy whixc couplc; Iler mot her harl a n
11tlrvei1 trul pregnancy a n d a ~ ~ ~ t nrIrIivery. pic Both hrothcrs are healt
hy
HPI
PE
Low weight and height for agc: marked pallor; flaccidity ancl l c t h i ~ f i ys
leepiness. Lo fical ne~unlngic ; signs; It1 ngs clear: h ~ a r t sound uith slig
ht aortic ~ s t o l i c ejertinn tn1xrn11~- d l ~ to ( e aiicri~ia): nbdomcn sof
t; no masses; n o hcpatoinejialv: splccn har-rlv palpable: I I O lymphadcnopathy
.
CRI:: megalohlastic anemia: elevatrrl rrle;lTi corpl~sculnr vc~lume. LTh: illcre
ased orntic acid exct-eiion with rol-matinrl c)rorotic
Labs
a d d crystals.
Treatment
Adrn i nisti.ar inn ol'uridine and cvl idiile. Sternids,
Discussion
Orotic aciduria is an arrtosomal-recessive disorder of pyrimidine synthesis: il
is cawed b y a deficiency ctf- ~ heirlvrrke sy1erl.i r rx-otidylic p~;rapl~ospho
ryl,ue-orotldylic dec;lrhoxy!.lasc with rrslillilr~~ mtgalohl;~stic nnc~niadric
to irnpaircd h i t h r s i s of
?wear-old male prrsentq wirh progressive mental retardation, IOJCC 11 vomiting,
a n d hyperactivity with purposeless trln\,rnlerl ts.
HPI The child derclopc-d -normally for the firs1 2 lo ? months. He is I fairer t
han I ~ i fihlinqs and, unlike tiletn, has blue eves. He was r tmr-n outs id^ th
e Ynitcd Smtes and did not unclrrgo ally screening fur cnngenital disorders.
PE
Child i s blond with blue eyes. perspires heavily, is nlen tally renrded, anrl h
as peculiar "mousy" odor; hpertonia w i l I1 l~vpcimc~i\,e deep t ~ n c l n n rc
floses on npllrologic cxarn.
Labs
G ~ ~ t h r~c s t i c (I~acteriaI inhibition a s y m e d ~ o d ) prjsi~ive(due t
o increased blood phenylalanine levels) : increased ~ u - i nary pkenylpyru-uvic
and c~rtl1~hydr~.y1'pI1cn~~1;1ceric arid: normal concentration of teb.ahydrobio
pte&.
Imaging
Mk rlelayerl hone agc.
Treatment
Dier Corrnnlas low in phrnplalaninc. Avnirl aspartame. Tyrosine supplementation.
PKU is an autosomal-recessive disorder caused bv a deficiency of the enzyme phe
n~12dmhe hydroxylase. A nennatill screellting psuwuri for I he cletection of PKU
i~ in cffect throrlgliaut the Un i l e d Statcs.
'Discussion
PHENYLKETONURIA (PKV)
XD/CC
,4 7-war-old fumale presents wit11 anxiety, dizziness, sweating, and nausea foll
owing brief' episodes o l exercise.
These syrnptorns are relieved by eating at14 do [lot nccur if the patic11t is Cr
equen tlp I'cd small mcals.
HPI
Labs
Hypoglycemia fc>llowi~ i gI w i ~ r . fasting: alaninc faiIs t o increase bloocl
sugar: fr~lctoscor g1;vcernl administriltion rcstoves blood g11~cosc normal. LC
>
Liver biopsy for c-ri~y~ne assavs 1-weal9rleficirnry nf phosphoe n o l ~ y t - ~
~r carIrc)xpkinase, an enzyme of gluconur~gcnesi~; vat no excess glycogen stora
ge I-evcalcd .
Micro Pathology
Treatment
Discussion
Frrr~ue~lt small rrzealq L pt-event episode? o hvpoglvccmia. n C
Phosphounolpyi-uvakcarboxykinase (PEPCK) rleiiri~ncy prcvents py-uvnte from beir
lg cot~ver~ed phnsphoenolpvruvate. to T h i s drflciencv inzerreres; with glucon
eogencsis from 3-carbon precursors (e-g.,alanine) that cntcr tlrc gluconcogmrtic
pathwav at or helow the py-umtr Icvcl.
PHOSPHOENOLPYRUVATE CARBOXYKINASE DEFICIENCY
ID/CC
A 2-month-old child is broughl to rhe pediatrician becauw of failure to pin weig
ht, inc-t-easing weakness,insufficient strength ln hrcast feed, anrl lethargy.
HPI
I-Te is the secnnrl-horn son of a hcalthv white co~iple; his inothcr's pregnancy
ancl rlellvcrv wvrr uiievet~lft~l.
PZ Mild Fnwis; shalIr1w~ respir~tions: increase in size of tongue
(M,\(:KOC;LOSS!A) : moderatc hepatomrg:dy:significant generalized muscular flacc
idity.
Labs
CBC: normal. Lytcs: normal. GIucose, BUN, crear inine normal. ECG: short P - k w
ide QRS; left-& deviation.
CXR: extreme cardiomegaly and congestive heart faiiurc.
Imaging
Gross Pathology
of heart (up to five times Significant increa5e in size and normaI) ; t lesscr e
xtcn t, hcpaturnegal!: o
0 m
Z
Micro Pathology
Ex1ensit.e intracvtoplarmic and lysosomal depositior~of glycog~n o n myocardial
fihers as we11 as in striated muscle fibers, kidney, and liver.
Poor prognosis; associated with early death frnnl cardiopd-
m
w rn
Treatment
monary failure.
Discussion
Pompe's discasc is a type II glycogen storage disease (generali y e i l ) . TII~
S Fat:~l ~lisorcIerir caused ly an autosomd-recessive deficiency in the lysosoma
l enzyme (only glycogenolysis with Ivsnwmal involvement) a-l,4+1cosidase (ACID W
\LT~SE),wi lh ref~rlting accu~nulatian structurally normal ~lvcog-en rhe of it1
heart, muscle, htid~~ry, liver. anrl
POMPE'S DISEASE
ID/CC
A 40-~~;11--01d tisits h i s family tlnclnr becaltse of a chmnic, male recurrent
rash otl hi5 hands, face, and other mn-exporred areas; the patient's urine turn
s dark brown-black if left standing, and he hits noticed that recurrences coinci
dr with alcohoZ intake.
HPI
He reports haling used hexachlorohemene a a pesticide Tor s so~rie years (a fiir
rgicide show11to ht il~~ocialed porphvria wit11 cutanea hrda) .
Skin erythema with vesicles and bullae on sun-exposed areas; skin at these sites
is Ci-i-iahlcand shows presence of whitish plaqi ~ e ( "MTLFA") (dri e t o phr>
tosetlsit king effect of'uropors phyrin); skin of race also sl~ows hyjertrichosi
s ancl hyperpigmen tation.
RE
Labs
Miatson-Schwart~ test negative.Uh:markedlyelevated~ary uroporphyrin levels; slig
htly elevated urinary coproporphprin levels. Fecal isr)coproporpl~yrin normal; e
levated transfenin, serum and hepatic iron; elevated serum transaminascs.
I,ivel- shnws sicI~~-osis, hllllilr, fihi-mis, and inflammatory changes.
Gross Pathology
Micro Pathology
Skin biopsy demonstrates iron deposils, intense porphyrin flllorpwence, a~lrl lo
ng, thin cytoplasmic inclusions.
Treatment
Repeated phlehntoinies; avoidance af sunlight, alcohol, iron, a n d estrogens. L
ow dnse antimalarials.
Porphvria cutanea tal-da (PCT), in contraqt t other hepatic o porphy-kas, is mor
e common amon< men t h a n women. PCT i s causccl by partial l s of activity of
hepatic uroporphyrinogen os decarboxylase; lesions are caused l y cwerprod tlcti
nn and excretion of uroporphyrin.
Piscussion
f l PORPHYRIA CUTANEA TARDA
IO/CC
/\ 5-1 ear.c~ld xirl i q refel-I-udto a hrrnatologi<t for an e~.ala~a~ion cr I r
hron ic anemia that has been unresponsive to nutritional .
sl~pplementation.
HPI
Rot11 parent7 are cli~lically nnrmal anrl art. Fmst cousins who are Amish. Thc p
aticnt has n o history of passagc of darkcolo~-erl z~riilr r rcciirrt-nt in rcct
iorzs. o Low ~ v c i g h and hcight fcjr age: pallur: mild jaundice: spleen l l~
nrrly p~lpable: livrr ria! rnlarged.
PE
Labs
I :BC/PRS: anemia; markedly increased reticdocye count: peripheral hlood rrveals
macrt,-o~~locutosic tli a few wi eci~inoryws; sickle cclls 01- hpl~c~-ocvteq e
r i Hyperno s~ . I~ilir-ubinurnia (pl-iinarily lincor!jl~gatrd).UA: urinary I1r
nio~ideri pl-esent . Reduced serum haptogIollin; diminished 11 activity o f p y
a t e kinase in REGS on sprctropliotome~ry.
Treatment
Discussion
Exchange transfusions, Splenect 0 1 1 1 ~ .
I -
!'vr~~vate kinase d~ficieilcy inherited as arl autosomal-recessive is trait a n
d usllally pl-odl~ces mild sv-mptoms (hernol?)ticanemia): 2,3Sliphosphoglycerate
accumulates. sli i rting t h e hetnqlohinnhvgen dissnriation curve to the right
( d ~ l e ruducrd affinity to
*
PYRUVATE K I N A S E DEFICIENCY
ID/CC
A 6-month-oldmale i q brol~ghtto a pediatriciarl for evalllatin~l ~ f c listless
ness,lethargy and fixed gaze.
His parents are Ashkenazi Jews.
HPI
PE
Excessive extensor startle response ta noise ( HWERACT ~ 1 :~c1liId 7 ) is sleep
): and hyporonic ~iil.ith poor head control a~irl fixed we: a appeilrl; 1 Iimr t
l-ansl~zccnt 0 skin; cherry-red mcular spot round ort f~tndltscopic exam.
Diffuse gliosi5; celrhral and macrllar degeneration: u p to 50%) inci-rase i n b
rain ~ v ~ i g l(due ro deposil inn of spl~i~~golipid) t l . Ne~lrnnalsrvclling
w tti cytoplasmic deposits o f gangliosides i (LEHKA RODIPS) .
Prmr prognosis: patients usuallv die of pneumonia hefnre reach-
Gross Pathology
Micro Pathology
Treatment
ing the age of 3.
Iliscussion
Ta\-Sachs disease i q a n autoqornal-recessive rlisorrler of sphi11the absence o
f the enzyme golipid nletaholism characterized hexosaminidase A, producirlg exce
ssive storagt. of gangliosidc GM-2 in I!~sosorne~ restricted to the cells of the
& e n dnervous system. Ganglioside GM-2 is a glycoyd~ingolipid with sphingosine
, a long-chain hasic molecule, as its b a c k h o ~ ~ e along wit11 an attached
sugar and a terminal N-acu~lglt~cnsamine. Prcnatal diagnosis can be maclu at tfi
r 141h week of prcgn;iilcy.
.?!
TAY-SACHS DISEASE
ID/CC
\ i
7-yar-olrl !male i s I>rnnghcto a pediatrician for evaluation of cpisr,cles of C
a~gue. restlessness. anxiem. nnasca, lightheadedness, vomjti~q, and sweating.
The sunlptoms appear wtlrrl lie doe4 nor eat rreqlient meals and sulrrsiclu whil
e h e is eating. He also has a historv of b r u i s e easily.
HPI
PE
Patient has 'bdoll-Face" Facies; weight low for age; tend011 xanihamas: pun-puri
c patches over skin; marked hepatomeply.
Labs
Lactic ;~cidosi>; 11?7~c.r~lipirlrtnia; marked increase in serum uric acid (pati
cn t tnay exhihi t gout symptoms); marked hypoglyce&, prolonged bleeding time; l
ack of rise in serum glucose following SC epinephrine o r I' glucagon hut striki
ng i~irrease lartatc; C in nortnal i~rinar-y catecho1arninc.s.
US: 1iep;atsmegaly: kidney a l ~ n enlarged bitaterally.
Lwet and kidneys enlarged (1.5. lyle 1r1 glvcclgen frorage disease. nr C:or.i's
disease, it1 whirh there i5 no renal involvement).
G )
Imaging Gross Pathology
rn
z m
2
vl n
Micro Pathology
Ilepatocvtcs ct~ntaining variable-sized glycogen-lipid droplets on l i t e r bio
p~v; nuclrar p l y ~ o g e t ~ owpn; large glycogrn dtpositv ~is in kiclnry; ske
letal ancl cardiac muscle riot invc)lved (vs, type V dvctlgun storage disease, o
r McArtlle'? rliwase, in ~vhich skclutal musclu is invulved) .
Frequent meals to prevent h~pnglvccmia,
Treatment
Discussion
\'nn Gierke's diwaw i q an al~tosornxl-recesFive glycogen storage d s i raqe ( y
e 1) t-eq111 ng fi-nnl a deficiency of glums~Gphosphatase ti and ; ~ c c l ~ ~
n u l : r,f t slrt~ctl~rallv ~ iu~~ nr~rmalglvcoget~ the liver it1
and kirincys.
VON GIERKE'S DISEASE
lD/CC:
A 10-year-old girl is hrought her parents 2 0 a dcrnialologist hecaur~ ora recen
t change in color and increase in size of a warty lesion on her face.
She has been sl~rering from excessive sensitivity to sunlight and thuq does her
hest to avoid thc sun as 1nuc11 possihlr=. as
A h ~ ~ n d afreckles on all ain-exposed areas; telangiectases: ut areas of redn
ess ( E R ~ E M A and hypnpigmen tation; hyperker) atosis on facc and dor-sum of
hands: hard. nodl~lar lesion on rig111cheek; no regional lympllader~oparf-~y.
HPI
PE
Labs
Gross Pathology
Basic lab work normal.
Generalized h y ~ e r p i g m r tatioi~ t h desquarnativ~ n \%i spots on sun-exp
osed areas.
Riopgy nf cheek lesiun reveals hyxrkeratosis wit11 melanin deposition; squarnow
cell carcinoma. Avoidance of sumlight , 131-orec! against s~mlighl. ion Surgical
r.emo\.al of cancer.
Micro Pathology
Treatment
Discussien
Xeroderma pigmen tosu~n an autosomal-recessiverlisordeiis [hat ic: usuallv manif
ested in chilrlhnod. I t is character-i?ed I)? exces.;ive qensitiviq to 111 triv
iolet light due= to impaired endonuclease excision repair mechanism of ultraviol
et lighidamaged DNA L r IigIi t causes (cross linking of pyrimidine rrsidues) in
dermal fibrohlastr;.There is a marked ~ c r ~ d e n cICIvdevelop skin cancer (s
quarnuu~ cell and basal cull r al-cinc>~na).
ZD/CC
A 40-year-old woman prrscnts with weakness, easy fatigability,
nauqea. and diarrhea.
HPI
She has had a long and severe course of rhel~matoid arthritis for wl~iclxshe has
heen taking m e t h o ~ x a t e Folic acid antagonist) (a
VS: nortnal. PE: pallor: tnilcl tangle inflamnlation (ctussms); f~~niluscnpic ex
am normal; chest s o ~ ~ nwithin normal limits; ds abdomen shows no hcpatospleno
megalv; no l!mphadenupatl~v; no neurologic signs {vs. vitamin B,, dficiency lneg
aloblastic an em ia) .
PE
Labs
CRC: hypersegmented PMNs (> 5 to 7 lnhes) ; mepploblastic ' RBCs (mean corpuscul
ar volurnc > 100); vitamin R,, lwel normal; falate lc\rl in RBCs low {as, titami
n HI., deficiency mrgalohlastic anemia).
Folic acid supplemmmtiion
Treatment
Discussion
Folic acid is found mainly i n g r t r n lraves and is impnrtant Tor the synthes
is of DNA and RNA. Ir a190 acts a a roenqme for 5 l -carbon transfer a n d i inv
olved in methylation reactions. s Dcfjciency is assouiatt-d with alcoholi.;m, pr
.epanry (VIE(;%LODLZSTIC. .WEML\ OF IPFXGNWCI'). clie~arv deficiencieq, and drug
s ~ t i c l las TMP-SMX, tn~rho~r-exate, phenytoin, and p r o p ~ a n i l .
= f"
3 0
7
&
ANEMIA-FOLATE
DEFICIENCY
ID/CC
A I -year-old infant presents at a clinic tvi tIl lassitude, poor muscle tone, a
nd cl~layedmotor dcvelopmenr.
HPI
The mntl-ter is a known I drug user anrl ha5 two alder children V who arc in the
custody of the state social ser~ices agency.
PE
VS: tachvcardia; tachypnea. PE: pallor: partial alopecia; ulceration o f ski11 a
t corners of mouth (crr~nosr5): smooth tongue; nails hreak easily and are spoon
shaped {KOILONYCHI~) .
Labs
1n'POCHROMIC ANEMLR);
PBS: abnormally small and pale RBCs ( M I C : R O ~ ! C . mcs 01' different sitr
s (ANI~OCYTOSIS) and diFferrnt Jlapcs (IJOIKI~.O~XTOSIS). Decreaqed serum iran;
increased total iron-hindixlg capacity and reduced perren rage s a l u r ~ t i o
nartificial increased m s f e r r i n (due to attempt to ; transport as much ir
on as possible) ; low serum ferritin (due to insufficient iron slored as fcrriti
n).
Gross Pathology
A~rophic glossitis.
Micro Pathology
Erythroid hyezplasia with decreased bone marrow imn stores rhrorlic diuease, whi
ch is characterized hv increased irori stores).
on Prussian blue staining (w. aile~nia of
Treatment
Discussion
Con trnl cause of iron drficiency; suppletnental imn.
Trot>-deficiency anemia i the mosl common came s f b n i c s blood loss, ~ ~ s t
~ a l l y gastrointestinal or gynecologic: i1 is secondary tu a deficiency o ir
on required for normal hemoglobin f synthesis. Direren tiate li-om anemia of chr
onic disease, in which ferritin i s high and transfert-in i q low.
ANEMIA-IRON
DEFICIENCY
ID/CC
'4 5fhcarnId black remale complains of weakness, dizziness,
anorexia, nausca. :md occasional vomiting over the pasl 3 months.
HPl
Shc has also uspcricnccd shortness of breath (titie to dimit)islied oxvgen-carry
ing capacity) a well a numbness and tinghq q q in the extremities (Rue to megalo
blaqtic peripheral neuritis).
PE
Sliglitlv ictrric eyes: hepato~plei~ornegalv: ~ a o t hheel!-red sn , tongue (GL
OSSI I 1s) ; loss of balance, vibratory, and position sense in hot11 lorr7r.r c-
xtrrrnitic*; (due to poslerior and laieral column involvemu~~t; folic acid defic
iency). vs.
Labs
CHC: rnacrocytic, Irypnchl-nmic anrmia (MCV > 100) ; leukopenia (4,000)with hype
rsegmented neutrophils; thro~nbncytopenix. I-l\perhilir~ibinenlia (2.5 m g J d L
normat 0.1 to 1.0 ing/ctI,); arhlorhydria (no hvdrochloric acid in gautric j ~
~ i c rpmitive ): Schilling test; low blood vitamin B12 IweIs: RBC rolate normal
.
Dorm1 and lateral spinal columns are small, with itxunal degen-
Gross Pathology
cr;ition; flat, atruphic mucosa; low of'rugal folds i t ) stomach; increasrd rer
l marrow in hone.
Micro Pathology
Meploblastic and hypercellular bone marrow with erythroid hvperplasia: accum~ila
tionof hemt>sideriil it) Kt~pfrer cells:
chronic atrophic ~ a s t r i t i s .
I
m
3
3
z
Treatment Discussion
Paren!el-al v i f i l t ~ ~ i Rl ,?. t
K
Pernicious anemia is megalohlastic anemia caused by malah~rjrprion vi iamitl R ,
? becatlre nf lack o f intrinsic factor in OF o kr;~.stric juice (intrinsic fac
tol-.?iccrctedhv p a r i c ~ cclls, is indiql pensable for vi tarnin B l oabsorp
tion). Antibodies against ptri.ic parietal cells are al~nnst inwl-ialjly prmenr
in the aclul t form of
0
z r )
r 0
m o
the disease. Atlas Links
I1 1-1 '-1 H-BC-077 .1
ETTT7 IMI-043
7
ANEMIA-VITAMIN
B I Z DEFICIENCY
ID/CC
A 31-vear-old black male who works as a. Peace Corps v o l ~ ~ i l t e c r i11 G
hana visits his inudical officer complaining of cxtrcinc weakness and fatigue;he
also complains of a yellowing of his skin ancl slight fever.
HPI
Nc i m s prescri hed primaquine for radical
(clue c Plr~ o r.modi?l?~ uiuy~kr) .
tr-eatment OF malaria
PE
VS: tachvcardia (FIR (35). n ~ i l d PE: jaundice; cirru~noral ancl nail
l>ih(I pallor; n o ht.patosplcnc~mr~~ly:~ ~ i n t l cof. PE normal. rrn r
Labs
Elevated indirect bilirubin. CBC/PRS:low hemoglobin and hernatocrit (9.3/33) wit
h reticulocvtosi~(b11~:~01.\5lri) ; spheroqtes in pcriphural blood smear; Heinz
bodies (precipitated he~noglobi in WCs: low blood levels of G6PD (diagxiustic).
11) Llh: hemoglnbin uria.
M'ithdmal of ol'rending drrig.
Treatment
Discussion
Gl~~cose-6-phosphate dehvrh-ogen:tse (GRPD) deficirricy i q at1 X-linked recewiv
e rlisol-def-wen in alloltt l .?'lo01' Amel-ican 1~lack malcs. MFithinfections o
r exposure tr, certain drugs ( e . g . , qillFi I I T I I ~ ar~tirnalarials, F,
nitrofi~r.antoin), patients deficient in GtiPIl present with a hemolytic anemia
due to increased RBC sensitivity to oxidant damage. GfIPU is lie rate-limiting e
n m e in rhc I i M P shunt that provicles NADPIH.
GLUCOSE-6-PHOSPHATE DEHYOROGENASE DEFICIENCY
IDJCC
A 7-year-nlcl inale is 131-oughtto the emergency room I~ecause of rvrakness and
the spontaneous appearance or painft~l swelling of 1~0th kr~ec-joinls (dur to hc
m;~rth~-osis)well ar hliick. carry as stools (GI 1,lrcding).
T h c child has ;I historv of prolonged bleeding following mhor injuries. Hi\ t
i i ; ~ ~ e r - r unclr d i d o f ;i "lde~cIing ~ ~ F O T ~ C I - . " ~i~l C l'a
llor; swollen. r r - v r h e ~ n a t o ~ ~ s , kneejoinrs with blood actender c
t ~ ~ n ~ ~ lit) t ion a svi~cwial capsl~le I F ~ ~ . ~ K T H K:O S I ~ ) (I num
erous brnises seen at areas or minimal rcpcated rr;lltnia. Blccrlin~ time and PT
normal; prolonged P I T ; reduced lercIs o f factor 1711 url irnmri~ioass;rv:.i
vno~.ial fluid hemorrhagic.
HPI
PE
Labs
Imaging
Sk hilateral
knee eff~lsions.
Micro Pathotogy
Svnovium matrshow hyperplasi;~ wit11 heninri tIerin in syynouial
rnacr O P ~ > ~ ~ X P S .
Treatment
Nonphal-m:lcologic' therapy inrolvcs patient eclucarion , avoidance of contact s
ports, avoidance o f aspirin i m r l o ~ h ~ t NSMDs (dur 10 a n I ipl;~ieleragg
re+atitlg el'fert). orthopedic evaluation ancl ph-ical thcraps and hepatitis vac
cination. Factor VIrI suppIernentation i u effective in con ~l-c>llinq s1)ontane
nuq and rr.;l~uii;a~ic hem( jrrhxqe. Demopressin nmp Inc used prophylacticall? i
n pticnt< with milcl hemophilia, prior to rni~~nisul-gical ~~l-ot-~~lttre~. Amin
ocaprnic acid inn\. he w e d to stop hleccling that is r~nrcsponsive L~ctor to \
?IT or c ~ ~ s I ~ o ~ I - o s = , ~ I ~ . H c ~ r ~ o p l ~ i-4 ; q an X-linked
recessive disorder that is nratlilii i Ve'rr~rd F>lt.~ciing tn anrl i h due l o
x deficiency in coa*gwp;Lllation
factor Vm.
I
2 5
? 0
r l
G,
2 zr
D
Discussion
r o
t 0 ,
<
Atlas Link
Lrl_U PED-018
H E M O P H I L I A , TYPE A
Itl/Ct
A 9-month-old infant is brought to the pediatrician because of jaundice, letharv
, and easy fatigability.
HPI
The parents of the child arc immigrants of northern European
origin.
PE
Labs
Pallor; mild ,jaundice;palpable splenomegaly.
CBCJPBS: rnicrocytic anemia; small, rounded, dark M C s Iackit~g centr.aI pallor
; negative Coornhs' tcsl. Elevated indirect bilirubin; increawd reticufocyres; i
ncreased mean corpuscular hemoglobin count (> 95); decreased MCT': abnormal KIIC
osmotic lysis test. Fnlic acid. Splenectorny.
Hereditary spherovosis is a congenital, autosomal-dominant discrrder characteri~
ed hemolytic anemia with spherical RECs hy ancI spl~nornegaly. i causer! by a de
fect in RBC membrane It s spectrin with loss or the t~orrnafbicor~cavitv and a h
igher rate o f splenic seq~~estration hemofvsis. If left untreated, i t may give
and r i s r to pigment gallstones and cholecystiris.
Treatment Discussion
HEREDITARY SPHEROCYTOSIS
ID/CC
X newborn rnale in the normat n t l r s ~ r v nclted to he cyanotic; is the pedi
atrician i.; called even though the c11iId docs not wem to be in respiraiorv cli
srress.
HBI
That mornirlg h e had undergone circumcision (a henzocaine ointment was used).
Cyanotic; l t ~ n g s clear and rvell ventilated: hcart sounds rhytlim ir; nu mt
irmurs heard; no cardiopul monarv problems evident.
PE
Labs
CBC/Ll;les: normal. PPatelet5, LlTs, BUN, and cr-patinine normal. U G s : PO, no
rmal. Methernoglahin level 18% total hemoglobin.
CXR: normal
Oxvgen for acute symptoms. Methylene blue ( itlcr-eases activi-~ity nC methcmogl
ohin rcductnse) . Methetnoglohin is an oxidizcd (FLIWC) forrn of hemoglobin dlat
cannot function prnperlv as a carrier of nn7gen.The dirnitlishcd ox~gen-cart-vi
ng capariw that results pmdrices heaciache, light headedness, atlcl dyspnea. Dru
gs such as dapsone and I ~ e ~ ~ z o c a as well as dves such as anilines oxidiz
e hemoglobin ine to its ferric form, 3s do deficiencies of lVkDH methemodohin re
ductase. In neonates there is a transient deficierlcy of [hi5 enntne, and tIhF i
s more susceptihlc tiran HhX oxidation.
Imaging
Treatment
Discussion
x m
3
3
Y
<
0
Z
l-7
r 0 G1
E <
G I
7
METHEMOGLOBINEMIA
T'D/CC
A 19-yenr-olcl male cnrtles 10see tile trtirse at the mllegc heal111 rlep;~l-(me
ncomplaitli z ~ g ahdominaI and lumbar pain, which I of charactet-islirally occ1
1i-s when be takes his multivitamin pills two time5 ; week (il-on. inrectionq. a
n d v:~rrit~ntintl precil~itating l are f i c ~ o r q;)h e has also noticed dar
k brown urine the morning aftrr tie has the p i l i 1 1 (dur t o h ~ r n o l v s
i ~ ) .
HPI
He ha.; j l i s t left his p s l w ~ t s go t o c o l l e ~ u n d is excited abo
ut to a his 11~rvfn1111c1 fk~r~edarn; likes 10drink exce~sive he arnoltnw of' hc
cr.
Marker1 pallor; I ~ l n g fields clear lo auscultncior~; heart wuntla ~~orin;~l:
ahdnme 11 sort a n d nonzenrfer will1 nr) masqes or peritoneal signs; no focal
ncuroIogic signs.
PE
Labs
CRC: nr>rmur.ytic, nonllrachromic hemolpic anemia with reticulocytosis. Hemoglob
inemia and hemoglobinuria; stacrose hemolysis test positive; acidified serum tes
t positive (HAM'S TEST); decreaqed haptoglobin; efevated LDH; decreaqed leukocyt
e alkdine phosphatase.
I - I e r n o s i c l e r i of liver, rpleet~. and kirlr~ey. Ster.nids, tmnsft~q
ior~ ~ salincwa~11tcI c f RBCs during crises. Ordl i r o n q~~~>plrtt~entatiotil
>e 1 l f ~ l i 1h ~ shoiild h~ I I S P C ~ tn;y 1 ~ t cautio~~sly. it may precip
itate transicnr hcrnoly~is. ;IS Similarly, heparill rnav a r c e l e r n ~ e Iie
mdysis. hli t irs z~se ~ h r o r n l m ~ i c in ro~l~plicacioiis appeal-q w a r
r a n ~ e d . Paros~smal alucturn;~l hemuglohinuri:~s a n acquired drfcct of i t
he red hloor! cell 1nenlh1~111~, makis~g erytllrocy~es l n u ~ u a l l v ~ seils
i~fvr Frrurn complement (there is also increased binding to of'C3h).TI i s cll;t
racteri7ccl by rpisoclcs of hrrnolysi.; with hemi* gIohinuria that occur d ~ ~ r
i ~ l e e p ~ c a u of carbon rlioside ng h s~ r-vtr~~tior~ (which lowrrs the p
H, thus e n h a ~ i c i n g o r n p l e r n e ~ ~ t c slcrivity: lirst ~r>icler!
r rn e i t 1 rhe tnornit~g red-brown). Patient5 ~ i is arc also prcdisposrd to
devrloping venous tllrt~mhosr,r.
Gross Pathology
Treatment
Discussion
PAROXYSMAL NOCTURNAL H E M O G L O B I N U R I A
IO/CC
1 2
33-1 ear-nlcl ~r ornnn pr-ewnts to a cl i r ~ i c with marked wealmess (dlie tn
Sl)pt>kaEemia).
HPI
Two years ago. she u ~ ~ d c r ~ va n t e ~rretcrolithotomy reno~rreteral for st
ones.
PE
VS: rachvpnca. PE: generalized muscle weakness: h e a r t s o ~ ~ n d s
rz~ith fc-1.v skipped l ~ e a l s ;I (I~ypokalemia givt=s rise to Fevert= T rhv~
lin~i:ts); ;I clirriinist~~.d in~es~iriaI l h a l ~ i no peritonea! s i p s . pe
~:
Labs
I.vte~: inrl-eawd urinary potassium cxcre~iuu(due tn insufficient Elyclroq~ninn
availahfc. ~ vth pr>tassium ux.cch;~r~~ed qodium) , i for TPSUI tirig in marked
hypokalemia (2.5 mEq J L ). I V ~ G S : rlecrrased HCO., (clue 10 r a i l t ~ l
to maintain n r ~ r n ~ gradicrlt o f hydrogen -~ al ions in dista t renal r u b
~ ~ l ewi\it!i HCC>.,loss) ; hyperchlorernic s, metabolic acidosis ( nonnal ani
on SIP). Normal w r u m r a l c i ~ z t ~ ~ :
high alkalinc pho.;phatase. LJ,4: urine alkaline: I~yprt-ralciuria.
Imaging
KLJB: r-adiopaque left kkirlner slones: meclullarv renal calcificarioi~.
Neph~.oralcino~i%. Bical-bonutc: pota.isi~rrrt3 r d vi tanli 11D
hlctaholic acidosis is C ~ I U S P ~ renal t 11l1111;3r T I P ~ Y C in trans13y
~J port. Type I (rlistal) involv~s wlective delicicncv of tubul;lr H" secret i r
~ n (pr-nrhireq tvpical In-perc hlorc tnic-hppc~kd. lcmic acirloS ~ F with n o
r m a l anion pap). T ~ p e (proximal) involves the JI ir1;ibility Z ~ rei~llvcl
rhHCO, (also h y o k a l e t ~ ~ i c ) . 111 en tail5 I Type the iilat3ilih. LO
prorluce NI I, due to pcsristcntly luw GFR vt,lurnes (normokalc~nic). Type IV is
tliie t o pi-irr1at.y or d r ~ ~ g inducrd h v p o ; ~ l d o ~ t e r o n i ~ ~
n (hypcrli;llemi~. 1
Gross Pathology
Treatment Discussion
z
rn
v
i3
J
0
1 C
; l E
o r I 3 <
0
"
RENAL TUBULAR ACIDOSIS
IQ/CC
12 38-vear-olcl rlect riciar~i s rushed to the etnergcrlcv room after recri~ing
accidental high-voltage electric shock ~ . h i l e an fixing a power- line.
On aclmiqsinn, a Folev ca t h e w is inwrted, vielding reddishbrown urine (due t
o myogEahin). VS: rachvcardia: RP n o m a I . PE: confusion; disnrientation; pat
ient complains or muscle pain in right arm, leg, and huttack; h a r d srrerely s
wollcn and has an obliqr~e-shapedburn; "outlct ~vcl~~nd" locaied in righ c glute
al region and ankle.
Markedly increased serum BUN and matinine (due to acute tr~l~ular r~ecrosis) urc
a normaI. Lytes: hyperkalemia. ; Hyperpf~osphatemia; hypcrnricem ia; hvpocalcenr
ia (clue t o caIcium binding to necrotic mtlwle): increased serum CK (due tu m ~
~ s c destruction) ; myodobinuria. lu
Urine alkalini7a1 ion (with T bicarbonate) ; tignro~ic V rehydration {to prevent
pigment drposi tion and acute rt~hular necrosis); mnnnirol: prcvent further mur
c IP damagr from compartment s\mdr-omcs (cialuate need for fasciotamv). Correct
electrolvte abnormalities. Hernodialysis may bc required in severe cases. b
HPI
PE
Labs
Treatment
Discussion
Mvoglohinuria and reduced renal perfr~siorlfrom volume depletion may cause acr~t
e t~lbular damage. Othcr causes of rhab dnmvolysis (destructionof striated muqcl
e) includr crush injuries. hcrr~in ovel-close. prolo~~ged u~~cnnscieusness one i
n
ID/CC
.4 -l-y~ar-r>lcl male is I~rr,ugli 10 the perliatric clinic. beca~ise t of
eay Fatigability and difficultywalking o T a few months' duration.
HPE
Thc child'% mo~lic-r rmlicerl that hi5 c a l v ~ c has have itlcreased in
size (pseudohypertrophy).
PE
Child well developrd but show proximal muscle weakness in ~ l ~ ~ t iand ~ r gir
dle: diflirultv standing and walking; l d pelvic "rlimlls 11p on hinlself" to ri
se from sitting to sti1ntIitlg (Go~darrs' SIGV).
Labs
C & LDR. and glucose phosphoisomera~e l elevated: absent dystmphin expr-esqinn o
r1 i t ~ ~ r t ~ ~ ~ n o . ~ t a i n biopsy. or mr~scIe
Replac~menr normal musclc protein wit11 fihrorattv t imlle. of giving riw to p~r
udohpcrtsophv.
E>eg-f.tlrrationand atrophy o muscle fibcrs wit11 ringed fihel-s C susrotmrling
normal tissuc.
Gross Pathology
Micro Pathology
Treatment
Prngrrosis i s poor; with diwl>ilih.occurring within a few year's and deal h by
the early 20s. Treatment is supportive. Rcfer for genetic counsrIing.
nucherlne'q muscular dpqtrophv is an X - l i e d recessive cl isorder characteri
zed bv a deficiency ill rnuqcle d y s i m p h . a suhsarcolemmal cytoskcletal pr
atein that stahiFizes thc sarcolcmma d ~ ~ r iconrt-action and reiaxation. Its c
ourse is relentlessly psot~g gressivr. ending in de:zth from cardiac anrl reqpir
atnl-vmuscle involvement.
Discussion
Atlas Links
[_lLLL PG-BC-085, PM-BC-085
FFTlT FED-049
DUC'HENNErS MUSCULAR DYSTROPHY
r 40-v-vear-nld male i s hr-ought l o r h r ER 1.1-om 'bar because of \ a cod~ls
ion aftel-falling from n I~ar stool.
TIlc paticlit'< Friends say that his dirt consists mainh c ) f alcnholic drinks.
They also stale that h e has lolrl dctailed and believable stories about his pa
st adventures that llnve subsequently been Found to be unbue (c:nh7;.4~rrr_~~rt,
u). short- and long-term His memory is severely impaired.
Ataxia; ocl~lomot~r abnormalities. i~lcjtldi~lg nystagrnlls and ophthalmopiegia.
CR: r~iacrncyr arieinia ( m o s ~ :C: ic lilcely heconrlary tn c i r ~ ~ c v ) .
thiamine (B, ) levels. Low Gross Pathology
Micro Pathology
Treatment
lihate defi-
Bilateral atrophy o f mammillary bodies and thakan~zls.
Irntnedi;~te thiamine ( h , ) administration parentcrallv: thc w o n e r treatme
nt i \ aclmini5tcrcd. t11c Ic-5s pcrmanenl 111cqeqtlehe. ,4lcoholics should ~tls
i,recrive oral o r IV folnte as ~vcll a as sn~~lriri tzimir~.Monitor rarrfully f
or delirinm trernens secondary rn alcohoI withclrawal.
Discussion
reversible with thiamine treament. A dclay in trram~rn nlav c a t l s ~ t progre
5sion to KorsakofYs pychosis with purrnancn t dementia. Patients 1 ar eFv rctrn-
11 to nnrirlal. Palients alf(> ofl PI] have wet beriberi ( h igh-oultput carrlia
c failurc) , dry beriberi (pcriphrml net iropalhy with irrl~~air.tr~eiltrlislal
rnutor- arid se11~01.y l n r t i ~ n )and nf fi , cer-rl~r~al hel-il>er.i(tnrllo
r a n d rngni tive i trlpair~ncn . Werniche's t) encephalopathy consists of a tr
iacl uf confuqion, ataxia, and ophthalmoplegia. Kol-sakorrs psvchnsis is charact
urizcd Ily retrogade/antesograde amnesia and confabulation.
WernirLc's encephalopathv is Cor t l ~ c most part
Atlas Link
WERNICKE-KORSAKOFF SYNDROME
ID/CC
Afiel- a r ~ u t i t prlvic- e x a i i ~; "Lveilr-c>1~1female i s rel'erred by l
~ ,l he1 L ~ n ~ ipliv~iciarlto an cndocl-inoltrgist Sol- an rvalr~ationof lv "l
ack of a palpable cervix."
HPI
The pasient s t a t r s that she has never had a menstrual period.
Bitatera1 breast tissue present: absence OF pnbic and axillary hair: vagina ends
in blind pouch: clir nrntiiegalu: al-nall atl-ophir testis found 0 1 1 sight i
n g ~ ~ i n canal. a!
increased
PE
Labs
W anrl testosterone. K a ~ + ~ o46.m t~c:
i1i1i1 o~,ari es
Imaging
Treatment
LIS: Iitcrw
;II>wII~.
Treat as woman, reqect cryplot-chid 1c5tis ancl luok Tor t l ~ e irifnabdominal
o n r (clue t o h i ~ 1 1 risk o f m;~liznarlcv).
Discussion
iiis~risiivii y r;j;t~cl~-nn~e, t tcqticular Cc~ni;Zlsc) kr1ur4n ;i t ~ d r o g
t ~ ~ i i~i;.atic>n rh:~~rterizt=d a genot~picarlv is hv 1n;llr- inrli~icl~~;ll
(iclztt~crrr?~ -16,;Yk) who p~-rscrlss with ;I f I I I ~ L I 1 1 0 ~ 1 ~ ~ halli
tlts with I~rt.;iu~ dcvelopmer~tanfl c l - y p t n r c l i i ~ l i ~i t~ i c clu
e 10 a Y chromo~; some gent=rlrfrct I llat C:+IISPS 1 ~ I P testosterone recepto
r protein
*
P R I M A R Y AMENORRHEA-TESTICULAR
FEMINIZATION
ID/CC
A "year-old black remale visits her gvnecolopst because she thinks she might bc
pregnant becanse of tack of menses far the past 4 months.
She is a pentathlon athlete who is training to compcte in her ~ I O ~ I Islale's
tournament next fal' Et. She is scxuallv active, uses P a tile "rhvthtn method"
for birth control, a d has never missed a menstrual period.
No hrcast enIargement: no softening of ccrvix; no bluish discoloration uf cervix
(hot11 presumptive signs of pregnancy); no al~dorninal pelvic masses or palpahi
e uterus; no llirsutism or or virilizatinn.
Serum and urinary P-hCG negative for pregnancy serum prolactin and TSH nurn~al;
decreased serum FSH: no tvi thdrawal hlccding after administration of progestero
ne.
MPI
PE
Labs
Imaging
Treatment
XR, skull: normal sella.
Advise patient to either gain enough weight tn restol-P Inenseq or take oral con
traceptives to prevent osteoparosis. The most common cause of secondary amenorrh
ea i s pregnancy. Women who are involved in vigorous physical exerrise ancl who
lose weight may present with a f~~nctional gonarlotmpin deficit. When hod! weigh
t f d l s > 15%of ideal weight. G n M secretion from the hypothalamus is decreas
ed, producing a sccondarv amenorrhea. The inhibitory effect of cstrogeens on hnn
e resnrpiion is also lust, predisposing patients ~u an increased risk for nsteop
ol-osis.
Discussion
SECONDARY AMENORRHEA
-
IDJCC
A 5-vear-old white mafe is hroughr ta the emergenrv room with a fracture of his
right forei~rmthat he sustainccl after falling ofT a co~~ch.
HPT
Thisisthefifthhonefracture~hat~hechilrlha~cu~tainedinthe past 2 years.
Bluish sdera; right leg and right arm slightly defru-rned (1-om poor healing c ~
past fracturi-s; mild kyphosis and scoliosis of f thoraric spiric: hypotonia an
d laxity of right leg a n d arm; partial conduction deafness in Imth ear?.
PE
Imaging
Micro Pathology
XR: fi-ar ture 01' radius and ulna with e.c-idencc of ostcopcnia.
Marked rhinning of bone cortices {E:C.G<HEI,L, ~"ORTFX);~nrl raref-action of'tra
beculae (due to abnormal synthesis of type I collagen); ;~ht~ormal softening or
tnofh enamel.
Supportive.
Treatment
Discussion
?\!w called hr-it1le h o n disease, osteojienesis imperfccta i~ an ~ autosomal-d
orninant dinorcles of npc I collragc-n svnihesis in wllich ~llerrs rlefici ent c
kssiricar inn clue in inadequate ortenid i
formation.
Atlas Link
r ??ED-052
'1
OSTEOGENESIS IMPERFECTA
ID/CC
A 43-war-old ma1r chuss plavcr is bro~lght Ihe crrlergencv to rnnrn complaining
of act11e nausea: lie hac vomited five times, reel5 vet-? lighthcadcd, and has a
severe headache.
I-le wctlt out clrinking lasr night to celebrate his virtorv ill a chew tourname
nt he attcncled Iast wcck in Mcxico. W l ~ i l e in Mexico, hc cr>ntr;iutcdacu t
c amrbiaqiu that is curren tl!, I~eing tl-u;itud with metronidaeole.
VS: n~at-kptl hrhvr;~rdia(HR 1 20); hypotension ( Bl' T)O/tiO). PE: an xi on^. r
lchyrlr;~ted, and confused with scverc iiausea.
HPI
PE
Labs
CU(;/LFTs: normal. Alnylase normal. ;213G/l,yter: ~nilcl hvpokalcmia aiid nietah
ol ic nlkaloqis (TIIIP 10 votniting)
Supportive, n' Fl~~icls. antiem~tics, disctlntinuar~ccc~f-alcohol.
Treatment
Discussion
Ett~;~nol clrgratlril hy alctll~vl is drhvrlrogenasc to ,~cetafdehycIc=, whir11
it1 t ~ t s n degraclrrl tu acetic acid ty a i ~ ( ~ t l l e ~ ir acetalrlehyde
rlel~ydrogcnxse. This acetaldehyde dehydrogenase is inhibitecl by clisulfir:t~-n
, resulting in the aceumlllation of acetaldehyde, which proditcer nausea. vonlit
irlg. headache. and hypotension ( A h - r , ~ n ~~..m.tx). -s~ .Metronidazole. s
olne ccphalc>.;porins, ;ui>cl other. cltAugs 111-rav~ h t a t n ~ . ; r -ike etY
ecr an l
tvhei~r r ~ ~ ~ s u t n r d cniicomit?ntl~ \\.it11 alcc >hol.
ID/CC
A premature (32-week4rl) rvhii~ lnalc inhnt i~ hrought ro thc i rlr~llsirp ra1-P
unit aZ'fel- a cesarean rlel i v ~ r y .
His
HEI~I
HPI
IEPT had hi rr1-tri~ne.;ter bleeding and cnnt~lctior~s tlmt
clid nut stop nit11 rcst a~lrl conscrvativr Ire-i~krr~rnt.
PE
VS: tachvl~nea. PE: chi l(l weighs 3.8 Ih; cyanosis; dyspnea: rises ;icccbcmrynl
11scle.cuf r - r h p i ~ ~ ~ inasal flaring. un:
Labs
IYBC;~: hypoxt,mia; h?.pc~-c;ipnia. Dccruased leci~liiii /sphi tlgonyelin (L/S)
rd-ltio (L/S ratio r ~ u r n ~ , ~ > y 1.3 tn 2.0 in 40% f I 2: of nrwhorns with
rrspir;~tor,y distress syndrc ~ i i i r ) .
CXR: hil;irer.~ll reticular pulmonary infiltrates and a telcctasis.
C~nerali;lrrl I elerrasi? in purple-color-ccl lui-lg: euhinop tiilic a
-D
Imaging
Gross Pathotogy
Treatment
Ve~~~iIarol-y suppo1.1,fluid. acid-hasc :lud clcc trolytc balance, i~ntiiilotir.
:nclministrstion of surfactant: *tnArrirls I,cli,i-c hill h to speetl l u ~ l g
rn;ttlirirv.
Respiratory distress syndrome r)F the iirlz~1~or.n llie in ost is
t
z 3
;o
Discussion
<
deficiency of surfactant, a lipapl-otein p r n d ~ ~ c e d type II hy pnputrlorl
.te rclls that contains the phospholipiti rlip;lltnitc>vl lecithin. Fct:rl lung
maluritv iri;~!. 1~ r n r a s t ~ ~byrtlh e L/S n r i o . ~ Tlw .iyi~tlmnir migI
11 h r pi-pveiilerl I)? giving betamethasone to PI'rAlI;1t)! rzlomrn. viiace n'p
e 11 pnelm-loc~te dirfcrentii~~iun cell i s dcpcndcnt on steroids. Complicr~tic)
i~.i~ c l ~ ~pc; l r t ~ n tduct~ts ir 31 arterinuif, ~ Z I ~ I I ~ O I Iair T ~
: I leaks, and hl-oncIloplr Imonnry d ~ s p l m i : ~ .
Atlas Link
I 1 T i PG-BC-083 1 _ _l
MYALINE MEMBRANE DISEASE
ED/CC
A 36-yeardd fcrnale nonsmoker visits her family doctor hecause st~e ha5 becorne
incrcasingly short of breath (DISPNEA); her symptoms first appeared only during
exercise but now occur even when ?he is at rest.
HPI
PE
She also compIains 01 frequent URTs and modcrate weight loss
Thin female with increased antemposterior diameter of chest (RARKFI.-SI IAFTT) C
HEST) ; decreased breath outi ids bilaterally; hyperresonance ro prrc~~ssion; re
tardation of expiratory flow.
Labs
CBC: increaqed hematocrit. PFTs: FlN,JF7'C < 75% (diaqostic of airflow obstructi
on). ECG: right ve~ltricular hypertrophy.
CXK: hyperlucent lung fieldq; flaitening of diaphragm and decreased lung marking
s at periphery
Imaging
Gross %thology
Destruction oCalveolaa-walls rliszal to thc terminal bronchiole with hyper.aerat
iun (EMFHTSEMA): panachar type (COTTON I ~ U LL:KG): more severe at lung bases.
I '
Treatment
S~nrlard treatment for COPD patients. Replacement therapy with a,-pt-oteascinhib
itor.
PoIl\it.nnts, cigarette smoke, and infections increase PMNs and macrophagcs in t
he lung and thus producr a number of proteoIytIc en~ymes. Damage tn lilng tiwur
due to these enzymes is controller1 by the globulin a]-antitrypsin,which inhihi~
s trypsin, neutrophil, elastase, and coltagenasc. A deficiency 01 this en7.yrne
causes excessive lung tissue destruction and panacinar emphysema (cigarette smok
ing is associated w i ~ h cer~trilohular ihe type). Paticnts may also drvelop li
ver damage.
Discussion
9
111
ALPHA-1-ANTITRYPSIN DEFICIENCY
ID/CC
A l."l-~~c;~s-old fcillalc. collcqe \;tr~tlr~itI > r - o ~ t~ irll ~ ernel-is. g
llle grllcv I-oonl ~ C ' L ~ L I CI[ numbness c ~ he1 rare nurl Sect rogcthcrW
f will1 ;I sensation of suffocation ;~firl stirr twisting of the hands ( { ; I R
I ~ ~ F F ~ ) <PIS\I): ~IIL=SP ~ V I I I ~ ~ ~ O;~ro\c~ o l l t ~ w a11 ~ y ~ I
I -\I. IIIS i~ T rrlcilt with !lei l>ovfrienrl. A friend rcllnl-ts tliat 111r p
atic.111 ,I history o f anxiety-inducetI 712% colitis. gastritis, and migraine.
HPE
PE
YS:marked tachypnea (RR -1-0): ~nclivral-rli;r IR 90) ; h ~ q ~ e r l e ~ j (1 s
iot~(13P 140,/!)0). I'E: ~ ~ a t i capprel~ensiveI I I C ~,111xi~ nt ; )IIF: p!i
yqical ~ s i i m olhc1-wisc ~lur-m;ll.
XRCis: low Pco2;respiratory alkdosis (cause o 'rctrr~ly) low l : hicarhnnatp Irc
t c-rttnp~nvar~ primarv Iowrriux or PC:(>,) for
Labs
Imaging
CXK: normal
fi~\,c- 13i11ir1ll breathe in and out of a bag or- givc 5 % C 0 , mis11II'P.
Treatment
~2
r
3
0
z
Discussion Ansicn~ h~~crvc.ntil;~liurii cc~rrlrxlot~ is ; t>cc~~r.i.eiiceERs. Th
c. in ti011s (~IIPFKI HNTTL.-\TI(. , c;lu\inq ;I I o w ~ ~ . i n g I't 0,: tllc
rcsull)u) crf' ing respi~ i~tor-y ;~lh;ilosis pr.odt~ces t~nstablc ;tn .rlcpnI;~
rizatir,n ~ f c ~ h rrl i ~ t aFegrnentq of~nflror l ncrvc? with ~ ~ ~ ~ ~ ~ ( l
lC-I;1TIV.i f l i l l l Alk;lln.iis ; ~ l u t , srt.; in nio~ic>ri torl\y>eiis;l
lrlr.y ;I rlrcl-caw in hicart>c,ri;ire 1el.t-1 1 0 H I ; I ~ I I I Z I 111-3 as
C ~ O S Cto 110r1na1 p o s ~ i h l ~ . ~I~ as x
3
<
~
HYPERVENTILATION
ID/CC
A 3l;year+>lddivorcce living in rtu-a1 Maine is h r n l ~ g lby amllui~ Iancr to
the ER with l x r two children, w h o were all found unconscious in her home hy
~niiitai-ypersonnel.
A rtceilt "EI Niiio"prorl~~ced weather thar resulted in a bad powcr failt~re: a
rcsnlt, she had been using charcoal and a as wooden stove inside her house Tor h
eating purposes.
HPI
PE
Skin bright red
(CHERRW~IED CYLVOSISJ ; pulse arrhytllmic; patient regains ronscic~usnes~ 0 1 3
after administration of 100% 'oxygen ~0 hut rcmains dro\\~sy, disoriented. and n
auseous and ccm~plains of a severe headache (clue to cerebral edema); hyperrefle
xia noted as well as positive Rorrlberg's tcst.
Labs
Increased carboxyhemodohin (> 25%)). 14RGs: n ~ e ~ ~ l > acidosis. olic
Imaging
Treatment
CT/MR: bilateral globus pallidus lesions.
Onc hunclrect percell1 oxygen, assiqtecl ver~~ilation necessary. if W y t ~ e r
- ~ hi~ r oxyacn cham her. a
Cotnmon sources of CO are car exhaust. pipes, and fires. Carbon mnnaxide has a r
n ~ ~ c h greater afinily for I~ernoglobin 111ano n ~ ~ g e(330 timcs morc). If
patient is pregnant, damage tn n the Cetuq i~ rleva~~ating (HhF has prater &niv
to CO than HbA}. Long-term sirle ei'r~ct.5 such a4 mernorv problems, lack of co
ordinarion, and even conm~lsions crJmmrrti after itltoxicatior~. are
Discussion
CARBON MONOXIDE P O I S O N I N G
fD/CC
.52-vr;11--oErl c ~ h u ~ u r~liitu male coines Icr 11i.i f i % l ~ l itloctol-
c o n !~ pli~ining ~ scrcrr, pair1 in thr- first rnetatarsophatangeal (MTP) r f
joint (PODAGRI) t l i ; ~ ~ hcg,in at night a f ~ r ii11a epi.;orIe of' hinge .
eating ar~rl drinking. H~itrl~~~it~toh~iiigan:~virlmeatcatcranrld~nk.;redwinerve
ry i ~ i g h t Hiri h i a r r v i s Ggnificant for relnnval of kidney stones . (
i u k arid 5tnnec).
HPI
PE
15: fever (38.2-C). PE: right M W joint red. h o t , nnrl s ~ v o l l c ~ l ; p
a i n l ~ lol aclive anrl pi~ssiuern~nrinti: ~ tophaceous ~lcposiwn lef'l i
r a i arltl olectanon Inlnitis.
Labs
Elevated serum ilric acid. UA: t~r*alr ci.vs~alr.Increaqed ESR. CBC: Irl~lrocv~l
sis ne11rl-npliilia. with KR: p ~ ~ r ~ r h e c lerosion5 i r l right big 1Icre
:II blTP,joirll, protluc-n~~i i n g "overhaa+q" v i c ~ ~ l r < .
Top1~iarc while. srlfi, I I I N ~ U I ~ I ~lljil<l;rq ni' 11r:lte deposits vcit1
1 cal. ciCic;irioils seen mainly it1 wlm-ial ~ n c n ~ b ~ u i c ~ , s h e i ~ t
h ~ . tciirlr~ri and car c:~rtili~gcs.
Imaging
Gross Pathology
Micro Pathology
?bpi I i and wiic~vialfluid aspirrl~ioiishow cl~itraticrs ~ i r i negativclv bir
efringent, needle-shaped crytals of irrir arid salts: gi;mt (.ell Ct~i.m;~iir~t~
neutl-opt~ilic with inliltration.
; O
I
3
C
Treatment
C : o i ~ l ~ i foi~aciite \ l a ~ r ;iwIiiiiii isler holil-IVu n ti1 negatirulv
~i w rli,~ri-Iiril c c ~ i -or. pain diwy)l>ear-.;:cornline wit11 nspirin or o
s NS,kllls. Lc>ng-tt=l-m treatmcn t with allr~pzu-in01iir~d/oi. pi obcnccid.
0
d
3
k
i rnet;~holi.jm\vith a rewlt ing inD ~ ~ C U S SC:c 1111~i s , rlisr)i.der. p t
~ r i n c ~O CI-rnscin scl-urn iiriu acid 1rr.t.l and d e p c ~ s i i~ is sp~era
l t ti~sues; 1 7 10 20"; nf raws m:iv rlrvelop ncpht-nlitliiasir. 111 l;tttb sli
igrs, 0 uratc deposits in thc kitlnt-v iI-na>, le;~rl rlii.onir ~ l y e l n n e
p h r i t i ~ , lo artrr ioli~r sclerosis, 11 ypertriision. srld renal railurc.
Atlas Links
m T
PM-BC-094
IM2-052A, IM2-052B
*
GOUT

Anda mungkin juga menyukai